Chapter 6: Carbohydrate Metabolism

Pataasin ang iyong marka sa homework at exams ngayon gamit ang Quizwiz!

*The answer is C.* Oxygen is part of the last oxidation reaction in the electron transport chain, which is used to generate ATP. Thus, a lack of oxygen will decrease ATP generation and slow down electron transport. In other words, less NADH will turn to NAD+. And if NAD+ is not regenerated, it cannot be used for pyruvate oxidation, and other intermediates in the TCA cycle.

How does lack of oxygen in the cell decrease TCA cycle activity? (A) synthesis of lactic acid from pyruvate is inhibited (B) increased malonyl CoA inhibits carnitine acyl transferase I. (C) electron transport cannot be used to regenerate NAD+ (D) epinephrine levels are increased (E) glycogenolysis is decreased, causing less requirement for the TCA cycle

*The answer is B.* This patient has Hurler syndrome, which is a mucopolysaccharidoses characterized by an accumulation of heparan and dermatan sulfate.

A 7-year old child has severe developmental and mental problems and is labeled as hyperactive. He is suffering gradual hearing loss and vision impairment. A photo of an ophthalmologic examination is shown below. His tissues are puffy, giving his face a "coarse" appearance and hepatosplenomegaly was noted. Histological studies indicated normal amounts of Sudan Black stainable material in tissues. However, lysosomes of tissues showed the presence of PAS positive materials, which were also found in urine. Which of the following is the accumulating materials? A. Glycogen B. Glycosaminoglycans C. GM2 Ganglioside D. Sphingomyelin E. Mixture of undigested molecules

*The answer is D.* Fructose 2,6-bisphosphate activates glycolysis by stimulating the rate-limiting enzyme - phosphofructokianse 1.

An increased concentration of fructose 2,6-bisphosphate in hepatocytes will have a positive regulatory effect on which of the following? (A) Gluconeogenesis and glucose-6-phosphatase (B) Gluconeogenesis and phosphoenolpyruvate carboxykinase (C) Glycolysis and glucokinase (D) Glycolysis and phosphofructokinase 1

*The answer is D.* GLUT is an abbreviation for glucose transporter, and describes a family of sugar transporters with varying distribution and activity. GLUT 4 is found in fat skeletal muscle, and heart, and mediates insulin-stimulated glucose uptake. Insulin acts to translocate GLUT 4 to the plasma membrane. GLUT 4 in skeletal and cardiac muscle is also stimulated by exercise through an insulin-independent pathway

A 27-year-old man runs a 10K race and drinks a 16-oz bottle of a sugar-containing drink. which transiently raises his blood glucose, triggering a release of insulin. The released insulin acts on which of the following to increase absorption of glucose by skeletal muscle? A. GLUT 1 B. GLUT 2 C. GLUT 3 D. GLUT 4 E. GLUT 5

*The answer is E.* Only option E is consistent with the constellation of clinical findings presented. Major clue is the positive Heinz body preparation.

A 44-year-old man from Limpopo Province in South Africa, living in the United States and receiving antibiotic therapy for a urinary tract infection, has a self-limiting episode of hemolysis, back pain, and jaundice. The peripheral blood smear reveals a nonspherocytic, normocytic anemia, and Heinz bodies are seen in some of his erythrocytes. Which of the following genetic deficiencies is most likely related to his hemolytic episode? A. Homocysteine methyltransferase B. Pyruvate kinase C. Dihydrofolate reductase D. Ferrochelatase E. Glucose 6-phosphate dehydrogenase

*The answer is A.* This woman is suffering from McArdle disease, also known as glycogen storage disease type V. In t his disease, there is a deficiency in skeletal muscle glycogen phosphorylase. Sufficient glycogen is stored inside these muscle cells, but in times of strenuous exercise, it cannot be broken down, leading to painful cramps, burning, and soreness in the affected muscles. In extreme cases, myoglobinuria can result. Glycogen phosphorylase is a necessary enzyme in the glycogenolysis pathway, which removes 1,4-glucosyl residues from the glycogen molecule, freeing glucose-1-phosphate

An 18-year-old woman presents to her family physician after she experiences severe cramping when running to catch a bus. She notes finding herself unable to jog with friends and recounts an episode when she had the same cramps and a burning sensation in her legs after he raced a friend for half a mile. The next day her legs felt weak and sore. She notes being able to take long, slow walks without experiencing cramping or burning, A muscle biopsy is obtained. Lab findings show lab findings of myoglobinuria and elevated creatine kinase. The enzyme that is most likely deficient in this disease is found in which part of the cell? A. Cytosol B. Golgi apparatus C. Mitochondria D. Proteasome E. Rough endoplasmic reticulum

*The answer is D.* I-cell disease is a lysosomal storage disease caused by a defect in the gene that initiaties the synthesis of the mannose 6-phosphate signal that targets acid hydrolases to the lysosome, resulting in accumulation of materials within the lysosome due to decreased degradation. None of the other choices relate in any way to I-cell disease or lysosomal function.

An 8-month-old boy with coarse facial features, skeletal abnormalities, and delays in both growth and development is diagnosed with I-cell disease based on his presentation, and on histologic and biochemical testing. I-cell disease is the consequence of: A. decreased production of cell-surface glycoproteins. B. an inability to ubiquitinate proteins. C. an inability to glycosylate proteins. D. incorrect targeting of lysosomal proteins. E. increased synthesis of proteoglycans.

*The answer is D.* Mannose 6-phosphate is placed on proteins that are targeted for the lysosome.

During the processing of particular N-linked glycoproteins, residues of mannose 6-phosphate are generated. Which of the following proteins is most likely to undergo this step in processing? (A) Apo B receptor (B) The citrate transport protein of the inner mitochondrial membrane (C) IgG (D) Lysosomal α-fucosidase (E) Mitochondrial isocitrate dehydrogenase

*The answer is B.* Red blood cells do not have mitochondria or a nucleus, therefore, metabolism of glucose in these cells occurs via glycolysis and the hexose monophosphate (HMP) shunt. Glycolysis provides energy for erythrocyte survival, whereas, the HMP shunt provides the reducing agent NADPH to prevent oxidant damage. In the initial oxidative portion of the HMP shunt, glucose 6~phosphate is converted to 6-phosphogluconolactone, and one molecule of NADPH is formed. This reaction is catalyzed by glucose 6-phosphate dehydrogenase. Yhe rate limiting enzyme of the HMP shunt In the second reaction of the oxidative portion of the HMP shunt, 6- phosphogluconolactone is hydrolyzed to ribulose 5-phosphate by the enzyme 6- phosphogluconate dehydrogenase producing a second molecule of NADPH. In erythrocytes, hydrogen peroxide produced by partial reduction of molecular oxygen is detoxified by glutathione peroxidase. Glutathione is oxidized during this reaction. The regeneration of reduced glutathione is carried out by the enzyme glutathione reductase using NADPH as an electron donor. NADPH in red blood cells is produced solely by the HMP shunt, and this is how the HMP shunt contributes to protecting red blood cells from oxidative stress. Defective generation of NADPH due to defects in the oxidative portion of the HMP shunt will increase the susceptibility of the RBCs to oxidative damage, and glutathione reductase deficiency will lead to similar clinical picture (choice B).

Red blood cells from a 24-year-old African American male who suffers from periodic hemolysis demonstrate a low activity of glucose-6 phosphate dehydrogenase. Deficiency of which of the following erythrocyte enzymes has the same pathophysiology as this patient's condition? A. Pyruvate kinase B. Glutathione reductase C. Bisphosphoglycerate maltose D. Hexokinase E. Transketolase

*The answer is C.* When a cell with the pentose phosphate pathway has need for more pentose phosphates , but no additional NADPH . the non-oxidative enzymes produce pentose phosphates from fructose-6-phosphate and glyceraldehyde-3-phosphate.

When a cell with the pentose phosphate pathway has need for more pentose phosphates , but no additional NADPH: (A) glucose-6-phosphate dehydrogenase is activated. (B) the oxidative and non-oxidative enzymes of the pentose phosphate pathway are active. (C) the non-oxidative enzymes produce pentose phosphates from fructose-6-phosphate and glyceraldehyde-3-phosphate. (D) all enzymes of glycolysis and pentose phosphate pathway are active. none are true.

*The answer is E.* Hepatic fatty acid oxidation generates energy in the postabsorptive period when pyruvate is being converted to OAA for glucose biosynthesis.

When fatty acid β-oxidation predominates in the liver, mitochondrial pyruvate is most likely to be A. carboxylated to phosphoenolpyruvate for entry into gluconeogenesis B. oxidatively decarboxylated to acetyl CoA for entry into ketogenesis C. reduced to lactate for entry into gluconeogenesis D. oxidatively decarboxylated to acetyl CoA for oxidation in Krebs cycle E. carboxylated to oxaloacetate for entry into gluconeogenesis

*The answer is A.* Succinyl-CoA synthase catalyzes substrate level phosphorylation by synthesizing GTP from GDP.

Which enzyme catalyzes substrate level phosphorylation? (A) succinyl-CoA synthase (B) citrate synthase (C) alpha-ketoglutarate dehydrogenase (D) Aconitase (E) succinate dehydrogenase

*The answer is D.* AMP and fructose 2,6-bisphosphate are activators of glycolysis and inhibitors of fructose 2,6-bisphosphatase.

Which of the following are activators of glycolysis AND inhibitors of gluconeogenesis? 1. AMP 2. ATP 3. Citrate 4. Fructose 2,6-bisphosphate 5. fructose 1,6- bisphosphate (A) 1 and 3 (B) 2 and 4 (C) 3 and 5 (D) 4 and 1 (E) 5 and 3

*The answer is E.* . High-energy phosphate bonds are added to the substrates of glycolysis at three steps in the pathway. Hexokinase—or, in the case of liver, glucokinase—adds phosphate from ATP to glucose to form glucose-6-phosphate. Strictly speaking, this is not always considered a step of the glycolytic pathway. Phosphofructokinase uses ATP to convert fructose-6-phosphate to fructose-1,6-phosphate. Using NAD+ in an oxidation-reduction reaction, inorganic phosphate is added to glyceraldehyde-3-phosphate by the enzyme glyceraldehyde-3-phosphate dehydrogenase to form 1,3-diphosphoglycerate. The enzymes phosphoglycerate kinase and pyruvate kinase transfer substrate highenergy phosphate groups to ADP to form ATP.

Which one of the following enzymes catalyzes high-energy phosphorylation of substrates during glycolysis? a. Pyruvate kinase b. Phosphoglycerate kinase c. Triose phosphate isomerase d. Aldolase e. Glyceraldehyde-3-phosphate dehydrogenase

*The answer is A.* Succinate is not a substrate coupled to the production of NADH. It is coupled to the production of FADH2.

A biochemistry graduate student isolated all enzymes of TCA cycle to produce NADH, oxidation of which of the following substrates in the citric cycle is not coupled to the production of NADH? (A) Succinate (B) Malate (C) α-ketoglutarate (D) Isocitrate (E) Pyruvate

*The answer is B.* Citrate, formed from oxaloacetate and acetyl CoA by the enzyme citrate synthase, inhibits phosphofructokinase and allosterically activates acetyl CoA carboxylase. Citrate synthase regenerates a molecule of CoA and is an important regulator of the tricarboxylic acid cycle. It is inhibited by adenosine triphosphate.

A metabolic process is pictured below. Which intermediate in this process inhibits the rate-limiting enzyme of glycolysis and activates the rate-limiting enzyme of fatty acid synthesis?

*The answer is A.* Non-glucose monosaccharldes (kg, galactose, mannose, fructose) enter the glycolytic pathway at different points as intermediates of glycolysls. Of these, fructose is the only one whose metabolites bypass phosphofructokinase. one of the key enzymes involved in regulating the rate of glycolysis. As a result, fructose is metabolized by the liver faster than the other monosaccharides and is rapidly cleared from the bloodstream following dietary absorption. Metabolism of fructose in the liver begins with phosphorylation by fructokinase to fructose-1 -phosphate (F1 ). Aldolase B can use both fructose-1.6-bisphosphate and F1P as substrates, it converts F1P into dihydroxyacetone phosphate (DHAP) and glyceraldehyde. Glyceraldehyde can be either phosphorylated to glyceraldehyde-3 phosphate by triokinase or converted to DHAP. DHAP is converted by triose phosphate isomerase to glyceraldehyde-3-phosphate, which continues down the glycolytic pathway. (Choices B, C, D, and E) Galactose-1-phosphate, glucose-1-phosphate, glucose-6-phosphate, and mannose-6-phosphate enter glycolysis upstream of phosphofructokinase, a major rate-limiting enzyme of glycolysis. This slows down the rate of their metabolism relative to fructose and its metabolites (eg, F1P). *Educational Objective:* Dietary fructose is phosphorylated in the liver to F1P and is rapidly metabolized because it bypasses PFK-1, the major rate-limiting enzyme of glycolysis. Other sugars (eg, glucose, galactose, mannose) enter glycolysis prior to PFK-1 and as a result are metabolized more slowly.

Biochemistry researchers are investigating the speed at which various carbohydrates are metabolized within the liver. They hypothesize that different monosaccharides delivered to the Iiver have different rates of intracellular metabolism. Which of the following substances is most likely to have the fastest rate of metabolism in the glycolytic pathway? (A) Fructose 1-Phosphate (B) Galactose 1-Phosphate (C) Glucose 1-Phosphate (D) Glucose 6-Phosphate (E) Mannose 6-Phosphate

*The answer is A.* Normally, 1 mole of ATP is used to convert 1 mole of glucose to 1 mole of glucose-6-phosphate and a second to convert 1 mole of fructose-6-phosphate to the bisphosphate. Two triose phosphates are produced by cleavage of fructose-1,6-bisphosphate. As the two triose phosphates are converted to pyruvate, four ATPs are generated; two by phosphoglycerate kinase and two by pyruvate kinase. Net, two ATPs are produced. If pyruvate kinase is completely deficient, two less ATPs will be produced, and thus the net ATP production will be zero. It is unlikely that the embryo would survive with a complete deficiency of this enzyme.

In an embryo with a complete deficiency of pyruvate kinase, how many net moles of ATP are generated in the conversion of 1 mole of glucose to 1 mole of pyruvate? (A) 0 (B) 1 (C) 2 (D) 3 (E) 4

*The answer is E.* The enzyme controlling the first step in gluconeogenesis is pyruvate carboxylase. It catalyzes the conversion of pyruvate to oxaloacetate. Pyruvate is absolutely dependent upon the presence of the allosteric effector acetyl CoA or a closely related acyl CoA for its function. Under conditions of high-energy charge and high levels of acetyl CoA, oxaloacetate is utilized for gluconeogenesis. If low amounts of ATP are present, oxaloacetate is consumed in the citric acid cycle.

The activity of pyruvate carboxylase is dependent upon the positive allosteric effector a. Succinate b. AMP c. Isocitrate d. Citrate e. Acetyl CoA

*The answer is B.* Lipoic acid is an intermediate acceptor of the acetyl group formed in the reaction. Pyruvate dehydrogenase complex catalyzes an irreversible reaction that is inhibited when the PDH (E1) component is phosphorylated. The enzyme complex is located in the mitochondrial matrix. Biotin is utilized by carboxylases.

The conversion of pyruvate to acetyl CoA and CO2: A. is reversible. B. involves the participation of lipoic acid. C. is activated when pyruvate dehydrogenase (PDH, E1) of the pyruvate dehydrogenase complex is phosphorylated by PDH kinase in the presence of ATP. D. occurs in the cytosol. E. depends on the coenzyme biotin.

*The answer is D.* Glucose is the primary source of energy for the central nervous system. A sudden decrease in circulating glucose levels will therefore impair ATP generation, in turn impeding cognitive function. If hypoglycemia persists, then the patient will slip into a coma and eventually die. This unfortunately common complication in people with type 1 diabetes is a consequence of over supplementation with insulin. By contrast, a sudden decrease in circulating insulin, glucagon, fatty acids, or triglyceride would have little immediate effect on cognitive function.

A 27-year-old man has been rushed to the emergency department following his sudden collapse and entry into a state of unconsciousness. Examination of personal belongings revealed the patient is an insulin-dependent diabetic. A rapid decline in which of the following humoral factors likely triggered the sudden collapse of the patient? (A) Insulin (B) Glucagon (C) Fatty acids (D) Glucose (E) Triglycerides

*The answer is C.* The patient is having a hypoglycemic attack. Glipizide is a sulfonylurea that stimulates insulin release from the pancreas and can cause hypoglycemia. Glucagon, epinephrine, and glucocorticosteroids are all released to raise blood glucose levels. The symptoms observed in the patient are side effects of epinephrine, acting in the autonomic nervous system. Testosterone levels would not be altered in the presence of glipizide.

A 50-year-old male with Type 2 diabetes is taking glipizide to help control his blood sugar levels. On one day he could not remember if he had taken the medication, so he accidently took a second dose of the drug. Two hours later, he suddenly develops irritability, tremors, tachycardia, and lightheadedness The symptoms the patient is experiencing are caused by which one of the following hormones? (A) Insulin (B) Glucagon (C) Epinephrine (D) Glucocorticosteroids (E) Testosterone

*The answer is A.* Phosphoenolpyruvate carboxykinase converts oxaloacetate to phosphoenolpyruvate. It is a gluconeogenic enzyme required for the conversion of amino acid carbons and lactate (but not phosphoenolpyruvate or glycerol) to glucose. Acetyl-CoA from the oxidation of fatty acids is not converted to glucose. Fructose can be converted to glucose without the need for PEPCK activity (fructose to fructose-1-phosphate, fructose-1-phosphate to DHAP and glyceraldehyde, glyceraldehyde to glyceraldehyde-3-phosphate, then the production of fructose-1,6-bisphosphate from DHAP and glyceraldehyde-3-phosphate, loss of phosphate to fructose-6 phosphate, isomerization to glucose-6-phosphate, then loss of phosphate to produce glucose).

A patient presented with a bacterial infection that produced an endotoxin that was found, after extensive laboratory analysis, to inhibit phosphoenolpyruvate carboxykinase. The patient would have very little glucose produced from which one of the following gluconeogenic precursors? (A) Alanine (B) Glycerol (C) Even-chain fatty acids (D) Phosphoenolpyruvate (E) Fructose

*The answer is B.* Glucose-6-phosphate is common to all pathways. It can be converted to glucose-1-phosphate for glycogen synthesis or go directly into the pentose phosphate pathway, or proceed through fructose-6-phosphate in glycolysis. UDP-glucose is formed from glucose-1-phosphate and can be used to form glycogen, lactose, glycoproteins, and glycolipids.

Which one of the following metabolites is used by all cells for glycolysis, glycogen synthesis, and the hexose monophosphate shunt pathway? (A) Glucose-1-phosphate (B) Glucose-6-phosphate (C) UDP-glucose (D) Fructose-6-phosphate (E) Phosphoenolpyruvate

*The answer is D.* The patient has the symptoms of beriberi, which is due to a thiamine deficiency. Of the enzymes listed, transketolase would be less active because it requires thiamine pyrophosphate as a cofactor. The other enzymes listed do not require cofactors except for the three dehydrogenases, which require either NAD+ or NADP+, depending on the enzyme.

A chronic alcoholic has recently had trouble with their ability to balance, becomes easily confused, and displays nystagmus. An assay of which of the following enzymes can determine a biochemical reason for these symptoms? (A) Isocitrate dehydrogenase (B) Transaldolase (C) Glyceraldehyde-3-phosphate dehydrogenase (D) Transketolase (E) Glucose-6-phosphate dehydrogenase

*The answer is A.* The glycolytic pathway has three key irreversible enzymes: hexokinase, phosphofructokinase, and pyruvate kinase. Under conditions of limiting cellular energy (low-energy charge), ADP and AMP accumulate and positively regulate phosphofructokinase. Under conditions of cellular "plenty," ATP and citrate, both negative effectors of phosphofructokinase, accumulate. When phosphofructokinase is inhibited, glucose-6-phosphate accumulates and shuts off hexokinase ATP, inhibiting the regulatory enzymes of glycolysis, while a lower energy charge actually stimulates glycolysis.

Which of the following is an allosteric effector that enhances activity of phosphofructokinase of the glycolytic pathway? (A) Adenosine monophosphate (AMP) (B) Citric acid (C) Adenosine triphosphate (ATP) (D) Glucose-6-phosphate (E) Glucose

*The answer is E.* Unlike the liver, skeletal muscle cannot export glucose into the circulation. Once glucose enters the myocyte, it is destined for use by that cell. Thus, intramyocellular glycogen is used as a fuel source by skeletal muscle and therefore cannot contribute to the hyperglycemia observed in uncontrolled type 1 diabetes. In contrast decreased insulin-mediated glucose utilization by skeletal muscle and adipose will contribute to hyperglycemia, as will decreased insulin-mediated suppression of lipolysis will indirectly contribute to hyperglycemia, by providing alternative, non glucose fuels (fatty acids and ketone bodies) for organs such as skeletal muscle and the liver.

Which of the following is the least likely to contribute to the hyperglycemia associated with uncontrolled type 1 diabetes? (A) Decreased skeletal muscle glucose uptake (B) Decreased adipose lipogenesis (C) Increased adipose lipolysis (D) Increased hepatic gluconeogenesis (E) Increased skeletal muscle glycogenolysis

*The answer is C.* Nitroprusside is metabolized to produce nitric oxide. NO, normally produced by the vascular endothelium, stimulates the cyclase in vascular smooth muscle to increase cGMP, activate protein kinase G, and cause relaxation.

A 40-year-old African American man is seen in the emergency room for a severe headache. His blood pressure is 180/110 mm Hg, and he has evidence of retinal hemorrhage. An infusion of nitroprusside is given. Which of the following enzymes is affected most directly by the active metabolite of this drug? A. Phospholipase A2 B. Cyclic AMP phosphodiesterase C. Guanylate cyclase D. Cyclic GMP phosphodiesterase E. Phospholipase C

*The answer is C.* The toxic agent (example, 2,4-dinitrophenol) would uncouple oxidative phosphorylation, leading to a fall in ATP levels, increased respiration, and increased substrate utilization.

A patient has been exposed to a toxic compound that increases the permeability of mitochondrial membranes for protons. Which of the following events in liver cells would you expect to occur? A. Increased ATP levels B. Increased F1F0 ATP synthase activity C. Increased oxygen utilization D. Decreased malate-aspartate shuttle activity E. Decreased pyruvate dehydrogenase activity

*The answer is B.* The conditions of the cell indicate that NADPH is required for fatty acid synthesis, but there is no need for ribose-5-phosphate, as the cells are in G0 phase and are not undergoing DNA synthesis (so nucleotides are not required). The HMP shunt will utilize the oxidative reactions to generate NADPH, and then the ribulose-5-phosphate produced will use the nonoxidative reactions to regenerate glucose-6-phosphate. For this to occur, transketolase, transaldolase, glucose-6-phosphate dehydrogenase (as the major oxidative enzyme of the pathway), and fructose-1,6-bisphosphatase all have to be active.

A researcher is studying cultured human hepatocytes and is examining the specific condition in which fatty acid synthesis is activated, but the cell remains in the G0 phase of the cell cycle. Under such conditions, what would be the activity state of the following enzymes?

*The answer is A.* Aldolase B is a key enzyme involved in fructose metabolism. Hereditary deficiency of aldolase B causes fructose intolerance. When foods high in fructose and sucrose are consumed in the presence of aldolase B deficiency, there is an accumulation of fructose-1-phosphate and a decrease in available phosphate. This results in inhibition of glycogenolysis and gluconeogenesis within the liver~causing symptoms of hypoglycemia, jaundice, cirrhosis, and vomiting. Treatment involves avoidance of dietary fructose and sucrose.

An 8-month-old boy is brought to the emergency department because of vomiting and lethargy. His mother says the boy ate applesauce and fruit juice for lunch. Physical examination reveals slight jaundice and tachycardia, but infantile cataracts are not present. Laboratory studies show a glucose level of 60 mg/dL and slightly elevated liver enzyme levels. Which of t he following enzyme deficiencies does this boy likely have? A. Aldolase B deficiency B. Fructokinase deficiency C. Galactokinase deficiency D. Galactose-1-phosphate uridyltransferase deficiency E. Glucose-6-phosphate dehydrogenase def iciency F. Lactase deficiency G. Phenylalanine hydroxylase deficiency

*The answer is A.* While most tissues cannot utilize fructose, the liver, kidneys, intestine, and adipose tissue can. Genetic fructokinase deficiency causes no symptoms. It can be detected by urine measurements of fructose that spills over into the urine. Unless care is taken, this could be misinterpreted as glucosuria, like that seen in diabetes, since both fructose and glucose are positive for a reducing-sugar test. Liver hexokinase rarely phosphorylates fructose to fructose-6-phosphate because the liver enzyme has a much greater affinity for glucose. However, adipose tissue hexokinase produces fructose-6-phosphate, which then can be acted upon by fructose-1-phosphate aldolase, which splits it into dihydroxyacetone phosphate and glyceraldehyde. Glyceraldehyde and dihydroxyacetone phosphate proceed through glycolysis or gluconeogenesis through the action of triose kinase. Under normal circumstances, liver fructokinase phosphorylates fructose to fructose-1-phosphate, and fructose-1-phosphate aldolase acts upon it.

Familial fructokinase deficiency causes no symptoms because (A) Hexokinase can phosphorylate fructose (B) Most tissues utilize fructose (C) Liver fructose-1-P aldolase is still active (D) Excess fructose does not escape into the urine (E) Excess fructose spills into the bowel and is eliminated in feces

*The answer is D.* Lactase and maltase are intestinal enzymes not found in the serum. Therefore, ingested lactose is degraded, but injected lactose is not. If hepatic galactokinase is absent, the galactose segment of the lactose is not metabolized, but the glucose segment of the lactose can still be metabolized.

Following the intravenous injection of lactose into a rat, none of the lactose is metabolized. However, ingestion of lactose leads to rapid metabolism of this disaccharide. The differencein these observations is a result of: A. the presence of lactase in the serum. B. the absence of hepatic galactokinase. C. the absence of maltase in the serum. D. the presence of lactase in the intestine.

*The answer is B.* Infection is accompanied by generation of oxidants which the erythrocyte cannot cope with as its antioxidant defences are low because of the absence of glucose 6 phosphate dehydrogenase which would normally lead to the production of NADPH and use the reducing power of NADPH to maintain antioxidant action through production of reduced glutathione. Immunoglobulins are not involved nor do toxins attack the membrane.

Haemolytic anaemia can be triggered in people deficient in glucose 6 phosphate dehydrogenase by infection. What is the mechanism? (A) Infection leads to increased production of immunoglobulins which attack the erythrocyte. (B) Infection is accompanied by generation of oxidants which destroy the erythrocyte membrane. (C)Infection leads to weakening of erythrocyte cell membranes by the attachment of bacterial toxins. (D) Infection compromises the immune system which is unable to protect the erythrocyte.

*The answer is B.* If NADPH is needed for biosynthetic reactions but ribose-5-P is not needed, the cell will complete the oxidative phase, the nonoxidative phase, and gluconeogenesis.

If NADPH is needed for biosynthetic reactions but ribose-5-P is not needed, which of the following best represents the net reaction of the pentose phosphate pathway? (A) 4 Fruc-6-P + 2 glyceraldehyde-3-P → 6 ribose-5-P (B) 3 Gluc-6-P + 6 NADP+ → 6 NADPH + 3 CO2 + 2 Fruc-6-P + 1 glyceraldehyde-3-P (C) Gluc-6-P + 2 NADP+ → 2 NADPH + CO2 + ribose-5-P (D) both b and c will occur (E) None of the above

*The answer is D.* 6 molecules of ATP are required to convert 2 molecules of lactate into glucose in the mammalian liver.

The number of ATP molecules required to convert 2 molecules of lactate into glucose in mammalian liver is (A) 2 (B) 4 (C) 5 (D) 6

*The answer is C.* A low NAD+/NADH ratio limits the rates of the NAD+-requiring dehydrogenases. A low ATP/ADP or GTP/GDP ratio stimulates the cycle. AMP does not directly affect the cycle.

Which one of the following conditions decreases the oxidation of acetyl CoA by the citric acid cycle? A. A low ATP/ADP ratio B. A low NADH concentration due to rapid oxidation to NAD+ through the respiratory chain C. A low NAD+/NADH ratio D. A high concentration of AMP E. A low GTP/GDP ratio

*The answer is D.* The child has a form of galactosemia. The elevated galactitol enters the lens of the eye, and is trapped. The difference in osmotic pressure across the lens of the eye leads to cataract formation. Galactose is phosphorylated by galactokinase to galactose 1-phosphate, which reacts with UDP-glucose in a reaction catalyzed by galactose-1 phosphate uridylyl transferase to form UDP-galactose and glucose 1 phosphate. An epimerase converts UDP-galactose to UDP-glucose. Deficiencies in either galactokinase (nonclassical) or galactose-1 phosphate uridylyl transferase (classical) result in galactosemia, with elevated levels of galactose and galactitol (reduced galactose) in the blood. An intracellular measurement of galactose-1-phosphate can allow a definitive diagnosis to be obtained (such levels would be nonexistent if the defect were in galactokinase, and the levels would be greatly elevated if the galactose-1-phosphate uridylyl transferase enzyme were defective).

A 1-year-old child, on a routine well child visit, was discovered to have cataract formation in both eyes. Blood work demonstrated elevated galactose and galactitol levels. In order to determine which enzyme might be defective in the child, which intracellular metabolite should be measured? (A) Galactose (B) Fructose (C) Glucose (D) Galactose-1-phosphate (E) Fructose-1-phosphate (F) Glucose-6-phosphate

*The answer is B.* The child has McArdle's disease due to a lack of muscle phosphorylase activity. In this disorder, muscle glycogen cannot be oxidized during exercise and glycogen accumulates within the muscle. As such, lactate levels would be low, and the person could not tolerate intense exercise of brief duration and would rely on fuels from the blood (glucose, fatty acids, and ketone bodies) for energy. The person could engage in mild exercise of long duration, using these blood fuels. The liver would not be affected because it contains a different phosphorylase isozyme. While a muscle PFK-1 mutation can also lead to these symptoms (Tarui's disease), a PFK-1 deficiency also leads to hemolytic anemia, due to the poor ATP yield from glycolysis in the red blood cells due to the lack of PFK-1 activity in those cells. Hemolytic anemia was not observed in the patient described in this question. Glucose-6-phosphatase is only present in the liver (not in the muscle), and a mutation in glucose-6-phosphatase will affect fasting blood glucose levels, but not exercise tolerance.

A 10-year-old boy displays exercise intolerance, complaining of leg and arm pain after only a short period of exercise. The pain is worse if anaerobic activities are being attempted; for the more aerobic activities, the boy can complete the task, but at a much slower rate than others in the class. Hemolytic anemia was not observed in this child at any time. A muscle biopsy displayed elevated levels of glycogen in the muscle. A stress test in which the boy squeezed a rubber ball over and over again demonstrated a reduced production of lactate as compared to someone who did not display exercise intolerance. The most likely enzyme deficiency in this boy is which one of the following? (A) Liver phosphorylase (B) Muscle phosphorylase (C) Liver glucose-6-phosphatase (D) Muscle glucose-6-phosphatase (E) Liver PFK-1 (F) Muscle PFK-1

*The answer is B.* Glucagon aims to increase the levels of glucose in the blood. One way to do this is to break down glycogen stores through glycogenolysis, catalyzed by glycogen phosphorylase.

A 15-year-old type 1 diabetic faints after injecting himself with insulin. He is administered glucagon and rapidly recovers consciousness. Glucagon induces activity of (A) Glycogen synthase (B) Glycogen phosphorylase (C) Glucokinase (D) Hexokinase (E) UDP Glucose Pyrophosphorylase

*The answer is B.* Blood glucose decreases because insulin stimulates the transport of glucose into muscle and adipose cells and stimulates the conversion of glucose to glycogen and triacylglycerols in the liver. Ketone bodies are not made from blood glucose. During fasting, when the liver is producing ketone bodies, it is also synthesizing glucose. Carbon for ketone body synthesis comes from fatty acids. Insulin stimulates glycogen synthesis, not glycogenolysis. Muscle glycogen is not converted to blood glucose.

A 16-year-old patient with Type 1 diabetes mellitus was admitted to the hospital with a blood glucose level of 400 mg/dL. (The reference range for blood glucose is 80 to 100 mg/dL.) One hour after an insulin infusion was begun, her blood glucose level had decreased to 320 mg/dL. One hour later, it was 230 mg/dL. The patient's glucose level decreased because the infusion of insulin led to which one of the following? (A) The stimulation of the transport of glucose across the cell membranes of the liver and brain (B) The stimulation of the conversion of glucose to glycogen and triacylglycerol in the liver (C) The inhibition of the synthesis of ketone bodies from blood glucose (D) The stimulation of glycogenolysis in the liver (E) The inhibition of the conversion of muscle glycogen to blood glucose

*The answer is C.* The infant has Pompe disease, a loss of liver α-glucosidase activity. This is glycogen storage disease II. The finding of normal glycogen structure eliminates liver debranching and branching activities as being deficient. The missing enzyme is a lysosomal enzyme, and non-degraded glycogen accumulates in the lysosome, interfering with lysosomal function (hence, a lysosomal storage disease). The malfunctioning of the lysosomes is what leads to the muscle and liver problems. A defect in glycogen phosphorylase (liver) would lead to fasting hypoglycemia, and an enlarged liver, but not the muscle problems exhibited by the child. A defect in glycogen synthase would also lead to fasting hypoglycemia, but would not lead to severe muscle and liver disease. Additionally, in an individual with a defect in glycogen synthase, glycogen would not be found in the liver biopsy since it could not be formed. The figure summarizes steps involved in glycogen degradation, and the glycogen storage disease that results if an enzyme is defective.

A 3-month-old infant was brought to the pediatrician due to muscle weakness (myopathy) and poor muscle tone (hypotonia). Physical exam revealed an enlarged liver and heart, and heart failure. The infant had always fed poorly, had failure to thrive, and had breathing problems. He also had trouble holding up his head. Blood work indicated early liver failure. A liver biopsy indicated that glycogen was present and of normal structure. A potential defect in this child is which of the following? (A) Liver glycogen phosphorylase (B) Liver glycogen synthase (C) Liver α-glucosidase (D) Liver debranching enzyme (E) Liver branching enzyme

*The answer is C.* The child has the symptoms of von Gierke's disease, which is due to a lack of glucose 6-phosphatase activity. In this disorder, neither liver glycogen nor gluconeogenic precursors (e.g., alanine and glycerol) can be used to maintain normal blood glucose levels. The last step (conversion of glucose-6-phosphate to glucose) is deficient for both glycogenolysis and gluconeogenesis. Muscle glycogen cannot be used to maintain blood glucose levels because muscle does not contain glucose 6-phosphatase. A defective liver glycogen phosphorylase (Her's disease) will not affect the ability of the liver to raise blood glucose levels by gluconeogenesis. In addition, the lack of liver glycogen phosphorylase does not lead to lactic and uric acid accumulation, although mild fasting hypoglycemia can be observed. Defects in liver glycogen synthase (type 0 glycogen storage disease) will lead to an early death, with hypoglycemia and hyperketonemia observed. Muscle does not contribute to blood glucose levels, so a defect in muscle glycogen phosphorylase (McArdle's disease) will not lead to the observed symptoms, but will lead to exercise intolerance. A defect in pyruvate kinase will lead to hemolytic anemia, but not the other symptoms observed in the patient.

A 3-month-old infant was cranky and irritable, became quite lethargic between feedings, and began to develop a potbelly. A physical exam demonstrated an enlarged liver, while blood work taken between feedings demonstrated elevated lactate and uric acid levels, as well as hypoglycemia. This child most likely has a mutation in which one of the following enzymes? (A) Liver glycogen phosphorylase (B) Glycogen synthase (C) Glucose 6-phosphatase (D) Muscle glycogen phosphorylase (E) Pyruvate kinase

*The answer is E.* The brain can only use glucose or ketone bodies as an energy source. Even though the heart can use lactate for energy, the brain does not do so. If glucose levels are low, the only available substrate would be ketone bodies. Fatty acids will not cross the blood-brain barrier and are not a good energy source for the brain. The liver will convert fatty acids to ketone bodies for use by the brain. Amino acids are a good source of carbon for gluconeogenesis, but the brain does not oxidize amino acids at an appreciable rate. Glycerol cannot be used by the brain as an energy source as the brain lacks glycerol kinase, a necessary enzyme in the metabolism of glycerol. The treatment for a GLUT1 deficiency is a ketogenic diet-one high in fats such that ketone bodies are continuously generated to provide fuel for the brain.

A 3-month-old infant, who was experiencing seizures, was diagnosed with a GLUT1 deficiency, resulting in reduced glucose uptake into the brain. As a result, which one of the following substrates was providing energy for the brain? (A) Lactate (B) Amino acids (C) Fatty acids (D) Glycerol (E) Ketone bodies

*The answer is D.* The patient has HFI, which is due to a mutation in aldolase B. Sucrose would still be cleaved by sucrase, thus it would not increase in the stool. Fructose would not be metabolized normally, therefore it would be elevated in the blood and urine. Aldolase B would not cleave fructose 1-phosphate, thus its levels would be elevated and the product, glyceraldehyde, would not be produced.

A 3-year-old girl has been a fussy eater since being weaned, particularly when fruit is part of her diet. She would get cranky, sweat, and display dizziness, and lethargy, after eating a meal with fruit. Her mother noticed this correlation, and as long as fruit was withdrawn from her diet the child did not display such symptoms. The problems the girl exhibits when eating fruit is most likely due to which one of the following? (A) Decreased levels of fructose in the blood (B) Elevated levels of glyceraldehyde in liver cells (C) High levels of sucrose in the stool (D) Elevated levels of fructose-1-phosphate in liver cells (E) Decreased levels of fructose in the urine

*The answer is A.* Epinephrine, via binding to its receptor, activates a Gs protein, which binds to and activates adenylate cyclase. Adenylate cyclase will convert ATP to cAMP. As cAMP levels increase, the cAMP binds to the regulatory subunits of protein kinase A, allowing the regulatory subunits to be released from the catalytic subunits. This activates protein kinase A, which then phosphorylates both glycogen synthase and phosphorylase kinase. Glycogen phosphorylase is activated by phosphorylation by phosphorylase kinase. Thus, of the events listed, activation of adenylate cyclase is the initial event.

A 50-year-old male with Type 2 diabetes is taking glipizide to help control his blood sugar levels. On one day he could not remember if he had taken the medication, so he accidently took a second dose of the drug. Two hours later, he suddenly develops irritability, tremors, tachycardia, and lightheadedness In response to the overdose of glipizide, the patient has released hormones that will lead to glucose being released by the liver. This occurs through an initial activation of which one of the following liver enzymes? (A) Adenylate cyclase (B) Protein kinase A (C) Glycogen synthase (D) Phosphorylase kinase (E) Glycogen phosphorylase

*The answer is B.* The patient has become hypoglycemic due to excessive release of insulin from the pancreas. Glipizide (glucotrol) is a sulfonylurea drug that stimulates insulin release from the pancreas. If taken in excess, the insulin will promote fat and muscle cells to take up glucose from the circulation, leading to hypoglycemia and insufficient blood glucose levels for normal brain function. Lactic acidosis may result from such an overdose, but it would be secondary to the hypoglycemic symptoms observed. Elevated ammonia levels would not occur, as glipizide does not alter amino acid metabolism. The high levels of insulin released by the drug would inhibit fatty acid release from the adipocytes, and therefore the precursors for ketone body synthesis are not available, and ketoacidosis would not occur

A 50-year-old male with Type 2 diabetes is taking glipizide to help control his blood sugar levels. On one day he could not remember if he had taken the medication, so he accidently took a second dose of the drug. Two hours later, he suddenly develops irritability, tremors, tachycardia, and lightheadedness. The patient is experiencing which one of the following due to his drug overdose? (A) Hyperglycemia (B) Hypoglycemia (C) Lactic acidosis (D) Ketoacidosis (E) Hyperammonemia

*The answer is B.* Glucagon is the major catabolic hormone that counters insulin's effects. It can raise blood glucose through stimulation of gluconeogenesis and glycogenolysis. Adding insulin would exacerbate the metabolic situation, as excessive insulin is causing the problem. Amylin suppresses glucagon action, and would not overcome the effects of the high insulin levels. Epinephrine can help counter insulin and raise blood glucose, but would be dangerous in a patient with known coronary artery disease (CAD) and palpitations.

A 50-year-old male with a history of coronary artery disease presents with episodes of lightheadedness, tremors, palpitations, hunger, headache, weakness, and confusion. He is fine between episodes. He had such an episode at his last doctor's visit, at which time blood was drawn for various analyses. The lab results revealed high insulin, high C-peptide, and low blood glucose levels. In order to treat the patient's symptoms during an episode, which one of the following would be safest to administer? (A) Epinephrine (B) Glucagon (C) Insulin (D) Amylin (E) Testosterone

*The answer is A.* The symptoms and lab results are classic for insulinoma. An insulinoma is a tumor of the pancreatic β cells that episodically releases large amounts of insulin. At those times, the patient experiences the symptoms of hypoglycemia. A pheochromocytoma is a tumor of the adrenal gland that episodically releases epinephrine and norepinephrine throughout the body. A pheochromocytoma would not lead to hypoglycemia (epinephrine stimulates the liver to export glucose), or high insulin or C-peptide levels. If the patient were injecting insulin, the C-peptide should be low, as exogenous insulin lacks the C-peptide. Neither a liver tumor nor a carcinoid tumor would release insulin to the blood.

A 50-year-old male with a history of coronary artery disease presents with episodes of lightheadedness, tremors, palpitations, hunger, headache, weakness, and confusion. He is fine between episodes. He had such an episode at his last doctor's visit, at which time blood was drawn for various analyses. The lab results revealed high insulin, high C-peptide, and low blood glucose levels. Which of the following would be most consistent with his symptoms and lab values? (A) Insulinoma (B) Pheochromocytoma (C) Exogenous insulin injection (D) Carcinoid tumor (E) Liver cancer

*The answer is C.* The patient described in the vignette has lactic acidosis secondary to hypoperfusion of his tissue and subsequent metabolism of glucose via anaerobic glycolysis. In the process of glycolysis, glucose is ultimately converted to pyruvate. Glycolysis takes place in the cytosol, and oxidative phosphorylation, which requires the presence of oxygen, occurs in the mitochondria. The fate fo pyruvate generated during glycolysis is dependent on the presence of oxygen. When inadequate amounts of oxygen are present in the tissues, pyruvate is converted to lactate by the enzyme lactate dehydrogenase in order to regenerate NAD+ from NADH. Thus, under anaerobic conditions, increased amounts of lactate by the enzyme lactate dehydrogenase. Increased lactate levels result in metabolic acidosis, and patients with metabolic acidosis will attempt to compensate by causing a respiratory alkalosis. This is accomplished by hyperventilation and loss of CO2 (an acid when dissolved in the blood). In the presence of oxygen, pyruvate is preferentially converted to acetyl-CoA by the enzyme pyruvate dehydrogenase. Acetyl-CoA enters the mitochondria and undergoes oxidative phosphorylation.

A 64-year-old male hospitalized with severe abdominal pain and hypertension begins to hyperventilate. Laboratory testing reveals metabolic acidosis, an increased anion gap and a high plasma lactate level. This patient's findings are best explained by a low activity of: (A) Pyruvate kinase (B) Lactase drhydrogenase (C) Pyruvate dehydrogenase (D) Pyruvate carboxylase (E) Enolase

*The answer is A.* The patient has osteoarthritis, and wants to use glucosamine/chondroitin sulfate to provide cushioning in the joint. These molecules are proteoglycans, which consist of long, linear chains of glycosaminoglycans attached to a core protein. Each chain is composed of repeating disaccharides, but disaccharides are, by definition, only 2 sugars. A polyol is a polyalcohol. A glycolipid is a sphingolipid, and does not contribute to joint stability. The typical glycoproteins are not found in the joints, nor are they available as oral supplements as the proteoglycans are.

A 65-year-old patient complains of occasional swelling, pain, and a scraping sensation in the knees. X-rays show a narrowing of the joint space. The patient only wants to take "natural" oral products to help reverse this condition. The products available for consumption are examples of which one of the following types of compounds? (A) Proteoglycan (B) Polyol (C) Glycolipid (D) Disaccharide (E) Glycoprotein

*The answer is D.* . The only intermediate included on the list that the pathway of gluconeogenesis from glycerol has in common with the pathway of gluconeogenesis from lactate is glucose-6-phosphate. Glycerol enters gluconeogenesis as DHAP. Therefore, it bypasses the other compounds (pyruvate, oxaloacetate, malate and phosphoenolpyruvate) through which the carbons of lactate must pass on its pathway to glucose synthesis.

A common intermediate in the conversion of glycerol and lactate to glucose is which one of the following? (A) Pyruvate (B) Oxaloacetate (C) Malate (D) Glucose-6-phosphate (E) Phosphoenolpyruvate

*The answer is B.* . The boy lacks glucose-6-phosphate dehydrogenase activity (an X-linked disorder) and, in response to the drug (most likely primaquine), has developed a hemolytic anemia due to an inability to regenerate reduced glutathione to protect red blood cell membranes from oxidative damage. The yellow in the eyes is due to a buildup of bilirubin, as the released hemoglobin from the red blood cells cannot be adequately metabolized by the liver, and converted to the more soluble diglucuronide form. The abdominal pain may be due to bilirubin stones being formed in the gall bladder. Glucose-6-phosphate dehydrogenase produces NADPH in the red blood cells, which is required for glutathione reductase, the enzyme that converts oxidized glutathione to reduced glutathione. The drug does not block heme synthesis, the absorption of iron, or affect radical oxygen species metabolism (superoxide dismutase or catalase).

A couple and their two sons were going to visit Panama in the summer, and obtained drugs from friends (who had these leftover from their trip the year before) to help combat the possibility of acquiring the malarial parasite while in that country. The family members took one drug every day while visiting, then, once they arrived back home, they had to continue the drug treatment for an additional week. During the trip, one of the sons complained of being tired; after the family returned home, he was even more tired and complained of pain in his upper abdomen. He was taken to the emergency department where it was determined that he was anemic. Careful examination demonstrated a slight yellowing in the whites of his eyes. In the presence of the drug, the boy had difficulty in carrying out which one of the following reactions? (A) The synthesis of heme (B) The conversion of oxidized glutathione to reduced glutathione (C) The absorption of iron, reducing hemoglobin synthesis (D) The conversion of superoxide to oxygen (E) The conversion of hydrogen peroxide to oxygen

*The answer is D.* The increased concentrations of pyruvate, lactate, and alanine indicate that there is a block in the pathway leading from pyruvate towards the TCA cycle. A deficiency in pyruvate dehydrogenase would lead to a buildup of pyruvate. Pyruvate has 3 fates other than conversion to acetyl-coA by pyruvate dehydrogenase: conversion to oxaloacetate by pyruvate carboxylase, reduction to lactate by lactate dehydrogenase, and transamination to the amino acid alanine. Thus, because pyruvate builds up, an increase in lactate and alanine would be expected if pyruvate dehydrogenase was deficient. It is also important to keep in mind that Pyruvate dehydrogenase catalyzes the irreversible conversion of pyruvate to acetyl-CoA. If it's absent, pyruvate will be used in other pathways instead. Pyruvate will be converted to alanine via alanine aminotransferase and to lactate via lactate dehydrogenase.

A full-term female infant failed to gain weight and showed metabolic acidosis in the neonatal period. A physical examination at 6 months showed failure to thrive, hypotonia, small muscle mass, severe head lag, and a persistent acidosis (pH 7.0-7.2). Blood lactate, pyruvate, and alanine were greatly elevated. Treatment with thiamine did not alleviate the lactic acidosis. Which of the following enzymes is most likely deficient in this patient? (A) Alanine aminotransferase (B) Phosphoenolpyruvate carboxykinase (C) Pyruvate carboxylase (D) Pyruvate dehydrogenase (E) Pyruvate kinase

*The answer is D.* People who abuse alcohol frequently do not eat while binge drinking, so it is most likely that his liver glycogen stores became depleted and could not increase his blood glucose levels. The metabolic stress leads to the increase in secretion of epinephrine and other hormones that mobilize fatty acids from store triglycerides in adipose cells. These fatty acids undergo beta-oxidation in the liver but are converted to ketone bodies because of the inhibition of the TCA cycle by high levels of NADH produced by the oxidation of ethanol first to acetaldehyde and acetate. Key gluconeogenic dehydrogenases are also inhibited by the elevated levels of NADH, including lactate dehydrogenase, glycerol-3-phosphate dehydrogenase, and malate dehydrogenase.

A known person who abuses alcohol is found lying semi conscious at the bottom of a stairwell with a broken arm by his landlady, who called an ambulance to take him to the emergency room. Initially laboratory studies showed a relatively large anion gap of 34 (normal range: 9-15). His blood alcohol was elevated at 245 mg/dL (intoxication level: 150-300 mg/dL), and his blood glucose was 38 mg/dL (low normal). The patient's large anion gap and hypoglycemia can best be explained by which of the following? (A) Decreased secretion of glucagon (B) Increased secretion of insulin (C) Increased urination resulting from the diuretic effect of alcohol (D) Inhibition of dehydrogenase enzymes by NADH (E) Inhibition of glycogenolysis by ethanol

*The answer is D.* AMP will activate muscle glycogen phosphorylase b allosterically, allowing glycogen degradation to begin before any hormonal signal has reached the muscle. The addition of epinephrine to the muscle requires activation of adenylate cyclase to initiate glycogen degradation, and adenylate cyclase has been inactivated in this cell line. Muscle lacks glucagon receptors, so cannot respond to this hormone. An increase in intracellular calcium would lead to glycogen degradation (via activation of phosphorylase kinase b), but magnesium does not have the same effect as calcium. Increases in ADP levels will not activate glycogen phosphorylase b; the allosteric activator is specific for AMP. The table below summarizes the allosteric interactions involved in glycogen metabolism.

A muscle cell line has been developed with a nonfunctional adenylate cyclase gene. Glycogen degradation can be induced in this cell line via which of the following mechanisms? (A) Addition of glucagon (B) Addition of epinephrine (C) Increase in intracellular magnesium (D) Increase in intracellular AMP (E) Increase in intracellular ADP

*The answer is B.* The major difference between Type 1 and Type 2 diabetes is the ability of the body to produce endogenous insulin. Patients with Type 1 diabetes do not produce insulin, whereas patients with Type 2 diabetes do produce insulin, but have difficulty responding to the insulin. When insulin is synthesized as preproinsulin, it is then modified and the C-peptide is removed from the molecule, to produce active insulin. Persons with Type 1 diabetes would be lacking C-peptide (exogenous insulin that is injected also lacks C-peptide), whereas persons with Type 2 diabetes would be producing C-peptide. The levels of glucagon and epinephrine would be similar in both types of diabetes. Since the patient is on insulin already, measuring the level of mature insulin in the blood would be unhelpful. HbA1c levels measure glycemic control over the past 6 weeks, and are usually elevated in both types of diabetes. Measuring HbA1c would not enable one to distinguish between Type 1 and Type 2 diabetes in this patient.

A patient has been diagnosed with Type 1 diabetes in their late teens and is being treated with exogenous insulin, but a second physician is not convinced that the patient has Type 1 diabetes, but rather has Type 2 diabetes. A measurement of which one of the following would allow the physician to determine which diagnosis is correct? (A) Insulin levels (B) C-peptide levels (C) Glucagon levels (D) Epinephrine levels (E) HbA1c levels

*The answer is A.* By 2 to 3 hours after a high-carbohydrate meal, the patient's glycogen stores would be filled. Glucagon would stimulate glycogenolysis, and blood glucose levels would rise. Gluconeogenesis would still be impaired, but since glycogen levels are high, the liver would be able to export significant amounts of glucose.

A patient presented with a bacterial infection that produced an endotoxin that was found, after extensive laboratory analysis, to inhibit phosphoenolpyruvate carboxykinase. Administration of a high dose of glucagon to this patient 2 to 3 hours after a high-carbohydrate meal would result in which one of the following? (A) A substantial increase in blood glucose levels (B) A decrease in blood glucose levels (C) Have little effect on blood glucose levels

*The answer is C.* Thirty hours after a meal, liver glycogen is normally depleted, and blood glucose level is maintained solely by gluconeogenesis after this time. However, in this case, a key gluconeogenic enzyme is inhibited by an endotoxin. Therefore, gluconeogenesis will not occur at a normal rate and glycogen stores will be depleted more rapidly than normal. Blood glucose levels will not change significantly if glucagon is administered after 30 hours of fasting.

A patient presented with a bacterial infection that produced an endotoxin that was found, after extensive laboratory analysis, to inhibit phosphoenolpyruvate carboxykinase. Administration of a high dose of glucagon to this patient 30 hours after a high-carbohydrate meal would result in which one of the following? (A) A substantial increase in blood glucose levels (B) A decrease in blood glucose levels (C) Have little effect on blood glucose levels

*The answer is D.* Insulin stimulates glucose transport into muscle and adipose cells through mobilization of GLUT4 transporters from internal vesicles to the cell surface. Insulin does not significantly stimulate glucose transport into tissues such as liver, brain, or RBCs, which utilize different variants of the glucose transporters. Only GLUT4 is insulin-responsive.

A patient with Type 1 diabetes self-injected insulin prior to their evening meal, but then was distracted and forgot to eat. A few hours later, the individual fainted, and after the paramedics arrived they did a STAT blood glucose level and found it to be 45 mg/dL. The blood glucose level was so low because which one of the following tissues assimilated most of it under these conditions? (A) Brain (B) Liver (C) Red blood cells (D) Adipose tissue (E) Intestinal epithelial cells

*The answer is C.* The woman will be able to breastfeed her baby because she can produce lactose from amino acids and other carbohydrates. She will not have to eat pure galactose, or even lactose, to do so. Glucose, which can be provided by gluconeogenesis or obtained from the diet, can be converted to UDP-galactose (glucose → glucose-6-phosphate → glucose-1-phosphate → UDP-glucose → UDP-galactose). UDP-galactose reacts with free glucose to form lactose. α-Lactalbumin is a protein that serves as the modifier of galactosyltransferase, which catalyzes this reaction. The amino acids of α lactalbumin can be used to produce glucose, but the immediate products of α-lactalbumin degradation are not lactose. Carbohydrates cannot be synthesized from fats.

A pregnant woman who has a lactase deficiency and cannot tolerate milk in her diet is concerned that she will not be able to produce milk of sufficient caloric value to nourish her baby. The best advice to her is which one of the following? (A) She must consume pure galactose in order to produce the galactose moiety of lactose. (B) She will not be able to breastfeed her baby because she cannot produce lactose. (C) The production of lactose by the mammary gland does not require the ingestion of milk or milk products. (D) She can produce lactose directly by degrading α-lactalbumin. (E) A diet rich in saturated fats will enable her to produce lactose.

*The answer is B.* Lactase converts lactose (milk sugar) to glucose and galactose. In the absence of lactase activity (the chemotherapy is destroying the rapidly growing cells, such as the intestinal epithelial cells, where lactase is found), the lactose enters the colon, where the bacterial flora metabolize it to produce gases and acids. The gases produce flatulence, and the acids lead to an osmotic imbalance that drives water to leave the colonic epithelium and enter the lumen of the colon, leading to the diarrhea. Sucrase converts sucrose (table sugar) to glucose and fructose. Amylase helps digest plant starches. Isomaltase releases glucose residues from branched oligosaccharides. Trehalase splits trehalose, which is glucose α-1,1-glucose (a disaccharide).

A woman undergoing chemotherapy for breast cancer has developed bloating, diarrhea, and excess gas whenever she drinks milk. She never had this problem before. The symptoms the woman is experiencing is due to a reduced synthesis of which one of the following enzymes? (A) Sucrase (B) Lactase (C) Amylase (D) Isomaltase (E) Trehalase

*The answer is C.* Chemotherapy targets rapidly growing cells. The outer cells of the intestinal lining (brush border) are rapidly growing cells and are commonly affected by chemotherapy. Lactase is found in the brush border. Cancer metastases to the small bowel would not disrupt the entire small intestine. Cancer infiltration to the pancreas or salivary gland would also not affect lactase activity.

A woman undergoing chemotherapy for breast cancer has developed bloating, diarrhea, and excess gas whenever she drinks milk. She never had this problem before. Which one of the following best describes the mechanism causing the symptoms in the above-mentioned patient? (A) Chemotherapy damage to the salivary gland (B) Chemotherapy damage of the pancreas (C) Chemotherapy damage of the brush border of the intestine (D) Cancer infiltration into the small intestine (E) Cancer infiltration into the pancreas (F) Cancer infiltration into the salivary gland

*The answer is E.* The cake contains starch, lactose (milk sugar), and sucrose (table sugar). Digestion of starch produces glucose. Lactase cleaves lactose to galactose and glucose, and sucrase cleaves sucrose to fructose and glucose. Thus, the intestinal epithelial cells will absorb from the intestinal lumen, and then secrete into the blood, glucose, galactose, and fructose. The intestinal epithelial cells will not use these sugars as an energy source.

After digestion of a piece of cake that contains flour, milk, and sucrose as its primary ingredients, the major carbohydrate products entering the blood are which one of the following? Choose the one best answer. (A) Glucose (B) Fructose and galactose (C) Galactose and glucose (D) Fructose and glucose (E) Glucose, fructose, and galactose

*The answer is D.* This patient has exhibited symptoms of beri-beri disease, which is a result of a nutritional deficiency in vitamin B1 (thiamine). It is important to keep in mind that alcohol abuse can make it hard for your body to absorb and store thiamine. The active form of the vitamin, thiamine pyrophosphate, is a required cofactor for Alpha-ketoglutarate dehydrogenase.

After excessive drinking over an extended period of time while eating poorly, a middle-aged man is admitted to the hospital with "high output" heart failure. Which of the following enzymes is most likely inhibited? (A) Aconitase (B) Citrate synthase (C) Isocitrate dehydrogenase (D) Alpha-ketoglutarate dehydrogenase (E) Succinate thiokinase

*The answer is A.* The patient is lacking muscle glycogen phosphorylase and cannot utilize muscle glycogen for energy. This is another glycogen storage disease, type V, McArdle disease. The lack of muscle glycogen phosphorylase is why lactate production during exercise is very low. As shown in the figure below, there are many glycogen particles present in the muscle cells just below the sarcolemma, as the glycogen is not able to be degraded. Muscle damage also results from vigorous exercise, releasing myoglobin into the circulation, which is what leads to the reddish-brown urine after exercise. Alterations in liver enzymes (phosphorylase or PFK-1) would not affect exercise tolerance in the muscle. Muscle does not contain glucose-6-phosphatase, and this problem is not due to a lack of muscle GLUT4 transporters, as the muscle cannot utilize stored, internal glucose supplies.

An 18-year-old man visits the doctor due to exercise intolerance. His muscles become stiff or weak during exercise, and he sometimes cramps up. At times, his urine appears reddish-brown after exercise. An ischemic forearm exercise test indicates very low lactate production. A potential enzyme defect in this man is which of the following? (A) Muscle glycogen phosphorylase (B) Liver glycogen phosphorylase (C) Liver PFK-1 (D) Muscle glucose-6-phosphatase (E) Muscle GLUT4 transporters

*The answer is B.* When the individual begins to run away from the alligator, muscle contraction leads to calcium release from the sarcoplasmic reticulum to the sarcoplasm. This increase in sarcoplasmic calcium binds to the calmodulin subunit of phosphorylase kinase and activates the enzyme in an allosteric manner, in the absence of any covalent modification. The activated phosphorylase kinase will phosphorylate and activate glycogen phosphorylase, which will initiate glycogen degradation. When epinephrine reaches the muscle, phosphorylase kinase will be fully activated via phosphorylation by PKA. The activation of glycogen degradation under these conditions is not due to a decrease in blood glucose levels, insulin binding (insulin would not be released under these conditions), a decline in ATP levels (the AMP-activated protein kinase does not activate glycogen degradation), or lactate production, the end product of anaerobic metabolism. The figure below shows the stimulation of glycogen degradation, working through calcium activation of the calmodulin subunit of phosphorylase kinase.

An individual is taking a serene walk in the park when he spots an escaped alligator from the zoo. The individual runs away as fast as he can. Glycogen degradation is occurring to supply glycolysis with a substrate even before epinephrine has reached the muscle. This is due to which of the following? (A) Sudden decrease in blood glucose levels (B) Increase in sarcoplasmic calcium levels (C) Insulin binding to muscle cell receptors (D) Decline in ATP levels (E) Lactate production

*The answer is B.* Fructose gives a positive result in a reducing-sugar test and a negative result in a glucose oxidase test. It is a monosaccharide, and, so, is not cleaved by acid. Glucose gives a positive test result with the enzyme glucose oxidase. Sorbitol has no aldehyde or ketone group, and, thus, cannot be oxidized in the reducing-sugar test. Maltose and lactose are disaccharides that undergo acid hydrolysis, which doubles the amount of reducing sugar. This infant probably has benign fructosuria or the more dangerous condition, HFI. A galactose oxidase test would rule out the possibility that the sugar was galactose.

An infant, who was nourished by a synthetic formula, had a sugar in the blood and urine. This compound gave a positive reducing- sugar test but was negative when measured with glucose oxidase. Treatment of blood and urine with acid (which cleaves glycosidic bonds) did not increase the amount of reducing sugar measured. Which of the following compounds is most likely to be present in this infant's blood and urine? (A) Glucose (B) Fructose (C) Sorbitol (D) Maltose (E) Lactose

*The answer is D.* As AMP levels increase in the muscle due to the need for ATP for muscle contraction, and the activity of the adenylate kinase reaction, the AMP-activated protein kinase is turned on. One of the effects of the AMP-activated protein kinase is to increase the number of GLUT4 transporters in the muscle membrane, in a process similar to the action of insulin. This enables muscle to take up glucose efficiently from the circulation when internal energy levels are low. The ability of the muscle to take up glucose under these conditions is not due to an increase in epinephrine levels, an increase in sarcoplasmic calcium levels, or insulin binding to muscle cells. Under conditions as described in the question, insulin will not be present in the circulation to bind to the muscle cells. As the muscle does not contain glucagon receptors, there is no effect on muscle when glucagon is present in the circulation.

As an individual runs from an alligator, the muscle begins to import glucose from the circulation. This occurs due to which of the following? (A) Insulin binding to muscle cells (B) Epinephrine binding to muscle cells (C) Glucagon binding to muscle cells (D) Increase in intracellular AMP levels (E) Increase in intracellular calcium levels

*The answer is A.* Alanine and glutamine play an important role in transporting nitrogen throughout the body. Glutamine is produced by most body tissues and is catabolized primarily by the gut and kidney for maintenance of cellular metabolism and acid-base regulation, respectively. A significant portion of the glutamine used by these tissues is converted to alanine and released into the circulation. Alanine is also released by skeletal muscle tissue during protein catabolism as part of the glucose-alanine cycle that helps remove nitrogen disposal and as a source of carbon skeletons for gluconeogenesis. In the liver, alanine is transaminated by alanine aminotransferase to pyruvate with the amino group bring transferred to α-ketoglutarate to form glutamate. Almost all aminotransferase enzymes use α-ketoglutarate as the amino group acceptor. Thus, amino groups are funneled into glutamate during protein catabolism. Glutamate is further metabolized by the enzyme glutamate dehydrogenase, which liberates free ammonia and regenerates α-ketoglutarate. Ammonia then enters the urea cycle to form urea, the primary disposal form of nitrogen in humans. Urea subsequently enters the blodo and is excreted in the urine.

As part of an experiment, healthy volunteers undergo a 12-hour fast and then drink a solution containing radiolabeled alanine. Consecutive blood samples are drawn every 15 minutes for the next 3 hours. Initial blood samples detect the radiolabeled alanine, but analysis of later samples shows that the radiotracer is present in blood primarily in the form of glucose. Before alanine can be converted to glucose, its amino group is transferred to which of the following? (A) α-ketoglutarate (B) L-citrulline (C) Malate (D) Citrate (E) Oxaloacetate

*The answer is D.* At the beginning of the race, a runner running at a reasonable pace consumes energy at a ratio of approximately 75% carbohydrate: 25% fatty acids. However, by the end of the race, glycogen stores are for the most part depleted, and the generation of ATP must come from beta-oxidation of fatty acids, which produces reducing equivalents for oxidative phosphorylation. This required an increase in the amount of oxygen consumed.

During the course of a marathon race a runner expends a large amount of energy and must use stored sources of fuel as well as oxygen. Compared with the beginning of the race (first mile), which of the following best describes the utilization of glycogen and fatty acids as fuels and amount of oxygen consumed after running 26 miles?

*The answer is E.* The answer is E. High-energy phosphate bonds are added to the substrates of glycolysis at three steps in the pathway. Hexokinase—or, in the case of liver, glucokinase—adds phosphate from ATP to glucose to form glucose-6-phosphate. Strictly speaking, this is not always considered a step of the glycolytic pathway. Phosphofructokinase uses ATP to convert fructose-6-phosphate to fructose-1,6-phosphate. Using NAD+ in an oxidation-reduction reaction, inorganic phosphate is added to glyceraldehyde-3-phosphate by the enzyme glyceraldehyde-3- phosphate dehydrogenase to form 1,3-bisphosphoglycerate. The enzymes phosphoglycerate kinase and pyruvate kinase transfer substrate high energy phosphate groups to ADP to form ATP.

During the first week of a diet of 1500 calories per day, the oxidation of glucose via glycolysis in the liver of a normal 59-kg (130-lb) woman is inhibited by the lowering of which one of the following? (A) Citrate (B) ATP (C) Fatty acyl CoA (D) Ketone bodies (E) Fructose-2,6-bisphosphate

*The answer is D.* In this case the problem is insufficient oxygen reaches an area of cardiac muscle. In the absence of sufficient oxygen, the use of the electron transfer chain to generate ATP from ADP is severely compromised, and therefore all products feeding into that chain accumulate reducing the formation of tricarboxylic acid products. Fatty acid oxidation, formation of acetyl-CoA from pyruvate oxidation of ketone bodies, and oxidative phosphorylation are all decreased. In glycolysis under reduced oxygen conditions, one pyruvate is converted to lactate to reoxidize NADH to NAD+ to meet the requirement for glycolysis to continue. Therefore, there is increased formation of lactate.

Following an episode of retrosternal chest pressure with radiation to the neck and associated nausea and diaphoresis while at rest, a middle-aged man was diagnosed with unstable angina and possibly myocardial infarction. Subsequent laboratory findings of increased serum levels of troponin 1 and cardiac enzymes were consistent with the diagnosis. If the patient described above did indeed have angina (chest pain), which of the following change(s) in metabolism in the affected area would occur? (A) Increased oxidative phosphorylation (B) Increased rate of fatty acid oxidation (C) Increased conversion of pyruvate to acetyl-CoA (D) Increased formation of lactate (E) Increased use of ketone bodies

*The answer is E.* Glucagon in the liver and epinephrine in both the liver and muscle cause cAMP levels to rise, activating protein kinase A. Protein kinase A phosphorylates and activates phosphorylase kinase, which in turn phosphorylates and activates phosphorylase. These phosphorylation reactions require ATP. Branching enzyme is not a substrate for phosphorylase kinase. Phosphodiesterase inhibitors, such as caffeine, keep cAMP elevated, which allows protein kinase A to be active, which keeps phosphorylase kinase active, and in its phosphorylated form.

Phosphorylase kinase can be best described by which one of the following?

*The answer is A.* The net ATP yield from glycolysis is 2 ATP per glucose. According to the question, arsenic bypasses glyceraldehyde-3-phosphate dehydrogenase and 3-phosphoglycerate kinase, directly forming 3-phosphoglycerate. 3-phosphoglycerate kinase is one of the two substrate-level phosphorylation steps and normally produces 2 ATP (one for each of the two molecules of glyceraldehyde 3-phosphate formed from glucose). If these two ATP molecules are lost, the net yield of glycolysis is now 0 ATP.

The diagram below shows the effects of arsenic on the metabolism of glyceraldehyde 3-phosphate. As a result, in the presence of arsenic, how many net molecules of ATP would be created directly from the conversion of two glucose molecules to four pyruvate molecules? (A) 0 (B) 1 (C) 2 (D) 4

*The answer is H.* Glucose 1-phosphate is isomerized to glucose 6-phosphate, which then enters glycolysis. This skips the hexokinase step, which uses 1 ATP. Thus, starting from glucose 1-phosphate, one would get the normal 2 ATP and 2 NADH, but with one less ATP used, for a total yield of 3 ATP and 2 NADH.

When glycogen is degraded, glucose 1-phosphate is formed. Glucose 1-phosphate can then be isomerized to glucose 6-phosphate. Starting with glucose 1-phosphate and ending with two molecules of pyruvate, what is the net yield of glycolysis in terms of ATP and NADH formed? (A) 1 ATP, 1 NADH (B) 1 ATP, 2 NADH (C) 1 ATP, 3 NADH (D) 2 ATP, 1 NADH (E) 2 ATP, 2 NADH (F) 2 ATP, 3 NADH (G) 3 ATP, 1 NADH (H) 3 ATP, 2 NADH (I) 3 ATP, 3 NADH

*The answer is B.* The sequential cleavage of the α-1,4-glycosidic bonds of glycogen to release successive glucose-1-phosphate residues is known as glycogenolysis. The enzyme catalyzing this reaction is glycogen phosphorylase a, an active, phosphorylated tetramer formed by covalent modification of phosphorylase b, an inactive dimer. In glycogenesis or glycogen synthesis, activated glycogen synthase adds the glucose of uridine diphosphate (UDP)-glucose units to a growing glycogen polymer by forming α-1,4 linkages. In contrast to phosphorylase, glycogen synthase is inactivated by covalent phosphate binding. The same enzyme that inactivates glycogen synthase by catalyzing its phosphorylation activates another enzyme, phosphorylase kinase, which activates glycogen phosphorylase by phosphorylation.

Which of the following statements about glycogen metabolism is true? (A) Cyclic AMP-activated protein kinase stimulates glycogen synthase (B) Phosphorylase kinase is activated by phosphorylation (C) Phosphorylase b is inactivated by phosphorylation (D) Cyclic AMP levels are lowered by epinephrine and glucagon stimulation of adenylate cyclase (E) Glycogen synthesis is stimulated by glucagon

*The answer is B.* The citric acid cycle produces NADH and FADH2 from the cycle metabolites. These are subsequently reoxidised in the electron transport chain and provide energy in the form of ATP synthesized from ADP and Pi (inorganic phosphate). No oxygen is involved in the cycle. The oxygen atoms needed to form CO2 come from water. If there is no oxygen available, the cycle will not be able to proceed because NADH and FADH2 will not be reoxidised by the electron transport system. There is only a limited amount of NAD+ and FAD in cells. One molecule of GTP is produced by substrate-level phosphorylation in the cycle.

Which of the following statements about the TCA cycle is correct? (A) Oxygen is used to oxidise the acetyl group carbons of acetyl-CoA in the TCA cycle. (B) Three molecules of NADH and one molecule of FADH2 are produced in one turn of the TCA cycle. (C) Oxygen is not used in the TCA cycle, so the cycle can occur in anaerobic conditions. (D) The TCA cycle produces the water that is formed during the complete oxidation of glucose.

*The answer is D.* Glycogen is a highly branched polymer of α-D-glucose residues joined by α-1,4-glycosidic linkage. Under the influence of glycogen synthase, the C4 alcohol of a new glucose is added to the C1 aldehyde group of the chain terminus. The branched chains occur about every 10 residues and are joined in α-1,6-glycosidic linkages. Large amounts of glycogen are stored as 100- to 400-Å granules in the cytoplasm of liver and muscle cells. The enzymes responsible for making or breaking the α-1,4-glycosidic bonds are contained within the granules. Thus glycogen is a readily mobilized form of glucose.Choice B is incorrect, according to the above image there are less branch residues than residues in straight chains.

Which of the following statements about the structure of glycogen is true? (A) Glycogen is a copolymer of glucose and galactose (B) There are more branch residues than residues in straight chains (C) Branch points contain α-1,4 glycosidic linkages (D) New glucose molecules are added to the C1 aldehyde group of chain termini, forming a hemiacetal (E) The monosaccharide residues alternate between D- and L-glucose

*The answer is D.* Epinephrine stimulates both muscle and liver adenylate cyclase to produce cyclic AMP. In the liver, the increased cyclic AMP levels activate a phosphatase that dephosphorylates fructose-2,6-bisphosphate (F-2,6-BP) while deactivating a kinase that produces F-2,6-BP. Thus, F-2,6-BP levels are decreased and phosphofructokinase activity is decreased. In liver and muscle, F-2,6-BP is the major allosteric activator of phosphofructokinase. In skeletal muscle, however, the kinase responsible for the synthesis of F-2,6-BP is activated, not inhibited, by cyclic AMP. Thus, muscle sees an increase in glycolysis following epinephrine stimulation, while the liver experiences a decrease in glycolytic activity. In both tissues, glycogen phosphorylase is activated and glycogenolysis occurs. Under these conditions, glucose is utilized in muscle for ATP production relative to contractile activity, while the liver produces glucose for export to the blood.

Which one of the following activities is simultaneously stimulated by epinephrine in muscle and inhibited by epinephrine in the liver? (A) Fatty acid oxidation (B) Glycogenolysis (C) Cyclic AMP synthesis (D) Glycolysis (E) Activation of phosphorylase

*The answer is C.* All the enzymes named are glycolytic enzymes that carry out phosphorylation of glucose-derived substrates or of ADP to form ATP. However, only the reaction catalyzed by glyceraldehyde-3-phosphate dehydrogenase is a phosphorylation reaction coupled to oxidation that uses inorganic phosphate. In this reaction, glyceraldehyde-3-phosphate is converted to 1,3-bisphosphoglycerate by the addition of inorganic phosphate and the oxidation of glyceraldehyde 3-phosphate with the concomitant reduction of NAD+ to NADH + H+. This reaction is an example of a high-energy phosphate compound being produced by an oxidation-reduction reaction. The oxidation of the aldehyde group at C1 of glyceraldehyde-3-phosphate provides the energy for the reaction. The 1,3-bisphosphoglycerate can then be utilized to phosphorylate ADP to ATP through the action of phosphoglycerate kinase, which is the next step in the glycolytic pathway.

Which one of the following enzymes catalyzes phosphorylation with the use of inorganic phosphate? (A) Hexokinase (B) Phosphofructokinase (C) Glyceraldehyde-3-phosphate dehydrogenase (D) Phosphoglycerate kinase (E) Pyruvate kinase

*The answer is B.* Hexokinase is inhibited by its product, glucose-6 phosphate. PFK1 is activated by AMP and F-2,6-P. F-2,6-P does not inhibit glucokinase, nor is glucokinase present in the muscle. Aldolase is not inhibited by its substrate, fructose 1,6-bisphosphate. Pyruvate kinase is inactivated by glucagon-mediated phosphorylation in the liver, but not in the muscle. The muscle isozyme of pyruvate kinase is not a substrate for protein kinase A. In addition, muscle cells do not respond to glucagon as they do not express glucagon receptors.

Which one of the following is a regulatory mechanism employed by muscle for glycolysis? (A) Inhibition of PFK1 by AMP (B) Inhibition of hexokinase by its product (C) Activation of pyruvate kinase when glucagon levels are elevated (D) Inhibition of aldolase by fructose 1,6-bisphosphate (E) Inhibition of glucokinase by F-2,6-P

*The answer is A.* In the mitochondria, CO2 is added to pyruvate to form oxaloacetate. The enzyme is pyruvate carboxylase, which requires biotin and ATP. Oxaloacetate leaves the mitochondrion as malate or aspartate and is regenerated in the cytosol. Oxaloacetate is converted to phosphoenolpyruvate by a reaction that utilizes GTP and releases the same CO2 that was added in the mitochondrion. The remainder of the reactions occur in the cytosol.

Which one of the following occurs during the conversion of pyruvate to glucose during gluconeogenesis? (A) Biotin is required as a cofactor. (B) The carbon of CO2, added in one reaction, appears in the final product. (C) Energy is utilized only in the form of GTP. (D) All of the reactions occur in the cytosol. (E) All of the reactions occur in the mitochondrion.

*The answer is B.* This patient is suffering from deficiency of alpha-1,6-glucosidase, which is also known as glycogen debranching enzyme. The key to answering this question correctly is to keep in mind that Cori disease results in normal lactic acid levels and the question mentions that "the patient has normal serum lactate." There are several types of glycogen storage disorders. These disorders all share the common feature of having abnormal glycogen metabolism. The disorders are numbered in order of the pathway from the end (glucose release) to the beginning (breakdown of glycogen polymer). The particular point of breakdown in the pathway leads to the particular presentation of each disease. The vignette describes a case of type III disease (Cori disease), which is due to an inability to debranch glycogen. (Mnemonic: Cori = Can't Catabolize branches). Patients present with hepatomegaly and hypoglycemia. Biochemical analysis will shown abnormal short outer glycogen chains.

You are presented with an infant boy who has repeatedly experienced hypoglycemia and seizures. On your exam, you note decreased muscle tone, growth retardation, and hepatomegaly. The patient has normal serum lactate. You review results of a liver biopsy, shown in Figure A. Further biochemical testing reveals glycogen with abnormally short outer chains. Which of the following enzymes is most likely deficient in this patient? (A) Muscle glycogen phosphorylase (B) Alpha-1,6-glucosidase (C) Glucose-6-phosphatase (D) Acid maltase (E) Branching enzyme

*The answer is C.* Barbiturates are metabolized via cytochrome P450 enzymes, which are induced by their substrates. The induction of synthesis requires that heme be synthesized, and the first step in heme synthesis requires succinyl-CoA and glycine and occurs within the mitochondrial matrix. Thus, succinyl-CoA levels can drop in the matrix during heme synthesis, and anaplerotic reactions are required to keep the cycle going.

The concentration of TCA cycle intermediates can be reduced under certain conditions. Consider a patient who initiates taking barbiturates. During the initial phase of his taking this drug, which TCA cycle intermediate is reduced in concentration? (A) Citrate (B) α-ketoglutarate (C) Succinyl-CoA (D) Fumarate (E) Oxaloacetate

*The answer is E.* The patient appears to have a thiamine-responsive PDH deficiency. The enzyme fails to bind thiamine pyrophosphate at low concentration, but shows significant activity at a high concentration of the coenzyme. This mutation, which affects the Km of the enzyme for the coenzyme, is present in some, but not all, cases of PDH deficiency. All inborn errors of PDH are associated with elevated levels of lactate, pyruvate, and alanine (the transamination product of pyruvate). Patients routinely show neuroanatomic defects, developmental delay, and often early death. Elevated lactate and pyruvate are also observed in pyruvate carboxylase deficiency, another rare defect in pyruvate metabolism. Because PDH is an integral part of carbohydrate metabolism, a diet low in carbohydrates would be expected to blunt the effects of the enzyme deficiency. By contrast, fatty acid degradation occurs via conversion to acetyl CoA by β oxidation, a process that does not involve pyruvate as an intermediate. Thus, fatty acid metabolism is not disturbed in this enzyme deficiency.

A 1-month-old male showed abnormalities of the nervous system and lactic acidosis. Enzyme assay for pyruvate dehydrogenase (PDH) activity on extracts of cultured skin fibroblasts showed 5% of normal activity, with a low concentration (1 x 10-4 mM) of thiamine pyrophosphate (TPP), but 80% of normal activity when the assay contained a high (0.4 mM) concentration of TPP. Which one of the following statements concerning this patient is most correct? A. Elevated levels of lactate and pyruvate in the blood reliably predict the presence of PDH deficiency. B. The patient is expected to show disturbances in fatty acid degradation. C. A diet consisting of high carbohydrate intake would be expected to be beneficial in this patient. D. Alanine concentration in the blood is expected to be less than normal. E. Administration of thiamine is expected to reduce his serum lactate concentration and improve his clinical symptoms.

*The answer is C.* This child presents with hepatomegaly. cardiomegaly. and heart failure secondary to cardiomyopathy, all characteristic features of Pompe disease, one of the glycogen storage diseases. In this condition, glycogen accumulates in organs due to a deficiency of lysosomal α-1,4 glucosidase. This disease differs from many other glycogen storage diseases (notably von Gierke disease) by the relative prominence of skeletal and cardiac muscle symptoms (weakness) and the relative lack of hypoglycemic and acidotic episodes

A 1-year-old boy is brought to the physician by his parents because of severe hypotonia, The patient's mother states that he has had difficulty feeding because of progressive dyspnea and fatigue. Physical examination shows a liver edge about 5 cm below the right costal margin. and rates are heard on auscultation of both lower lung fields, A chest x-ray shows cardiomegaly. Deficiency of which of the following enzymes is most likely responsible for this patient's symptoms? A. Glucose 6-phosphatase B. Glycogen branching enzyme C. Lysosomal α-1,4-glucosidase D. Lysosomal spingomyelinase E. Myophosphorylase

*The answer is E.* A right-shift in the O2 binding curve is indicative of abnormally elevated 2,3-BPG secondary to a defect in red cell anaerobic glycolysis. Only pyruvate kinase participates in this pathway.

A 10-month-old child is being evaluated for the underlying cause of a hemolytic anemia. In the diagram shown below, the oxygen dissociation curve for hemoglobin in his erythrocytes is compared with the curve obtained with normal red cells. A deficiency of which enzyme is most likely to account for the hemolytic anemia in this patient? (A) Glucokinase (B) Glucose 6-P dehydrogenase (C) Pyruvate carboxylase (D) Glutathione reductase (E) Pyruvate kinase

*The answer is B.* The male most likely has McArdle's disease which is due to a myophosphorylase (muscle glycogen phosphorylase) deficiency. The reaction that is not occuring in this individual is the breakdown of glycogen to glucose-1-phosphate by myophosphorylase. McArdle's disease is an autosomal recessive, type V glycogen storage disease resulting from a deficit of muscle phosphorylase. As a result, the muscles are unable to breakdown glycogen into single glucose monomers. This leads to the clinical presentation of muscle weakness/cramps upon exertion and myoglobinuria ("tea-colored urine") due to the rhabdomyolysis. Another form of phosphorylase deficiency is Hers disease (Type VI) which is due to a lack of hepatic phosphorylase; it presents as hepatomegaly and fasting hypoglycemia.

A 12-year-old male presents to the pediatrician after two days of tea-colored urine which appeared to coincide with the first day of junior high football. He explains that he refused to go back to practice because he was humiliated by the other players due to his quick and excessive fatigue after a set of drills accompanied by pain in his muscles. A blood test revealed elevated creatine kinase and myoglobin levels. A muscle biopsy was performed revealing large glycogen deposits and an enzyme histochemistry showed a lack of myophosphorylase activity. Which of the following reactions is not occurring in this individual? (A) Converting glucose-6-phosphate to glucose (B) Breaking down glycogen to glucose-1-phosphate (C) Cleaving alpha-1,6 glycosidic bonds from glycogen (D) Creating alpha-1,6 glycosidic bonds in glycogen (E) Converting galactose to galactose-1-phosphate

*The answer is F.* The symptoms described suggest a diagnosis of McArdle disease, a type V glycogen storage disorder caused by a lack of myophosphorylase (muscle glycogen phosphorylase). The abnormal accumulation of glycogen in muscle tissue leads to painful cramps and myoglobinuria during strenuous exercise, with no increase in lactic acid in the blood as would normally be seen.

A 14-year-old girl is brought to the physician by her parents because of chronic fatigue. which is worse after exercise. Physical examination shows no respiratory distress, and no neurologic or cardiac abnormalities. A cardiac stress test, however, has to be stopped prematurely due to the development of painful cramps in her legs. The girl says that she has had severe cramping with exercise in the past as well. Laboratory studies performed immediately following the stress test show no increase in serum lactate. A deficiency in which of the following enzymes is most likely responsible for this patient's symptoms? A. Glucose-6-phosphatase B. Glycogen branching enzyme C. Glycogen debranching enzyme D. Liver phosphorylase E. Lysosomal acid-α-glucosidase F. Mytophosphorylase

*The answer is D.* This patient suffers from McArdle disease, a glycogen storage disorder in which myophosphorylase (an isozyme of glycogen phosphorylase) is deficient in muscle. The enzyme is responsible for liberating individual units of glucose-1-phosphate from branches of a glycogen molecule. Onset of the disease typically occurs in adolescence or early adulthood and is characterized by muscle cramping, rapid fatigue, and poor endurance during exertion. In the vignette, the patient undergoes a forearm ischemic exercise test. This test forces the muscle to use glucose as fuel. In the absence of myophosphorylase, insufficient glucose is liberated from muscle glycogen, and there is no elevation of lactate from the induced anaerobic glycolysis. Notably, severe myoglobinuria is also observed in some patients.

A 14-year-old high school freshman presents to her family doctor for a sports physical. She has not played organized sports in the past but is in good physical shape. She mentions that she experienced severe leg cramps after trying out for the soccer team last week. The night after the tryouts, she noticed that her urine had a reddish tinge. She has no other medical complaints. Her physician orders an ischemic forearm exercise test, which reveals no increase in venous lactate. An isozyme of which of the following enzymes is most likely def icient in this patient? A. α-1,6-Glucosidase B. Cystathionine synthase C. Glucose-6-phosphatase D. Glycogen phosphorylase E. Lysosomal a-1,4-glucosidase

*The answer is D.* The spleen is an organ of the reticuloendothelial system that contains approximately 25% of body's lymphoid tissue. One of the main functions of the spleen in adult humans is maintenance of erythrocyte quality in the red pulp by removal of senescent and defective red blood cells. The spleen accomplishes this function through the unique organization of its parenchyma and vasculature. Antibody production and B cell affinity maturation occur in the white pulp of the spleen, and the spleen also serves to remove antibody-coated bacteria and other opsonized material and cells from the circula.tion. An increase in any of these normal functions may result in splenomegaly. Pyruvate kinase is the enzyme in the glycolytic pathway that converts phosphoenolpyruvate to pyruvate resulting in the generation of a molecule of ATP. Pyruvate kinase is allosterically stimulated by fructose 1,6-bisphosphate. which is produced from fructose-6-phosphate by the enzyme phosphofructokinase. Allosteric stimulation of pyruvate kinase by fructose 1,6-bisphosphate results in stimulation of glycolysis. Red blood cells do not contain mitochondria, so the main metabolite of glycolysis is lactate. Any deficiency of glycolysis in red blood cells leads to hemolysis because of insufficient production of ATP and defective maintenance of red blood cell architecture. Excessive erythrocyte destruction by the spleen causes splenomegaly due to work hypertrophy (choice D). Work hypertrophy results from hypertrophy of the reticuloendothelial cells of the splenic parenchyma as these cells are involved in the removal of damaged RBCs. *Educational Objective:* Pyruvate kinase deficiency causes hemolytic anemia due to failure of glycolysis and resultant failure to generate sufficient ATP to maintain erythrocyte structure. In this case, splenic hypertrophy results from increased work of the splenic parenchyma, which must remove these deformed erythrocytes from the circulation.

A 14-year-old male is being evaluated for splenomegaly. Enzyme assays performed on circulating blood calls demonstrate low pyruvate kinase activity. Which of the following is the most likely cause of this patient's splenomegaly? A. Intracellular substance accumulation B. Passive congestion C. Inflammatory infiltration D. Work hypertrophy E. Neoplastic lesion

*The answer is C.* Hepatosplenomegaly accompanied by progressive neurologic deterioration in a young child should suggest several lysosomal storage diseases within the differential diagnosis. Niemann-pick disease is due to a deficiency of lysosomal sphingomyelinase {sphingomyelinase phosphodiesterase-1), leading to an accumulation of sphingomyelin in histiocytes. These cells are typically referred to as "sea blue" histiocytes Under electron microscopy, lamellar deposits of lipid may be seen as "zebra bodies" in the lipid-laden cells. Niemann-pick disease is more common among Ashkenazi Jews, and generally results in death by age 3. The cherry-red spot (actually retinal pallor that spares the macula) seen in approximately 50% of patients with Niemann-pick type A is also a characteristic of Tay-Sachs disease, but hepatosplenomegaly suggests Niemann-pick disease rather than Tay-Sachs.

A 15-month-old boy with a history of mental retardation, failure to thrive, and progressive neurologic deterioration is brought to the physician for a scheduled examination. Physical examination shows hepatosplenomegaly. Funduscopic examination shows retinal pallor sparing the macula in the left eye. Which of the following is the most likely diagnosis? A.Hunter syndrome B. I-cell disease C. Niemann-pick disease D. Pompe disease E. Tay-Sachs disease

*The answer is D.* Niemann-Pick (Type A) disease is characterized by hepatosplenomegaly, with or without cherry-red spots in the macular region, neurologic involvement (mental retardation, failure to crawl, sit, or walk independently).

A 15-month-old female infant is brought to the emergency room by her parents. The infant's mother did not receive routine pre-natal care, and limited information is available regarding the infant's pediatric care. The mother does reveal that the infant "doesn't seem like her other children" and has always been very "fussy."' Physical examination reveals a distressed infant who does not verbalize. Her abdomen is tender and enlargement of both spleen and liver are present. Opthalmoscopic examination fails to reveal cherry-red spots. After a brief hospital course, the infant dies and autopsy is performed. Neural tissue shows parallel striations of electron-dense material within lysosomes. A defect in which of the following was most likely present in this infant? A. Golgi-associated phosphate transfer to mannose B. Degradation of ganglioside GM2 C. Degradation of glucocerebrosides D. Degradation of sphingomyelin E. Synthesis of gangliosides

*The answer is A.* This patient most likely has McArdle disease (glycogen storage disease type V). This condition is caused by a deficiency of myophosphorylase, an isoenzyme of glycogen phosphorylase present in muscle tissue. Deficiency of this enzyme leads to a decreased breakdown of glycogen during exercise, resulting in poor exercise tolerance, muscle cramps, and rhabdomyolysis. The prognosis is generally good and symptoms can be improved by consuming simple sugars before beginning physical activity. During glycogenolysis glycogen phosphorylase showers glycogen chains by cleaving α-1,4-glycosidic linkages between glucose residues, liberating glucose 1-phosphate in the process. This occurs until 4 residues remain before a branch point (the "limit" dextrin). At this point, the debranching enzyme performs 2 enzymatic functions: 1. Glucosyltransferase cleaves the 3 outer glucose residues of the 4 that are left by glycogen phosphorylase and transfers them to a nearby branch 2. The enzyme α-1,6-glucosidase removes the single remaining branch residue, producing free glucose and a linear glycogen chain that can be further shortened by glycogen phosphoryiase

A 15-year-old boy is being evaluated for poor exercise endurance. He recently started weightlifting with some of his friends and is disappointed that he is "the weakest one by far." He says he tries as hard as possible, but his "arms feel like jelly after just a few repetitions." The patient also says he sometimes experiences severe muscle cramping and urine discoloration after periods of intense straining. Further evaluation reveals that his exercise tolerance can be greatly improved by drinking an oral glucose solution before beginning a strenuous activity. This patient is most likely deficient in an enzyme that catalyzes which of the following conversions?

*The answer is D.* Glucose-6-phosphate dehydrogenase (G6PD) deficiency often manifests in young adulthood or adolescents after a serious infection. Genetic mutation variants have been described, including the common G6PDA and G6PD Mediterranean, both of which are X-linked. Normal G6PD generates reduced nicotinamide adenine dinucleotide phosphate (NADPH) from oxidized nicotinamide adenine dinucleotide phosphate (NADP+), which is used to reduce glutathione. Reduced glutathione is used to detoxify oxidizing agents. Oxidizing agents are found at times of infection, as with certain drugs such as primaquine, and with ingestion of fava beans (called favism when symptomatic).

A 15-year-old boy presents with prolonged fatigue and mild jaundice following a serious infection. Blood tests reveal hemoglobin of 10.5 g/dL and an elevated reticulocyte count. A peripheral blood smear reveals Heinz bodies. Which of the following best describes the normal action associated with this patient's metabolic defect? (A) To generate glucose-6-phosphate in all cells (B) To generate glucose-6-phosphate in RBCs only (C) To generate mucopolysaccharides (D) To regenerate reduced nicotinamide adenine dinucleotide phosphate in all cells (E) To regenerate reduced nicotinamide adenine dinucleotide phosphate in RBCs only

*The answer is C.* The patient has glucose-6-phosphate dehydrogenase deficiency, and his red blood cells cannot convert oxidized glutathione to reduced glutathione due to a lack of NADPH. Fava beans contain a potent oxidizing agent that will, in some patients (but not all), lead to hemolytic anemia in individuals with glucose-6-phosphate dehydrogenase deficiency; in individuals with a normal G6PDH, the oxidizing agent is handled by glutathione. The red blood cells, under these conditions, do not have a problem in regenerating NADH, NAD+, or ATP.

A 25-year-old African American male, in good health, had read about fava beans in "Silence of the Lambs." For dinner one night, the man had liver with fava beans and a nice Chianti. About 8 h after eating the beans, the man was tired and weak. Blood work showed hemolytic anemia. This patient most likely has a defect in regenerating which of the following? (A) NADH (B) NAD+ (C) Reduced glutathione (D) Oxidized glutathione (E) ATP

*The answer is A.* Fabry disease is an X-linked recessive disease caused by a deficiency of the enzyme a-galactosidase A, which results in the accumulation of ceramide trihexoside. Accumulation of ceramide trihexoside in various tissues leads to symptoms including angiokeratomas, hypohidrosis, keratopathy, and acroparesthesias. Acroparesthesias are simply the sensations of tingling, tickling, or prickling sensation of the patient's extremities, which can be reported by patients with Fabry disease as described in the vignette. Later in life patients develop renal, cardiac, and cerebral disease due to accumulation of ceramide trihexoside in the vascular endothelium. Many sustain significant renal damage that necessitates renal transplantation. This can be mitigated or delayed somewhat with enzyme replacement, but most patients die from renal, cardiac, or cerebrovascular disease complications.

A 16-year-old boy comes to the clinic for a routine visit. On physical examination, there are several small red papules on the abdomen and scrotum, as well as hyperkeratosis. Review of systems is notable for intermittent paresthesias in the dig its bilaterally and heat intolerance. Laboratory evaluation shows: Na+: 138 mEq/L K+: 4.3 mEq/L Cl- : 105 mEq/L HCO3-: 27 mEq/L Blood urea nitrogen: 15 mg/dl Creatinine: 1.4 mg/dl Examination of this patient's vascular endothelium would most likely show a pathologic accumulat ion of which of the following substances? A. Ceramide trihexoside B. Galactocerebroside C. Glucocerebroside D. Lactosyl cerebroside E. Sphingomyelin

*The answer is B.* In hemolytic anemia, there is a disproportionate number of younger RBCs being turned over. They are turned over in less than 100 days.

A 16-year-old girl with a history of is brought to the emergency department by her parents because of severe nausea, and vomiting, and abdominal pain. Her breath smells fruity. Urinalysis shows ketones and glucose. She states that she has been unable to control her blood glucose accurately because she was too busy with her classwork, exams, and after-school activities. Further evaluation shows a glycosylated hemoglobin HbAIc) of 4.8% Which of the following conditions will most likely cause a measurement to have a value that is falsely low? A. Folate deficiency B. Hemolytic anemia C. Hemophilia D. Severe iron deficiency E. Vitamin K deficienct

*The answer is C.* Although a deficiency in a number of enzymes can result in exercise intolerance, the lack of an increase in serum lactate following ischemic exercise points to an inability to a defect in the breakdown of glycogen in the muscle. The muscle depends on glycogenolysis for intense exercise, and fatigue rapidly ensues when glycogen is depleted.

A 17-year-old boy presents complaining of an inability to perform strenuous exercise without bringing on painful muscle cramps and weakness. He indicated that mild to moderate exercise resulted in no problems. When he was administered an ischemic exercise test, his serum lactate concentrations did not significantly increase. A deficiency in which of the following enzymes is most likely the cause of the patient's muscle cramps? (A) Carnitine palmityl transferase II (B) Glucose-6-phosphatase (C) Glycogen phosphorylase (D) Glycogen synthase (E) Very long chain acyl-CoA dehydrogenase

*The answer is B.* The hexose monophosphate shunt in RBCs maintains glutathione in a reduced state. Glucose-6-phosphate dehydrogenase (G6PD) is required to catalyze nicotinamide adenine dinucleotide phosphate (NADP) to its reduced form (NADPH). In patients with G6PD deficiency, during oxidative stress there is a build-up of NADP, and NADPH levels are too low to maintain glutathione in a reduced state. As a result of lowered levels of reduced glutathione, there is a build-up of oxidants, which damages hemoglobin as well as enzymes. The affected RBCs are ultimately removed from circulation by the spleen, causing anemia.

A 17-year-old young man was recently prescribed sulfamethoxazole and trimethoprim (SMX/TMP) for a urinary tract infect ion. The patient calls the physician's office 3 days later complaining of fatigue, darkening urine, and a yellow tinge to his skin and sclera. The physician suspects that the patient has a glucose-6-phosphate dehydrogenase deficiency. Which compound builds up because of this deficiency? A. Hemoglobin B. Oxidized glutathione C. Oxidized nicot inamide adenine dinucleotide D. Reduced glutathione E. Reduced nicotinamide adenine dinucleotide phosphate

*The answer is B.* Galactosemia is an autosomal recessive disease caused by a deficiency of galactose-1-phosphate uridyltransferase, which is necessary for the metabolism of the galactose derived from milk lactose. The condition should be suspected in infants with growth failure, cataracts, liver disease, aminoaciduria, and mental retardation. A reducing sugar (galactose) is usually present in the urine. Most of the pathology is related to the toxic effects of galactose-1 -phosphate. Treatment involves strict dietary lactose restriction, which consists of more than simply withdrawal of milk; products because lactose is also present in many nondairy foods. Strict adherence to the diet can strikingly alter the course of this disease.

A 2-week-old boy is brought to the physician because of vomiting and diarrhea accompanied by jaundice. Two months later, he develops cataracts and ascites. The boy is switched to a milk-free diet, which stabilizes, but does not completely reverse, his condition. By 1 year of age, he develops mental retardation. Which of the following proteins is most likely deficient in this child? A. Beta polypeptide of a copper-transporting ATPase B. Galactose-1-phosphate uridyltransferase C. Myophosphorylase D. Transmembrane conductance regulator in exocrine tissues E. UDP-glucuronyltransferase

*The answer is B.* The child has classic galactosemia, a defect in galactose-1-phosphate uridylyltransferase. Due to the accumulation of galactose-1-phosphate, galactokinase is inhibited, and free galactose accumulates within the blood and tissues. The accumulation of galactose in the lens of the eye provides substrate for aldose reductase, converting galactose to its alcohol form (galactitol). The accumulation of galactitol leads to an osmotic imbalance across the lens, leading to cataract formation. Additionally, the increased galactose-1-phosphate, at very high levels in the liver, blocks phosphoglucomutase activity, resulting in ineffective glucose production from glycogen (phosphorylase degradation of glycogen will produce glucose-1-phosphate, but this cannot be converted to glucose-6-phosphate if phosphoglucomutase activity is inhibited). A defect in galactokinase will lead to nonclassical galactosemia, with cataract formation, but none of the feeding problems associated with classical galactosemia (associated with the accumulation of galactose-1-phosphate) are observed in nonclassical galactosemia. None of the other enzymes listed, if deficient, will give rise to the symptoms produced, particularly cataract formation. A defect in glycogen synthase would lead to reduced glycogen levels and fasting hypoglycemia. A defect in fructokinase leads to fructosuria (fructose in the urine), but no overt symptoms of disease. The figure below indicates the pathway for galactose metabolism and the defects in classical and nonclassical galactosemia.

A 2-week-old newborn was brought to the pediatrician due to frequent vomiting, lethargy, and diarrhea. Family history revealed that the child never seemed to eat well, and had only been breast-fed. Physical examination revealed an enlarged liver and jaundice. The pediatrician was suspicious of an inborn error of metabolism and referred the child to an ophthalmologist for a slit lamp exam, the result of which is shown below. An enzyme that may be defective in this child is which one of the following? (A) Fructose-1,6-bisphosphatase (B) Galactose-1-phosphate uridylyltransferase (C) Galactokinase (D) Glycogen synthase (E) Fructokinase

*The answer is E.* Lactose (galactosyl beta-1 ,4-glucose or milk sugar) is a disaccharide present in milk. lt is synthesized in the mammary gland by formation of a 1,4 glycosidic linkage between glucose and galactose. Lactose in the diet is catabollzed into glucose and galactose by an intestinal brush-border disaccharidase called lactase (a type of beta-galactosidase more specifically known as lactase-phlorizin hydrolase). Lactose intolerance is characterized by gastrointestinal upset upon ingestion of foods containing lactose, such as dairy products, and is caused by a deficiency of lactase (Answer E). Primary lactose intolerance is a very common disorder, particularly in people of African and Asian descent In contrast to most other races, subjects of Northern European descent maintain lactase activity throughout their life. Secondary lactase deficiency occurs in association with a number of small intestinal mucosal diseases such as celiac sprue and viral gastroenteritis. The underlying pathophysiology of this disorder is due to the fact that lactase is concentrated within epithelial cells in the microvilli of the small intestine (the brush border). When these cells are damaged in gastroenteritis, the damaged cells slough off and are replaced by immature cells that have low concentrations of lactase. The other answer choice options are important in the metabolism of galactose to either glucose or lactose. Galactose is first phosphorylated to galactose-1 -phosphate by the enzyme galactokinase (Choice B). Next, galactose-1 -phosphate uridyltransferase (GALT) catalyzes the conversion of UDP-glucose and galactose-1 phosphate to UDP galactose and glucose-1-phosphate (Choice C). UDP-galactose is then epimerized to UDP-glucose by UDP-galactose-4 epimerase, after which it can participate in the appropriate glucose-related metabolic pathways. Alternatively, UDP-galactose can be converted to galactosyl beta-1,4 glucose (lactose) by lactose synthase within the mammary glands as part of the formation of milk (Choice D). Galactosemia is an illness that is distinct from lactose intolerance, and it is characterized by symptoms that start soon after the initiation of breastfeeding. Galactosemia can be caused by a deficiency of GALT (Type 1), galactokinase (Type 2), or UDP-glucose 4-epimerase (Type 3). Excess galactose in patients with galactosemia is converted to galactitol by aldose reductase (Choice A), and high levels of galactitol are responsible for many of the symptoms associated with galactosemia (especially cataract formation). Educational objective: Secondary lactase deficiency can occur after viral gastroenteritis or other diseases that damage the intestinal epithelium. This disease causes abdominal distention, flatulence, and diarrhea after lactose ingestion.

A 2-year-old boy is brought to the local emergency room by his parents with complaints of fever and diarrhea for several days. Based on his clinical presentation and the time of the year, the on-call pediatric resident admits the patient for dehydration secondary to presumptive rotavirus-induced gastroenteritis. A few days after discharge, the patient is seen by his pediatrician for abdominal distention and diarrhea after each feeding. The symptoms improve significantly once dairy products are eliminated from his diet. Which of the following steps in galactose metabolism is most likely impaired in this patient?

*The answer is E.* The disorder demonstrates X-linked recessive inheritance, with affected male individuals inheriting a defective copy of the X chromosome from heterozygous (asymptomatic) mothers. There is no male-to-male transmission. Hunter syndrome is an X-linked recessive disorder that is related to Hurler syndrome. It is characterized by mild intellectual disability, hearing loss, coarse facies, hepatosplenomegaly, but no corneal clouding (which does occur in Hurler syndrome). It is caused by a deficiency in iduronate sulfatase, leading to accumulation of the mucopolysaccharides dermatan and heparan sulfate in affected t issues.

A 2-year-old boy is brought to the physician because his parents are concerned about a progressive loss of hearing. The child exhibits coarse facial features, stiff joints, hepatosplenomegaly, and hernias; his mental functioning is also diminished. Laboratory analysis shows a deficiency of iduronate sulfatase. Which of t he following types of inheritance is involved in this disease? A. Autosomal dominant B. Autosomal recessive C. Mitochondrial inheritance D. X- linked dominant E. X- linked recessive

*The answer is E.* This young child has Niemann-Pick disease, which typically manifests with hepatosplenomegaly and progressive widespread neurodegeneration. Fundoscopic examination reveals a cherry-red spot on the macula. Sphingomyelinase converts sphingomyelin to ceramide. Deficiency of sphingomyelinase in Niemann-Pick disease causes accumulation of sphingomyelin and cholesterol in parenchymal and reticuloendothelial cells. On light microscopy, these lipid-laden macrophages have a foamy appearance, as indicated by the arrows in this image, due to the presence of innumerable small vacuoles of relatively uniform size.

A 2-year-old boy presents to his pediatrician with hepatosplenomegaly, failure to thrive, and progressive central nervous system deterioration. On ophthalmoscopic exam a cherry red macula is noted. A bone marrow biopsy specimen is shown in the image. The pediatrician suspects that the boy will die by 3 years of age. Which of the following is the function of the metabolic enzyme deficient in this patient? A. Converts ceramide trihexoside to lactosyl cerebroside B. Converts galactocerebroside to cerebroside C. Converts ganglioside M2 to ganglioside M3 D. Converts glucocerebroside to ceramide E. Converts sphingomyelin to ceramide

*The answer is C.* A deficiency of glucose 6-phosphatase (Von Gierke disease) prevents the liver from releasing free glucose into the blood, causing severe fasting hypoglycemia, hyperlacticacidemia, and hyperuricemia. A deficiency of glycogen phosphorylase would result in a decrease in glycogen degradation, causing fasting hypoglycemia, but not the other symptoms. A deficiency of glycogen synthase would result in lower amounts of stored glycogen. Amylo-α(1→6) -glucosidase removes single glucosyl residues attached to the glycogen chain through an α(1→6)-glycosidic bond. A deficiency in this enzyme would result in a decreased ability of the cell to completely degrade glycogen branches. Amylo-α(1→4)→α(1→6)-transglucosidase deficiency would decrease the ability of the cell to make branches.

A 2-year-old boy was brought into the emergency room, suffering from severe fasting hypoglycemia. On physical examination, he was found to have hepatomegaly. Laboratory tests indicated that he also had hyperlacticacidemia and hyperuricemia. A liver biopsy indicated that hepatocytes contained greater than normal amounts of glycogen that was of normal structure. Enzyme assay likely confirmed a deficiency in which of the following enzymes? A. Glycogen synthase B. Glycogen phosphorylase C. Glucose 6-phosphatase D.Amylo-α(1→6)-glucosidase E. Amylo-α(1→4)→α(1→6)-transglucosidase

*The answer is A.* In general, you should associate primary hemolytic anemia with defects in glycolysis or the hexose monophosphate shunt (pentose phosphate pathway). Only one enzyme of those listed in the answer choices specifically involves these pathways and causes hemolytic anemia: pyruvate kinase (PK). PK is a glycolytic enzyme; PK deficiency is an autosomal recessive disorder, affecting males and females approximately equally. Consanguinity increases the incidence of any autosomal recessive condition If this enzyme is deficient, red cells have trouble producing enough ATP to maintain the Na+/K+ pump on the plasma membrane, secondarily causing swelling and lysine. Severe forms of PK deficiency are usually symptomatic in newborns and may be life-threatening. Milder cases of PK deficiency are often missed earlier in life and may or may not produce symptoms later in life. Increased concentrations of glycolytic intermediates in red blood cells occur. 2 to 3 times the normal levels of 2.3-bisphosphoglycerate (2,3-BPG or 2.3,-DPG) in red cells are present, leading to a lower affinity of Hb for O2.

A 26-year-old woman comes to the physician because of recent fatigue and pallor. She says that her sister and older brother have been diagnosed with hemolytic anemia but that two younger brothers are asymptomatic. The patient's mother and father are second cousins. Deficiency of which of the following enzymes is most likely to cause this disorder? A. Debranching enzyme B. Glucose 6-phosphatase C. Muscle phosphorylase D. Pyruvate carboxylase E. Pyruvate kinase

*The answer is C.* Metabolism of glucose through the hexose monophosphate (HMP) shunt serves two major functions: 1. production of NADPH as a reducing equivalent. and 2. synthesis of ribose 5-phosphate for nucleotide synthesis. The HMP shunt consists of two different types of reactions: oxidative (irreversible) and non-oxidative (reversible) reactions. All reactions of HMP shunt occur exclusively in the cytoplasm. In the oxidative portion of HMP shunt, glucose 6-phosphate is first converted to 6-phosphogluconolactone producing one molecule of NADPH. This reaction is catalyzed by glucose 6-phosphate dehydrogenase, the rate limiting enzyme of the HMP shunt In the second reaction of the oxidative portion of HMP shunt, 6 phosphogluconolactone is hydrolyzed to ribulose 5-phosphate by 6-phosphogluconate dehydrogenase producing a second molecule of NADPH. The non-oxidative reactions of the HMP are primarily designed to generate ribose 5-phosphate from intermediates of glycolysis. Erythrocytes utilize the reactions of the HMP shunt to generate large amounts of NADPH to maintain glutathione in a reduced state by the action of glutathione reductase. Reduced glutathione is important in protecting erythrocytes from oxidative damage resulting from oxidant drugs and oxidizing environmental toxins. In erythrocytes, the HMP shunt is the only major pathway that generates NADPH. Thus, defects in the oxidative portion of the HMP shunt result in poor protection of these cells against free radicals, hydrogen peroxide and other forms of oxidant stress. Oxidative damage to red cells causes denatured hemoglobin to form insoluble Heinz bodies resulting in erythrocyte destruction in the spleen. Additionally, oxidative stress results in stiffening of the erythrocyte membrane and hemolysis in die microvasculature due to an inability of the erythrocyte to deform and fit through capillary beds. The patient described in the vignette most likely has glucose 6-phosphate dehydrogenase deficiency (G6PD). G6PD is an X-linked disorder that results in episodes of hemolysis during oxidative and infective stress. The patient in this vignette was likely prescribed trimethoprim-sulfamethoxazole for UTI this drug has oxidant properties end can precipitate hemolysis in patients with this disease. *Educational Objective:* Glucose 6-phosphate dehydrogenase deficiency is a common X-linked disorder of the hexose monophosphate pathway that results in episodes of hemolytic anemia due to oxidative stress.

A 20-year-old African American student presents to the ER with malaise and dark urine. He was diagnosed with a mild urinary tract infection several days ago. Anemia is evident on the complete blood count and erythrocyte fragments are seen on a peripheral smear. Which of the following substrate flow pathways is most likely deficient in this patient?

*The answer is D.* The patient described in this question stem has a clinical presentation of bilateral cataracts, but he is otherwise asymptomatic. This presentation is most consistent with galactokinase deficiency, a form of galactosemia that causes a benign disorder characterized by cataracts without hepatocellular manifestations. Classic galactosemia, by comparison, results from galactose-1-phosphate uridyl transferase (GALT) deficiency, this is the most common form of galactosemia. Patients with GALT deficiency present with vomiting, lethargy and failure to thrive soon after feeding is begun. Other clinical findings of this disorder include impaired liver function, hyperchloremic metabolic acidosis, and aminoaciduria. This disorder results in severe symptoms after initiation of breast feeding. A normal newborn obtains a large amount of their daily calories from lactose present in breastmilk. Following degradation of lactose and absorption of galactose and glucose, galactose is phosphorylated to galactose-1 -phosphate by the enzyme gatactokinase. A deficiency of galactokinase results in elevation of galactose levels. Excess circulating galactose is converted to galactitol by aldose reductase and to galactonic acid by galactose oxidase. while galactonic acid can be metabolized by the HMP shunt, galactitol accumulates in cells. Excess galactitol is responsible for the formation of cataracts in patients with galactokinase deficiency. Dietary restriction of lactose results in improvement in symptoms in all forms of galactosemia. *Educational Objective:* Galactitol accumulates in the lens of patients with galactosemia and causes osmotic damage leading to cataract formation. Galactitol is formed from excess circulating galactose in galactosemia by aldose reductase.

A 21 -year-old male with vision impairment is found to have bilateralvlens opacities. He is otherwise asymptomatic and does not follow any specific diet. Laboratory evaluation reveals urinary excretion of large amounts of galactose. Activity of which of the following enzymes contributes most to this patient's eye condition? A. UDP-Hexose 4-epimarase B. β-Galactosidase C. Aldolase B D. Aldose reductase E. Glucose-6-phosphatase

*The answer is B.* The man has glucose-6-phosphate dehydrogenase deficiency and is incapable of regenerating reduced glutathione to protect red blood cell membranes from oxidative damage. In the presence of a strong oxidizing agent (the new drug the patient was taking), the red cell membranes undergo oxidative damage and the red cell bursts, leading to hemolytic anemia. This is all due to a lack of protective glutathione in the membrane. As the red cell lacks a nucleus, the cell cannot induce new gene synthesis. The drug the patient was taking does not induce ion pores in red cell membranes or inhibit the HMP shunt pathway. It also does not cause oxidative damage to bone marrow. The drugs to avoid while prescribing for a patient with a G6PDH deficiency include primaquine, dapsone, nitrofurantoin, and sulfonylurea.

A 23-year-old man of Mediterranean descent was recently prescribed ciprofloxacin to treat a urinary tract infection. After 2 days on the drug, the patient was feeling worse, and weak, and went to the emergency department. He was found to have hemolytic anemia. This most likely resulted due to which of the following? (A) Induction of red blood cell cytochrome P450s, leading to membrane damage (B) Oxidative damage to red blood cell membranes (C) Drug-induced ion pores in the red blood cell membrane (D) Drug-induced inhibition of the HMP shunt pathway (E) Oxidative damage to bone marrow, interfering with red blood cell production

*The answer is D.* There are a variety of glycogen storage diseases corresponding to defects in different enzymes in glycogen metabolism; most of these involve the liver. <cArdle disease, due to a defect in muscle glycogen phosphorylase (myophosphorylase), is restricted to skeletal muscle. The presentation described in the question stem is typical. Many affected individuals also experience myoglobinuria. Definitive diagnosis is based on demonstration of myophosprlorylase deficiency.

A 24-year-old woman comes to the physician because of severe muscle cramps and weakness with moderate exercise. She states that these episodes are occasionally followed by the excretion of reel urine. Which of the following processes is most likely deficient in this patient? A. Hydrolysis of galactocerobroside B. Hydrolysis of glucose 6-phosphate C. Hydrolysis of spingomyelin D. Phosphorolysis of glycogen E. Transport of neutral amino acids

*The answer is C.* Glucose is the major stimulant of insulin secretion. Glucose-induced insulin release from the beta cells requires the following steps: 1. Glucose enters the beta cell through glucose transporter type 2 (GLUT 2). 2. Glucose is metabolized by glucokinase to glucose-6-phosphate 3. Glucose-6-phosphate is further metabolized by glycolysis and the Krebs cycle to produce ATP 4. A high ATP to ADP ratio within the beta cell results in the closure of ATP-sensitive potassium (K-ATP) channels 5. Depolarization of beta cells results in opening of voltage-dependent calcium channels 6. High intracellular calcium causes insulin release Glucoklnase has a lower glucose affinity than other hexokinases. This allows it to function as a glucose sensor in beta cells by varying the rate of glucose entry into the glycolytic pathway based on blood glucose levels. Heterozygous mutations of the glucokinase gene cause a decrease in beta cell metabolism of glucose, less ATP formation, and diminished insulin secretion. This produces a type of maturity-onset diabetes of the young, which is characterized by mild, nor progressive hyperglycemia that often worsens with pregnancy-induced insulin resistance. Homozygous mutations lead to fetal growth retardation and severe hyperglycemia at birth.

A 24-year-old woman is diagnosed with gestational diabetes mellitus during her first pregnancy. Although her glycemic status improves markedly after delivery, her fasting glucose levels remain modestly elevated. The patient's past medical history is otherwise unremarkable, but her mother and younger sister had "high blood sugars" during pregnancy. If this patient's gestational hyperglycemia is genetically predisposed, she is most likely to have decreased activity in which of the following enzymes? A. Aldolase B. Enolase C. Glucokinase D. Lactate dehydrogenase E. Phosphofructokinase F. Pyruvate carboxylase G. Pyruvate kinase

*The answer is C.* Glucose-6-phosphate dehydrogenase (G6PD) deficiency is characterized by acute episodes of hemolytic anemia following administration of certain medications, infection, or ingestion of fava beans. G6PD is the first and rate-limiting enzyme of the pentose phosphate pathway, which produces two reducing equivalents that keep glutathione in its reduced state. Reduced glutathione is necessary to detoxify peroxides and free radicals that can accumulate within RBCs and cause damage to various cellular structures. Heinz bodies are small round inclusions seen within RBCs that comprise hemoglobin and other protein precipitates. G6PD deficiency is most common in people who are black, people of Mediterranean descent, and people from tropical Africa and Asia.

A 28-year-old African-American man is receiving primaquine therapy for treatment of malaria, which he contracted while visiting Asia. He presents to his physician after noting blood in his urine. Physical examination is significant for scleral icterus, and urinalysis shows hemoglobinuria. A peripheral blood smear shows spherocytes, bite cells, and Heinz bodies. Which of the following is the most likely diagnosis? (A) Alkaptonuria (B) Cystinuria (C) Glucose-6-phosphate dehydrogenase deficiency (D) Hereditary fructose intolerance (E) Hereditary spherocytosis (F) Lactase deficiency

*The answer is C.* Glycolysis is the biochemical pathway that involves the oxidation of glucose to pyruvate, which then goes on to enter the Krebs cycle. The rate-limiting step of glycolysis is the conversion of fructose-6-phosphate to fructose-1,6-bisphosphate by the enzyme phosphofructokinase. Citrate is an intermediate produced in the Krebs cycle that has a role in the regulation of glycolysis by inhibiting the action of phosphofructokinase.

A 28-year-old bodybuilder comes to the clinic for a mandated precompetition physical and blood t est. Her temperature is 37.4oc (99.30f ), blood pressure is 126/84 mm Hg, pulse is 80/ min, and respirations are 14/min. When asked if she has been fasting as required, she states that she forgot and had a carbohydrate-rich meal earlier in the day as she cannot afford to "lose a day of training. The patient has no medical complaints, and the physical exam is unremarkable. Which of the following has a role in the regulation of the rate-limiting step of glycolysis? A. Acetyl-coenzyme A B. Alanine C. Citrate D. Glucose-6-phosphate E. Reduced nicotinamide aden ine dinucleotide

*The answer is A.* The boy has developed MODY (maturity onset diabetes of the young), and one variant of MODY is a mutated glucokinase (an inheritable disorder) such that the Km for glucose has increased, and insulin release only occurs when hyperglycemia is present. Both an increase in ATP and NADPH are required for the pancreatic β-cell to release insulin. When pancreatic glucokinase has an increased Km for glucose, ATP levels can only increase at greater than normal levels of glucose. Thus, moderate hyperglycemia is not sufficient to induce insulin release. As insulin release occurs from the pancreas, liver, muscle, or intestinal hexokinase will not affect the process. The pancreas does not express hexokinase, only glucokinase. MODY is a monogenetic autosomal dominant disease of insulin secretion. There are at least six amino acid substitutions known in a number of different proteins. MODY1 is a mutation in the transcription factor HNF4-α: ∼ MODY2 is a mutation in pancreatic glucokinase. MODY3 is a mutation in the transcription factor HNF1-α while MODY4 contains a mutation in insulin promoter factor 1. MODY5 is a mutation in another transcription factor, HNF1-β. MODY6 is a mutation in neurogenic differentiation factor 1. MODY is not insulin resistance. Therefore, all the other aspects of insulin resistance syndrome are not present (obesity, hypertension, and hypertriglyceridemia). Since MODY is autosomal dominant, it can be traced through the family tree. It was thought at one time that the patient had to be young to present with this disorder, but patients up to age 50 have been reported. It is not type 1 diabetes mellitus as no islet cell antibodies are present. Glucokinase is acting as a glucose sensor for the β-cell. A mutated, less sensitive 3-sensor leads to mildly elevated blood glucose levels.

A 28-year-old male develops diabetes, as noted by constant, mildly elevated hyperglycemia. His father had similar symptoms at the same age as did his paternal grandmother. This patient is not obese, does not have hypertension, does not have dyslipidemia, and does not have antibodies directed against islet cells. This alteration in glucose homeostasis may be due to a mutation in which of the following enzymes? (A) Pancreatic glucokinase (B) Pancreatic hexokinase (C) Liver glucokinase (D) Muscle hexokinase (E) Intestinal glucokinase

*The answer is A.* This patient's constellation of back pain, jaundice, and anemia after receiving primaquine are suggestive of a diagnosis of glucose-6-phosphate dehydrogenase (G6PD) deficiency. G6PD catalyzes a step in the hexose monophosphate (HMP) shunt pathway, which generates NADPH, necessary for reactions that protect erythrocytes against oxidant accumulation. In G6PD deficiency, an X-linked disorder, certain drugs, including primaquine,• infections; or other exposures lead to oxidation of sulfhydryl groups on hemoglobin, resulting in insoluble masses that attach to the erythrocyte membrane, called Heinz bodies. Erythrocytes appear to have a bite-like deformity that may be due to unstained non-hemoglobin-containing portions of the cell membrane. The end result of hemoglobin denaturation is rigid erythrocytes that are subject to extra- and intravascular hemolysis

A 29-year-old African American male who recently returned from a trip to Uganda is diagnosed with malaria. After 3 days of therapy with primaquine, he develops low back pain, jaundice, and dark-colored urine. Laboratory studies show: Which of the following additional laboratory findings is msot likely to be reported? A. Positive Heinz body preparattion B. Positive direct Coombs test C. Decreased serum ferritin concentration D. Low mean corpuscular volume (MCV) E. Abnormal hemoglobin electrophoresis

*The answer is E.* This patient has lactase deficiency. This is characterized by abdominal cramps, bloating, and diarrhea.

A previously healthy 16-year-old girl is brought to the physician because of abdominal cramps, bloating, and loose stools for 6 months. These symptoms began after she ingested skim milk in an attempt to lose weight. She is at the 50th percentile for height and 75th percentile for weight. Physical examination shows no abnormalities. Stool studies show a 3+ Clinitest reagent response and pH of 5. After the patient ingests milk, there is an increased hydrogen concentration in expired air. A deficiency of which of the following enzyme activities is the most likely cause of the gastrointestinal symptoms in this patient? (A) Amylase (B) Carboxypeptidase (C) Fructose-1,6-bisphosphate aldolase (D) Galactokinase (E) Lactase (F) Sucrase

*The answer is D.* Pyruvate carboxylase is a mitochondrial enzyme that normally catalyzes the reaction Pyruvate + CO2 + ATP -> Oxaloacetate + ADP + Pi This reaction serves to make the oxaloacetate that is used within the citric acid cycle. A deficiency of this enzyme has very broad metabolic consequences because the citric acid cycle is downregulated, with resultant downregulation of the energy-producing capacity of the cell. In addition to a general loss of energy in the cell, pyruvate levels rise because pyruvate is not converted into oxaloacetate; alanine levels rise because the normal conversion to pyruvate is feedback-inhibited In hepatocytes, pyruvate carboxylase is also an essential enzyme in gluconeogenesis from the major gluconeogenic amino acid alanine. lt is also involved in converting lactate to glucose. Clinically, pyruvate carboxylase deficiency shows variable expression due to allelic heterogeneity. The North American type, a moderate phenotype, is associated with mental retardation and lactic acidosis. Alanine levels and pyruvate levels are typically elevated. A more complex and life threatening type is common in Great Britain and France. These children die before 3 months of age.

A 3-month-old boy is brought to the physician for a follow-up examination because of seizures, hypotonia and psychomotor retardation. He has been previously diagnosed with pyruvate carboxylase deficiency. Laboratory studies show elevated levels of lactate, alanine pyruvate, and ketoacids. This enzyme is normally used to directly synthesize which of the following molecules? A. Acetyl-CoA B. Citrate C. Malate D. Oxaloacetate E. Succinyl-CoA

*The answer is C.* The child is defective in a variety of oxidative decarboxylation reactions (pyruvate dehydrogenase, leading to a buildup of lactate and pyruvate; α-ketoglutarate dehydrogenase, leading to the buildup of α-ketoglutarate; and branched-chain α-ketoacid dehydrogenase, leading to a buildup of many of the other metabolites). Enzymes, which catalyze oxidative decarboxylation reactions, contain three catalytic subunits, E1, E2, and E3. E3 subunit, which contains the dihydrolipoyl dehydrogenase activity, is common among these enzymes. Thus, a mutation in E3 would render all of these enzymes inoperable, leading to a buildup of the α-ketoacid precursors. Defects in citrate synthase or malate dehydrogenase would not lead to the buildup of these α-ketoacids.

A 3-month-old girl developed lactic acidemia. Blood analysis also indicated elevated levels of pyruvate, α-ketoglutarate, and branched chain amino acids. A urinalysis showed elevated levels of lactate, pyruvate, α-hydroxyisovalerate, α-ketoglutarate, and α-hydroxybutyrate. A likely mutation in which of the following proteins would lead to this clinical fi nding? (A) The E1 subunit of pyruvate dehydrogenase (B) The E2 subunit of pyruvate dehydrogenase (C) The E3 subunit of pyruvate dehydrogenase (D) Citrate synthase (E) Malate dehydrogenase

*The answer is C.* The presence of hepatomegaly and hypoglycemic seizures highly suggests a glycogen storage disease. Only two of the enzymes listed are related to glycogen storage diseases glucose 6-phosphatase (deficiency produces von Gierke disease, from which this patient is suffering) and muscle phosphorylase (choice E deficiency produces McArdle disease). McArdle disease is relatively mild; causing only muscle weakness beginning in the second or third decades and no hepatomegaly, so this option can be excluded Von Gierke disease is a glycogen storage disease caused by a deficiency of glucose 6-phosphatase (type la) or the glucose-6 phosphate translocase (type ID). It typically presents between 3-4 months of age with hypoglycemia. hyperlipidemia, and lactic acidosis. Failure to thrive is also common in early life; convulsions may occur during episodes of profound hypoglycemia. The glycogen accumulation in von Gierke disease occurs primarily in the liver and kidneys, accounting for the enlargement of these organs Gout may develop later because of the derangement of glucose metabolism. Due to allelic heterogeneity, the onset of symptoms may be delayed and, in some cases, symptoms may be milder. Von Gierke disease, like most enzyme deficiencies, is inherited as an autosomal recessive disorder.

A 3-month-old girl is brought to the emergency department by her parents because of seizures. The mother did not have any prenatal care. The diagram shows the pedigree of the couple and their two children. Physical examination shows hepatomegaly and growth retardation. Laboratory studies show severe hypoglycemia and hyperlipidemia. A defect involving which of the following enzymes is most likely responsible for this patient's illness? A. Alpha-L-iduronidase B. Beta-glucocerebrosidase C. Glucose 6-phosphatase D. Hexosaminidase E. Muscle phosphorylase

*The answer is C.* The disorder is hereditary fructose intolerance, with a reduced ability to convert fructose-1-phosphate to dihydroxyacetone phosphate and glyceraldehyde. The specific defect is in aldolase B, with its activity reduced by as much as 85%. This problem is only evident when sucrose is introduced into the diet, and fructose enters the liver. The accumulation of fructose-1-phosphate, due to the reduced aldolase activity, leads to a constellation of physiological problems resulting in nausea, vomiting, and hypoglycemia. Elimination of fructose from the diet will reverse the symptoms. Galactokinase is needed for galactose metabolism; since the patient digests milk normally galactokinase activity is not altered. Similarly, glucose metabolism is not adversely affected (milk contains lactose, which is split into glucose and galactose), indicating that hexokinase and glucokinase activities are normal. The defect in aldolase B will hinder glycolysis, but the liver also contains aldolase C activity (this isozyme will not split fructose-1-phosphate), which enables glucose metabolism to be very close to normal. A deficiency in fructokinase will lead to an accumulation of fructose (not fructose-1-phosphate), which is released into the urine (fructosuria), but does not lead to the physiological symptoms exhibited by the patient.

A 3-month-old girl is brought to the pediatrician due to fussiness and lethargy. According to the parents, the baby was just fine until the mother needed to return to work, and the baby was being switched from breast milk to baby foods, formula, and fruit juices. At that time, the child cried while feeding, sometimes vomited, and had been lethargic. The baby's appetite seemed to have worsened. The parents thought that if only formula was used, the baby was better, but they really could not remember. Which possible enzyme defect might lead to this case presentation? (A) Galactokinase (B) Fructokinase (C) Aldolase (D) Hexokinase (E) Glucokinase

*The answer is C.* The child has nonclassical galactosemia, a defect in galactokinase. With this disorder, galactose cannot be accumulated within cells, and so it accumulates in the blood, spilling over to the urine. Because of its high level, the galactose can enter the eye and be reduced to galactitol by aldose reductase, trapping the galactitol within the eye. As galactitol accumulates, an osmotic imbalance is created, leading to cataract formation. However, since galactose-1-phosphate is not accumulating (as occurs in classical galactosemia, a defect in galactose-1-phosphate uridylyl transferase), the other effects seen with classical galactosemia (hypoglycemia and neurological deficit) do not occur. The sugar that is accumulating in the urine is galactose, which contains an aldehyde, which generates a positive response in a reducing test. A defect in fructokinase leads to fructosuria, a benign condition (fructose is not a substrate for aldose reductase, as it is a ketose and not an aldose). A defect in hexokinase would lead to elevated glucose levels, and can lead to sorbitol production in the lens of the eye, but the urine reducing sugar test was negative for glucose. A defect in aldolase would lead to the intracellular accumulation of metabolites, but not a great increase in circulating galactose.

A 3-month-old girl with developing cataracts is shown to contain a reducing sugar in her urine, but the glucose oxidase test was negative. She has had no problems eating, and her growth curve is at the 60th percentile. Fasting blood glucose tests show normal levels of circulating glucose. A likely enzyme deficiency is which of the following? (A) Fructokinase (B) Hexokinase (C) Galactokinase (D) Galactose-1-phosphate uridylyltransferase (E) Aldolase

*The answer is C.* The child has hereditary fructose intolerance, a defect in aldolase B activity in the liver. This leads to an accumulation of fructose-1-phosphate in the liver (and, as fructokinase has a high Vmax, a large amount of fructose-1-phosphate accumulates). At high levels, fructose-1-phosphate, through similarity in structure to glucose-1-phosphate, inhibits glycogen phosphorylase activity, leading to hypoglycemia (glycogen degradation is inhibited when blood glucose levels drop). The fructose is derived from the fruit juices introduced to the child's diet. Fructose does not inhibit debranching enzyme, and fructose-6-phosphate has no effect on glycogen phosphorylase (recall, one of the products of the glycogen phosphorylase reaction is glucose-1-phosphate, not glucose-6-phosphate). Galactose is found in lactose, which, while present in milk, is not found in fruit juice.

A 3-month-old infant, when switched to a formula diet plus fruit juices, begins to vomit and displays severe hypoglycemia after eating. Removal of the fruit juices from the diet seemed to reduce the severity of the symptoms. At the pediatrician's office, an inborn error of metabolism was considered, which could explain the hypoglycemia. Which explanation is most likely? (A) Fructose inhibition of the debranching enzyme (B) Galactose-1-phosphate inhibition of glycogen phosphorylase (C) Fructose-1-phosphate inhibition of glycogen phosphorylase (D) Fructose-6-phosphate inhibition of glycogen phosphorylase (E) Galactose inhibition of aldolase

*The answer is C.* This boy's presentation suggests classic galactosemia. Classic galactosemia has autosomal recessive inheritance and is caused by a deficiency of galactose-1-phosphate uridyltransferase. Without this enzyme, phosphorylated galactose cannot be converted to phosphorylated glucose to be used in glycolysis and gluconeogenesis. As a result, galactose-1-phosphate and galactitol accumulate in various organs in the body, including the central nervous system, liver, and eyes. Galactose and lactose (milk) should be avoided in patients with this condition. Galactokinase deficiency resulting in galactosemia is also a possibility in this case. However, the presenting symptoms would be less severe, causing only infantile cataracts and developmental delays related to sight (lack of development of a social smile and inability to track objects). Hurler syndrome can also present with corneal clouding and developmental delay. However, Hurler syndrome would classically manifest with gargoyle-like facies.

A 3-week-old boy is brought to the physician, because he seems smaller than other kids his age and has occasional vomiting associated with bottled milk feedings. He was born at home and did not undergo newborn screening. On physical examination, the patient is found to have jaundice, bilateral clouding of eye lenses, and hepatomegaly. The patient also shows signs of developmental delay. Which of the following enzymes is likely to be defective in this condition? A. 1-α-Hydroxylase B. Aldolase B C. Galactose-1-phosphate uridyltransferase D. Iduronate sulfatase E. Transmembrane copper-transporting ATPase

*The answer is D.* Elevated galactitol levels is the cause of the clinical symptoms of classic galactosemia. The urine reducing substances assay is used to detect inborn errors of carbohydrate metabolism such as classic galactosemia. Galactose is converted in two steps to glucose-1-phosphate. The first step is catalyzed by the enzyme galactokinase, which phosphorylates galactose to galactose-1-phosphate. The second step is catalyzed by galactose-1-phosphate uridyltransferase (G1PUR), which converts galactose-1-phosphate to glucose-1-phosphate. In the absence of G1PUR, upstream intermediates in galactose metabolism, including galactose-1-phosphate and galactitol, accumulate. A deficiency in enzymes involved in other aspects of galactose metabolism leads to a much milder presentation (ie, only infantile cataracts). Although treatment is not available, prevention of disease progression involves excluding galactose-containing foods (eg, breast milk and lactose-containing formulas) from the infant's diet.

A 3-week-old infant presents with failure to thrive, poor feeding, and lethargy. A physical examination reveals an enlarged liver and jaundice. Laboratory analysis reveals an elevated blood galactitol level and increased urinary reducing substance. Which of t he following changes in intermediates of galactose metabolism is expected in this patient? A. Decreased galactose B. Decreased uridine diphosphoglucose C. Elevated glucose-1-phosphate D. Increased galactose-1-phosphate E. Increased glycogen

*The answer is C.* Arylsulfatase A converts sulfatide to galactocerebroside. This enzyme is deficient in patients with metachromatic leukodystrophy, an autosomal recessive lysosomal storage disease in which patients cannot degrade sulfatides, leading to accumulation of cerebroside sulfate in both neuronal and nonneuronal tissues. There is abnormal myelination with widespread loss of myelination in the central nervous system and peripheral nerves, leading to the clinical signs. Metachromatic granules can be seen on histologic examination.

A 3-year-old girl is brought to her pediatrician because of a progressive loss of motor function and a decline in her cognitive abilities. On physical examination, it is noted that the patient has decreased deep tendon reflexes, truncal ataxia, and a decreased attention span in comparison to the child's last visit 6 months ago. The physician knows that her pathology is due to an abnormal accumulation of cerebroside sulfate in her brain, peripheral nerves, kidney, and liver. A deficiency of which of the following enzymes leads to this condition? A. α-Galactosidase A B. β-Galactocerebrosidase C. Arylsulfatase A D. Hexosaminidase A E. Sphingomyelinase

*The answer is D.* !-cell disease is a syndrome in which a patient is unable to phosphorylate mannose residues on enzymes destined for lysosomes within cis Golgi of cells. Given that the trans-Golgi have a phosphorylated mannose receptor that specifies enzymes as lysosomal, these nonphosphorylated enzymes are misdirected as secretory products from the cell into the plasma, leading to the clinical syndrome described here. This leads to the presence of inclusion bodies in the cytoplasm of cells, which can be identified using electron microscopy for additional examination.

A 3-year-old girl presents to a pediatrician with restricted joint movement. The physician also notes that she has coarse facial features and skeletal abnormalities. She is diagnosed with a rare condition and is enrolled in a research study, where a small tissue sample is taken for additional examination. Electron microscopy demonstrates numerous intracytoplasmic inclusions. Upon receiving this information, the physician explains to the patient's mother that her daughter has a disease resulting from the aberrant intercellular targeting of proteins. Which of the following biochemical steps is lacking in patients with this disease? A. Failure of targeting proteins from endocytosis to the nucleus B. Phosphorylation of glucose in the endoplasmic reticulum C. Phosphorylation of glucose residues at the trans-Golgi D. Phosphorylation of mannose residues at the cis-Golgi E. Phosphorylation of mannose residues in the endoplasmic reticulum F. Synthesis of proteins in the rough endoplasmic reticulum

*The answer is D.* The biologically active form of pantothenic acid is coenzyme A, an essential cofactor in numerous acetylation reactions, including those associated with the tricarboxylic acid (TCA) cycle. Coenzyme A is particularly important in the first step of the TCA cycle, as it binds with oxaloacetate to form citrate and then succinyl-CoA. Coenzyme A is also important in the synthesis of vitamin A, vitamin D, cholesterol, steroids, heme A, fatty acids, amino acids, and proteins. The pantothenic acid is actively transported into the cell and than undergoes ATP-dependent phosphorylations, which transform it into coenzyme A. Deficiency of pantothenic acid is rare, though it has been observed in severely malnourished people who complain of paresthesias and dysesthesias ("burning feet syndrome") and gastro-intestinal distress. *Educational Objective:* The biologically active form of pantothenic acid is coenzyme A, which binds with oxaloacetate in the first step of the TCA (Krebs) cycle to form citrate and then succinyl-CoA.

A 3-year-old male suffers from chronic diarrhea, greasy stool and anemia. The patient is significantly underweight. He has mild edema of the lower extremities and oral ulcers. Parenteral feeding and vitamin supplementation are started in this patient. Pantothenic acid is necessary for which of the following conversions in this patient's metabolism? A. Glucose to pyruvate B. Glucose to ribose-5-phosphate C. Alanine to glucose D. Oxaloacetate to citrate E. Glutamate to α-ketoglutarate

*The answer is B.* Glycogen is broken down by the enzyme glycogen phosphorylase, which is regulated through phosphorylation (active state) and dephosphorylation (inactive state). Phosphorylase kinase (PK) is the enzyme responsible for the phosphorylation of glycogen phosphorylase, whereas phosphoprotein phosphatase catalyzes its dephosphorylation. PK is regulated differently in liver than in muscles. Glycogen stored in the liver is used to maintain blood glucose levels during the fasting state, whereas glycogen in the muscles is used to provide energy for muscle contraction. In the liver, PK is activated primarily through the binding of epinephrine and glucagon to Gs-protein-coupled receptors, which increases cAMP concentrations and causes phosphorylation of PK (via protein kinase A). Skeletal muscle lacks glucagon receptors, but muscle PK can still be phosphorylated in response to an epinephrine-induced increase in cAMP concentrations. However, increased intracellular calcium is a more powerful activator of muscle PK. Release of sarcoplasmic calcium following neuromuscular acetylcholine stimulation allows for synchronization of skeletal muscle contraction and glycogen breakdown, providing the energy necessary for anaerobic muscle contraction. Epinephrine can also act on alpha-1 adrenergic receptors found in muscle and liver to increase intracellular calcium and promote glycogenolysis. *Educational Objective:* Synchronization of glycogen degradation with skeletal muscle contraction occurs due to release of sarcoplasmic calcium following neuromuscular stimulation. Increased intracellular calcium causes activation of phosphorylase kinase, stimulating glycogen phosphorylase to increase glycogenolysis.

A 30-year-old man comes to the office for a routine checkup. He has no significant medical problems and does not take any medications. The patient works as a fitness trainer and lilts weights recreationally. He has been consuming carbohydrate-rich food prior to his weightlifting sessions and claims that it increases muscle strength. A literature review shows that the rate of glycogenolysis within myocytes increases several hundredfold during active skeletal muscle contraction Which of the following substances is most likely responsible for the synchronization between these two processes? A. ATP B. Ca2+ C. cAMP D. Glucose-6-phosphate E. Lactate

*The answer is D.* Glucose-6-phosphate dehydrogenase (G6PD) deficiency is an X-linked recessive disorder that is particularly common in males of African or Mediterranean descent G6PD is the first enzyme in the pentose phosphate pathway, and it reduces oxidized nicotinamide adenine dinucleotide phosphate (NADP) to reduced nicotinamide adenine dinucleotide phosphate (NADPH) as it oxidizes glucose-6-phosphate. NADPH is a necessary cofactor for the enzyme glutathione reductase, which generates reduced glutathione. Reduced glutathione reduces peroxides into less harmful substances such as water and alcohol. A deficiency of G6PD makes erythrocytes more susceptible to oxidative stress, resulting in a hemolytic anemia that can be triggered by antibiotics such as sulfa drugs or bacterial infections. Heinz bodies, seen in this patient's peripheral smear in the vignette image, are a hallmark of G6PD deficiency; they represent accumulations of denatured hemoglobin in affected red blood cells. Unlike Howell-Jolly bodies, Heinz bodies only appear on peripheral smear after staining with methylene blue.

A 30-year-old man of African heritage presents to his primary care physician complaining of dizziness and lethargy. About a week ago, he finished a medicine prescribed by his doctor for bacterial sinusitis. He is normally in good health, eats a diet full of fruits and vegetables, and exercises daily. He denies any changes in his diet and has not experienced any abdominal pain or gastrointestinal distress, although he admits to tea-colored urine over the past 3-4 days. On physical examination, he appears pale and uncomfortable, with icteric sclerae. His blood pressure is 100/70 mm Hg, pulse is 110/min, and respirations are 19/min. A peripheral blood smear from the patient stained with new methylene blue is shown below. Laboratory studies show: RBC count: 3 x 106/mm 3 WBC count : 9500/mm3 Hemoglobin: 8 g/dl Hematocrit : 24% Mean co rpuscular volume (MCV) : 90 fL Which of the following is the most likely cause of this patient's anemia? A. α-Thalassemia B. Acute intermittent porphyria C. Blood donation D. Enzymatic deficiency E. Intrinsic factor deficiency F. Malaria G. Mineral deficiency H. Sickle cell disease I. Vitamin deficiency

*The answer is A.* Normocytic anemia, back pain and hemoglobinuria following oxidant stresses (dapsone therapy in this vignette) in a patient of Mediterranean descent are highly suggestive of Glucose-6-phosphate-dehydrogenase (G6PD) deficiency. G6PD deficiency is an X-linked recessive disorder most commonly found in Greeks, Italians and African Americans. The deficiency of G6PD reduces the synthesis of the reduced form of NADPH, which subsequently decreases glutathione (GSH) levels. Because GSH neutralizes hydrogen peroxide, the absence of GSH in G6PD deficiency allows peroxidase to oxidize hemoglobin. Anemia is often precipitated by oxidant stresses that induce hemolysis. Infections are the most common cause, followed by drugs including dapsone, antimalarials, and sulfonamide antibiotics.

A 32-year-old Greek male presents with a rash consistent with dermatitis herpetiformis and is treated with a course of oral dapsone. Over the following several days he becomes increasingly fatigued and experiences persistent back pain. Complete blood count reveals a normocytic anemia and urinalysis shows hemoglobinuria. Which of the following is most likely responsible for this patient's symptoms? (A) Glucose-6-phosphate dehydrogenase deficiency (B) Pyruvate kinase deficiency (C) Paroxysmal nocturnal hemoglobinuria (D) Hereditary spherocytosis (E) Cold immune hemolytic anemia

*The answer is E.* In the early fasting state, hepatic glycogenolysis is primarily responsible for maintaining blood glucose levels. Hepatic glycogen is capable of maintaining blood glucose levels for the first 10-12 hours of a fast. As hepatic glycogen is depleted, hepatic gluconeogenesis plays an increasingly major role in maintaining blood glucose. Gluconeogenesis uses lactate, glycerol, and alanine to generate glucose. This patient had his last meal approximately 8 hours before being found. At this point, hepatic glycogenolysis is the dominant process for maintaining blood glucose levels. Glycogen phosphorylase is the rate-limiting enzyme in glycogenolysis. It catalyzes the cleavage of glucose-1-phosphate residues from alpha-I ,4-glycosidic linkages in branched glycogen.

A 32-year-old man with no significant past medical history is found along a highway after being lost while hiking for the past 24 hours. He consumed all the food in his backpack 8 hours before being found. He is brought to the emergency department where his vitals are found to be a temperature of 98.7 F (37.0 C), blood pressure of 105/60 mmHg, heart rate of 95 beats/minute, and respiratory rate of 14 breaths/minute. Intravenous fluids are started, two minor lacerations on his left lower extremity are sutured, and he is given a meal. Prior to receiving medical help, which enzyme played a major role in maintaining blood glucose levels in this patient? A. Pyruvate kinase B. HMG-CoA synthase C. Phosphoructokinase-1 D. Hexokinase E. Glycogen phsophorylase

*The answer is B.* This patient is presenting with acute hemolytic anemia, which can occur in those with glucose-6-phosphate dehydrogenase (G6PD) deficiency after certain infections (eg, pneumonia, viral hepatitis), consumption of lava beans, or use of specific medications (eg, primaquine and certain sulfa drugs). GSPD catalyzes the first (rate-limiting) step in the pentose phosphate pathway (PPP). In the oxidative portion of the pathway, glucose-6-phosphde is converted to ribulose-S-phosphate and 2 molecules of NADPH are produced. The nonoxidative reactions of the PPP are used primarily to interconvert ribose-5-phosphate (used for nucleotide synthesis) into glycolytic intermediates that can be used for energy production. Because the PPP is the main source of NADPH, the pathway is particularly active in: 1. Cells experiencing high oxidative stress (eg, erythrocytes), where NADPH is used to regenerate reduced glutathione 2. Organs (eg, liver, adrenals) involved in reductive biosynthesis (eg, fatty acids, cholesterol, steroids) and cytochrome P450 metabolism 3. Phagocyte cells generating a respiratory burst (NADPH oxidase)

A 35-year-old man comes to the office due to fatigue, dark urine, and back pain. His wife returned from a business trip in Egypt several days ago and brought home a type of large fiat bean that he had never soon before. The patient ate some of the beans for dinner last night and first noticed the symptoms when he woke up in the morning. Physical examination shows jaundice and pallor. Laboratory studies show a hemoglobin level of 8 gfdL. Further evaluation reveals deficiency of an enzyme involved in the conversion of glucose to ribulose-5-phosphate. Impairment in the activity of this enzyme is most likely to inhibit which of the following biochemical pathways? A. ADP phosphorylation B. Cholesterol synthesis C. Glycogen storage D. Ketone body synthesis E. Protein degradation

*The answer is A.* During gluconeogenesis, substances such as lactate and alanine are converted to pyruvate. However, pyruvate cannot be converted to phosphoenolpyruvate directly as pyruvate kinase is unidirectional. To convert pyruvate to phosphoenolpyruvete, pyruvate first undergoes biotin-dependent carboxylation to oxaloacetate in the mitochondria. This reaction is catalyzed by pyruvate carboxylase. The activity of pyruvate carboxylase is increased by acetyl-coA, an allosteric activator of the enzyme. This critical regulatory step diverts pyruvate to pyruvate dehydrogenase when acetyl-CoA levels are too low, preventing the cell from becoming energy starved. When acetyl-CoA levels are high (indicating energy excess), pyruvate carboxylase can operate at full capacity and convert most of the pyruvate into oxaloacetate for use in gluconeogenesis.

A 35-year-old woman comes to the emergency department with nausea, vomiting, and fever. Her symptoms began 24 hours ago, and she has been unable to eat or drink anything since. She has a 3-year-old daughter who had similar symptoms 2 days earlier but is now fine. Laboratory studies show a blood glucose level of 82 mg/dL despite her 24-hour fast. Maintenance of this patient's blood glucose levels is facilitated by hepatic conversion of pyruvate into glucose. Which of the following substances is an allosteric activator of the first step of this process? A. Acetyl-CoA B. Alanine C. Citrate D. Fructose 2,6-bisphosphate E. Lactate F. Oxaloacetate

*The answer is E.* Patients with a deficiency of glucose-6-phosphate dehydrogenase (G6PD) are more sensitive to oxidative drugs (eg, sulfa drugs, antimalarials) that result in the production of reactive oxygen species (ROS). G6PD deficiency is more predominant in African Americans. Normally, when exposed to oxidative stress, RBCs shunt glucose breakdown through the hexose monophosphate pathway in which the enzyme G6PD produces NADPH from these products. NADPH allows for glutathione reductase to regenerate glutathione that acts as a sink for ROS and protects hemoglobin and membranes from oxidative damage. However, when there is a G6PD deficiency, RBCs cannot produce enough reduced glutathione, resulting in oxidative damage of proteins and membranes. This causes increased hemolysis that is clinically evident as hyperbilirubinemia (jaundice, as shown in the image). Hemolysed RBCs appear as small fragments, sometimes smaller than platelets on a blood smear .

A 36-year-old African-American man is prescribed trimethoprim-sulfamethoxazole for a bacterial infection. Three days after starting the drug, he returns to the clinic complaining of fatigue and shortness of breath. Physical examination is notable for the findings shown in the image. Underproduction of which of the following molecules is most likely responsible for this patient's reaction to the drug? A. α-Globin chain B. Glucose-6-phosphate C. Phosphoribosylpyrophosphate D. Reactive oxygen species E. Reduced glutathione

*The answer is A.* Under normal conditions, when oxygen is readily available, the pyruvate generated in glycolysis enters the mitochondria and is convened into acetyl-CoA by the action of pyruvate dehydrogenase, In severe exercise. particularly by individuals in poor physical condition, the oxygen demands of the skeletal muscle may exceed the ability of the heart and lungs to provide oxygen. In this setting, the muscles switch to anaerobic glycolysis and the pyruvate that is produced is converted to lactate by the action of lactate dehydrogenase, Much of the lactic acid thereby produced is released into the bloodstream where it causes a metabolic acidosis. In addition to the setting of severe exercise. lactic acidosis can also be seen in tissue hypoxia (seizures. cardiac failure. hypotension. carbon monoxide poisoning, severe anemia), with drug toxicity and toxins (phenformin, catecholamines, salicylate, isoniazid, cyanide), in congenital defects in gluconeogenic enzymes, and in many severe illnesses. In chronic obstructive pulmonary disease (COPD), patients are asked to exercise as part of their treatment. Exercise builds muscle mass that patients with COPD often do not have. COPD patients work hard to breathe; the muscles have a low threshold for anaerobic glycolysis that leads to lactic acidosis. The patient then tries to compensate for the lactic acidosis by respiratory alkalosis; however, because of the COPD he cannot compensate effectively. Exercise increases the muscle mass, leading to an increase in mitochondria. This in turn leads to more ATP, less anaerobic glycolysis, and a less dramatic need for respiratory compensation.

A 38-year-old man collapses while running a marathon and is brought to the emergency department. His wife states that he is not in shape but still decided to run. Arterial blood gas on room air shows PO2 of 100 mm Hg, PCO2 of 42 mm Hg, and pH of 7.30. Further evaluation shows increased lactate dehydrogenase activity. This enzyme is involved which of the following biochemical pathways? A. Anaerobic glycolysis B. Beta-oxidation of fatty acids C. Citric acid cycle D. Mitochondrial oxidative phosphorylation E. Pentose phosphate shunt

*The answer is D.* Phosphoenolpyruvate (PEP) carboxykinase functions as a regulato ry enzyme in the gluconeogenesis pathway. It acts in the cytosol to convert oxaloacetate to phosphoenolpyruvate.

A second-year medical student is under a lot of stress due to her upcoming biochemistry examination. She knows that physiological stress leads to release of endogenous glucocorticoids, which increases her blood glucose levels. To raise glucose levels, glucocorticoids activate the enzyme phosphoenolpyruvate carboxykinase to generate which of the following products? A. Acetyl-CoA B. Malate C. Oxaloacetate D. Phosphoenolpyruvate E. Pyruvate

*The answer is D.* The amino acid degradation pathways that produce oxaloacetate and pyruvate can also be used to synthesize glucose, and are commonly stressed in biochemistry books. These pathways are important in true starvation. Also stressed in biochemistry books is that acetyl-coA cannot be used for gluconeogenesis and thus fatty acids are not gluconeogenic. But this does not mean that "fat" cannot be used to make at lease some glucose. since the stored fat is predominately in the form of triglycerides and the glycerol backbone is glycogenic. Specifically, glycerol can be phosphorylated by glycerol kinase (which uses ATP) to form glycerol-3-phosphate, which can in turn enter the gluconeogenic pathway via oxidation to dihydroxyacetone phosphate This pathway contributes about 80% of the glucose in dieting obese individuals, and about 20% of the glucose (behind the amino acid pathways) in truly starving individuals.

A 38-year-old obese man comes to the physician for a routine examination. He says that he has been on a strict diet with very small amounts of carbohydrates in an effort to lose weight. Physical examination shows no abnormalities. Laboratory studies show a low-normal level of blood glucose. Which of the following is the source of the majority of the newly synthesized glucose in this patient? A. Amino acids from protein degradation B. Ethanol synthesized by bacteria C. Fatty acids from fat breakdown D. Glycerol from fat breakdown E. Purine catabolism

*The answer is E.* The child has a lack of branching enzyme activity, another glycogen storage disease, type IV (Andersen disease). In this case, the glycogen produced is a long, straight chain amylopectin, which has limited solubility, and precipitates in the liver (recall, the liver has the highest concentration of glycogen of all tissues). This leads to early liver failure (thus, the high bilirubin and transaminases in the serum) and death if a liver transplant is not performed. Defects in any of the other enzymes listed would lead to a different clinical scenario. Lack of glycogen phosphorylase or synthase, within the liver, would lead to fasting hypoglycemia, but not liver failure. Lack of these enzymes in the muscle would lead to exercise intolerance but would not affect blood glucose levels. Lack of α-glucosidase is Pompe disease, which also leads to an early death, but is due to the lack of a lysosomal enzyme, and there is no glycogen precipitation within the body of the liver. A lack of debranching activity is glycogen storage disease III, but would also lead to fasting hypoglycemia, without glycogen precipitation within the liver.

A 4-month-old infant is seen by the pediatrician for failure to thrive. Examination shows distinct hepatosplenomegaly. Lab results show elevated transaminases and bilirubin, suggestive of liver failure. The boy dies shortly thereafter, and upon autopsy, precipitated carbohydrate was found throughout the liver. The boy most likely had a mutation in which of the following enzymes? (A) Glycogen phosphorylase (B) Debranching enzyme (C) Glycogen synthase (D) β-glucosidase (E) Branching enzyme

*The answer is D.* Glucocerebrosides would accumulate in the cells because the missing enzyme is glucocerebrosidase.

A 40-year-old woman with a history of bleeding and pancytopenia now presents with leg pain. She describes a deep, dull pain of increasing severity that required pain medication. Computed tomography examination reveals erosion and thinning of the femoral head. A bone marrow biopsy is performed to confirm a diagnosis of Gaucher disease. What material would be found abnormally accumulating in the lysosomes of her cells? A. Mucopolysaccharide B. Ganglioside C. Ceramide D. Cerebroside E. Sulfatide

*The answer is C.* Decreased lactate production in the erythrocyte indicates a defect in glycolysis. Among patients exhibiting genetic defects of glycolytic enzymes, about 95% show a deficiency in pyruvate kinase. Pyruvate kinase deficiency is the second most common cause (after glucose 6- phosphate dehydrogenase deficiency) of enzyme deficiency- related hemolytic anemia.

A 43-year-old man presented with symptoms of weakness, fatigue, shortness of breath, and dizziness. His hemoglobin level was less than 7 g/dl (normal for a male being greater than 13.5 g/dl). Red blood cells isolated from the patient showed abnormally low level of lactate production. A deficiency of which one of the following enzymes would be the most likely cause of this patient's anemia? A. Phosphoglucose isomerase B. Phosphofructokinase C. Pyruvate kinase D. Hexokinase E. Lactate dehydrogenase

*The answer is E.* The majority of ATP used for cellular processes is generated by the oxidation of acetate in the tricarboxylic acid (TCA) cycle. The enzymes of the TCA cycle are located in the mitochondria and generate reduced nicotinamide adenine dinucleotide (NADH) and flavin adenine dinucleotide (FADH,) (Choices C, D, F, and H). These molecules drive the process of oxidative phosphorylation, which converts their reducing potential into high-energy ATP via the electron transport chain. ATP can also be generated by substrate-level phosphorylation, which involves the direct transfer of a phosphate group from a reactive intermediate to a nucleotide diphosphate (eg, ADP, GDP). Succinyl-CoA synthetase converts succinyl-CoA to succinate and uses the high energy thioester present in succinyl-CoA to drive GTP synthesis. This GTP can then be used to transphosphorylate ADP to ATP, or it may be utilized by specific GTP-hydrolyzing enzymes, such as phosphoenolpyruvate carboxykinase (converts oxaloacetate to phosphoenolpyruvate during gluconeogenesis). *Educational Objective:* GTP is synthesized by succinyl-CoA synthetase during the conversion of succinyl-CoA to succinate in the citric acid cycle. During gluconeogenesis, phosphoenolpyruvate carboxykinase uses GTP to synthesize phosphoenolpyruvate from oxaloacetate .

A 43-year-old man prospecting for gold in Arizona becomes stuck in the desert after his truck breaks down. He brought a large supply of water with him but only a few granola bars as food. After 3 days, he is able to flag down a passing vehicle and obtain transportation to the nearest settlement. During this ordeal, his liver begins to synthesize large quantities of glucose from source molecules such as alanine, lactate, and glycerol. As part of this process, phosphoenolpyruvate is formed from oxaloacetate in a reaction that requires a specific nucleoside triphosphate as a cofactor. Which of the following reactions directly synthesizes this cofactor?

*The answer is C.* Starch blockers contain a natural inhibitor of salivary amylase, which will block starch digestion in the mouth. The other amylase that digests starch, pancreatic amylase, would only be inhibited by the starch blocker if the inhibitor would survive the conditions of the acidic stomach without becoming denatured. There is no gastric amylase. Intestinal enteropeptidase activates trypsinogen, which is required for protein digestion, not starch digestion. Pancreatic lipase is required for dietary triglyceride digestion, and is not active toward starch.

A 50-year-old man has been trying to lose weight, but he enjoyed eating so much that he found it difficult to do so. He then reads about a product in the popular press, which guarantees that he can lose weight, as caloric intake due to starch ingestion will be reduced (a starch blocker). The over-the-counter product that he buys is claimed to inhibit which of the following enzymes? (A) Pancreatic trypsinogen (B) Pancreatic lipase (C) Salivary amylase (D) Gastric amylase (E) Intestinal enteropeptidase

*The answer is D.* Patients with chronic alcoholism are frequently deficient in thiamine, a necessary cofactor for pyruvate dehydrogenase, α-ketoglutarate dehydrogenase, and transketolase. Administration of glucose to thiamine-deficient patients can cause rapid depletion of the small amount of thiamine remaining in the circulation. This can result in neuronal injury within highly metabolic brain regions, leading to acute Wernicke encephalopathy. The metabolism of ethanol by alcohol dehydrogenase and aldehyde dehydrogenase consumes NAD+ and increases the NADH to NAD+ ratio This skewed ratio inhibits all pathways requiring NAD+; as a result, the entire citric acid cycle is inhibited. However, in the setting of Wernicke encephalopathy. thiamine-dependent enzymes are especially affected due to the lack of NAD+ and thiamine (Choices A, B, C, E, F, G, and H). *Educational Objective:* Pyruvate dehydrogenase and α-ketoglutarate dehydrogenase require thiamine as a cofactor. Administration of glucose to thiamine~deficient patients (eg, alcoholics) can result in Wernicke encephalopathy (eg, acute confusion, ophthalmoplegia, and ataxia) due to increased thiamine demand.

A 44-year-old homeless man is brought to the emergency department after police officers found him agitated and confused. During transport to the hospital, he is started on intravenous fluids with dextrose. On arrival, the patient is disorientated but cooperative. Physical examination shows brutes on his forehead, forearms, and shins. Extraocular findings include bilateral horizontal nystagmus and decreased lateral eye movements. He also has an unsteady gait with widely-spaced legs and short steps. The ambulance personnel state that the patient's extraocular movements were intact when they picked him up. A review of the medical record shows that the patient has been admitted to the hospital with alcohol intoxication several times before. Which of the following reactions is likely to be the most impaired in this patient?

*The answer is C.* Only choice C is characteristic of hemolytic jaundice; indirect hyperbilirubinemia with no spillover of the water-insoluble unconjugated form into the urine.

A 44-year-old man from Limpopo Province in South Africa, living in the United States and receiving antibiotic therapy for a urinary tract infection, has a self-limiting episode of hemolysis, back pain, and jaundice. The peripheral blood smear reveals a nonspherocytic, normocytic anemia, and Heinz bodies are seen in some of his erythrocytes. Which of the following sets of lab results would most likely have been obtained for this patient?

*The answer is B.* Fructose 2,6-bisphosphate helps control the balance between gluconeogenesis and g\lycolysls through inverse regulation of phosphofructokinase-1 (PFK-1) and fructose 1,6-bisphosphatase. Fructose 2,6-bisphosphate activates PFK-1, the main regulatory enzyme involved in glycolysis, which converts fructose 6-phosphate to fructose 1,6-bisphosphate. The opposite reaction (fructose 1,6-bisphosphate to fructose-6-phosphate) occurs in gluconeogenesis and is catalyzed by the enzyme fructose-1.6-bisphosphatase (inhibited by fructose 2,6-bisphosphate). The interconversion of fructose-6-phosphate and fructose 2,6 bisphosphate is achieved by a bifunctional enzyme complex composed of PFK-2 (increases fructose 2,6~bisphosphate levels) and fructose 2,6 bisphosphosphatase (decreases fructose 2,6-bisphosphate levels). Insulin causes activation of PFK-2, leading to increased fructose 2,6-bisphosphate levels and augmented glycolysls. High concentrations of fructose 2,6-bisphosphate also inhibit gluconeogenesis leading to decreased conversion of alanine and other gluconeogenic substrates to glucose.

A 45-year-old man is referred to an endocrinologist for newly diagnosed diabetes mellitus. A week ago, his primary care physician noted an elevated fasting serum glucose level. The endocrinologist discusses the different treatment options available, including oral and injectable medications. He recommends treatment with a medication that alters glucose metabolism within the liver by increasing the concentration of fructose 2,6-bisphosphate within hepatocytes. Which of the following conversions will be inhibited by high intracellular concentrations of this metabolite? A. Acetyl CoA → fatty acids B. Alanine → glucose C. Fructose-6-phosphate → fructose~1,6-bisphosphate D. Glucose → glycogen E. NAD+ → NADH

*The answer is B.* The patient is suffering from the potential side effects of statins, namely, muscle damage and pain. This may occur due to an inhibition of coenzyme Q synthesis (which requires a product derived from mevalonic acid) and a lack of energy generation in the muscle. The reason this comes about is that statins are detoxified through a cytochrome P450 system, and the particular system that works on statins is inhibited by grapefruit juice. Thus, in the presence of grapefruit juice, the effective intracellular levels of statins are higher than in the absence of the juice, due to the decreased rate of destruction. The artificially induced higher levels of statins then lead to muscle damage. Statins inhibit the conversion of HMG-CoA to mevalonic acid (catalyzed by HMG-CoA reductase). Thus, answers indicating that mevalonic acid levels are increasing cannot be correct; they are reduced in the presence of a statin. Statins do not bring about calcium efflux from the sarcoplasmic reticulum.

A 45-year-old man was diagnosed with hypercholesterolemia, for which he was prescribed a statin. After a month on medications, the patient decided to adopt a healthier lifestyle and replaced eggs and coffee for breakfast with fruit juices and whole-wheat toast. Within 2 weeks of changing his diet, the man developed severe muscle pain in his arms and shoulders. The muscle pain could be the result of which of the following? (A) Induction of a detoxifying cytochrome P450 system (B) Inhibition of a detoxifying cytochrome P450 system (C) Increased mevalonate inhibiting actin/myosin interactions (D) Increased mevalonate stabilizing actin/myosin interactions (E) HMG-CoA stimulation of calcium effl ux from the sarcoplasmic reticulum

*The answer is E.* This patient's blood gases reveal a metabolic acidosis (low pH, low HCO 3-). The decrease in arterial CO2 resulting from increased breathing rate, is an appropriate compensatory response to the acidosis. The smell of acetone (the primary solvent in nail-polish remover) on the breath of a diabetic suggests ketoacidosis. This occurs when uncontrolled diabetes mellitus leads to overproduction of ketoacids and high levels of plasma acetone. Because acetone is highly volatile, it is excreted mainly by the lungs. Ketoacids, which include acetoacetate and 3-hydroxybutyrate, are normally produced in roughly equal amounts. However, in pathologic conditions such as diabetes and alcoholism, the ratio shifts to 3-hydroxybutyrate predominating. Therefore, the ratio of acetoacetate to 3-hydroxybutyrate actually decreases, while the net amount of ketoacids increases. In this situation the physician should specifically request a lap measurement of 3-hydroxybutyrate to have a more accurate measurement of the extent of the initial level on admission and to more accurately follow the resolution during treatment. Common causes of ketoacidosis include infection, missed insulin treatments, myocardial infarction cerebrovascular accident, and stress trauma.

A 45-year-old man with a history of poorly controlled diabetes mellitus is admitted to the hospital because of severe hyperglycemia. The patient has experienced increased thirst and frequent urination and has had episodes of nausea and vomiting. His temperature is 37.2°C (99.0°F}, pulse is 115/min. respirations are 25/min, and blood pressure is 138/68 mm Hg. Physical examination shows dry skin and dry mucous membranes and his breath smells like nail polish remover. An arterial blood gas shows the following: Arterial CO 29 mm Hg Arterial pH 7.22 Arterial HCO 15 mEq/L Which of the following measurements is most likely to be decreased in this patient compared to the normal range? A. Plasma acetone B. Plasma catecholamines C. Plasma glucagon D. Plasma 3-hydroxybutyrate E. Ratio of acetoacetate to 3-hydroxybutyrate

*The answer is B.* Glucose enters pancreatic β cells via GLUT 2 channels. GLUT 2 transporters are insulin-dependent glucose channels that are present on pancreatic β cells when glucose levels are low.

A 6-year-old boy is brought to his pediatrician's office by his parents, who report that the child has been unusually thirsty for the past week. He also has increased urinary frequency and has wet the bed three times in the past two weeks. A random blood glucose level is 215 mg/dL. The pediatrician suspects that the child has type 1 diabetes mellitus caused by autoimmune destruction of insulin-producing pancreatic b cells. Which of the following is the transporter for glucose to enter pancreatic β cells? (A) GLUT 1 (B) GLUT 2 (C) GLUT 4 (D) Simple diffusion

*The answer is D.* This patient presents with ophthalmoplegia, ataxia, confusion, and a memory deficit, which he tries to cover by inventing bizarre explanations (confabulation). This constellation of neurologic symptoms is characteristic of Wernicke-Korsakoff syndrome, which is caused by thiamine deficiency. In the United States severe thiamine deficiency is seen most commonly in chronic alcoholics. Thiamine deficiency can also damage peripheral nerves ("dry" beriberi) and the heart ("wet" beriberi). Dietary deficiency is more common in countries in which "polished rice" (rice with hulls removed) is a dietary staple. Pyruvate dehydrogenase is a mitochondrial enzyme that catalyzes the conversion of pyruvate to acetyl-coA. Cofactors and coenzymes used by pyruvate dehydrogenase include thiamine pyrophosphate (TPPQ from the vitamin thiamine), lipoic acid, coenzyme A (from pantothenate), FAD[H2] (from riboflavin), and NAD[H] (from niacin).

A 47-year-old man with a history of alcoholism is brought to the emergency department because he was found wandering in the streets. He appears disoriented and walks unsteadily. Physical examination shows an inability to adduct the right eye or to abduct the left eye. When asked about recent events that may have resulted in his condition, he responds with several elaborate but obviously fictitious stories. Altered activity of which of the following enzymes is most likely contributing to his symptoms? A. Fructose 1-phosphate aldolase B. Glucose 6-phosphate dehydrogenase C. Pyruvate carboxylase D. Pyruvate dehydrogenase E. Pyruvate kinase

*The answer is A.* This patient is likely suffering from Wernicke encephalopathy, a syndrome caused by a thiamine deficiency that is most commonly seen in malnourished alcoholics. It results in ataxia, ophthalmoplegia, and confusion. The mechanism of the deficiency is a combination of malnutrition as well as impaired absorption secondary to alcohol consumption. Long-standing thiamine deficiency leads to the irreversible Korsakoff syndrome, which is a late manifestation of Wernicke encephalopathy characterized by anterograde and retrograde amnesia. Thiamine, also known as vitamin B1, is used as a cofactor for multiple enzymes, including α-ketoglutarate dehydrogenase, pyruvate dehydrogenase, and transketolase. The presence of macrocytic anemia is common in alcoholics due to the toxic effects of alcohol on the bone marrow. They can also have megaloblastic anemia due to lack of folate caused by chronic alcohol consumption. The lack of folate will disrupt DNA synthesis, leading to megaloblastic anemia. Importantly, alcoholics can also exhibit microcytic anemia if they have a chronic underlying infection (anemia of chronic disease) or if they suffer from bleeding esophageal varices (iron deficiency anemia). Other clues that this patient is an alcoholic include the known history of esophageal varices, witnessed fall, disorientation, combativeness, and inability to track the physician's finger .

A 49-year-old man with known esophageal varices is brought to the emergency department after a witnessed fall while crossing the street. On arrival, he is mildly combative, and he is not oriented to where he is or what year it is. He is also unable to state how he got to the hospital. On physical examination, his vital signs are within normal limits, but he is unable to track the physician's finger from side to side. Laboratory findings are significant for a mild macrocytic anemia, but no hypersegmented neutrophils are seen. Imaging is negative for any cerebral lesions. The deficient vitamin in this patient serves as a cofactor for what enzyme? A. α-Ketoglutarate dehydrogenase B. δ-Aminolevulinate synthase C. Dopamine hydroxylase D. Homocysteine methyltransferase E. Pyruvate carboxylase

*The answer is A.* The symptoms suggest fructose intolerance, a deficiency in aldolase B. Deficiencies in fructokinase or galactokinase result in relatively benign conditions characterized by elevated levels of fructose or galactose in the blood and urine. Deficiency in β-galactosidase (lactase) results in a decreased ability to degrade lactose (milk sugar). Congenital lactase deficiency is quite rare and would have presented much earlier in this baby, and with different symptoms. Typical lactase deficiency (adult hypolactasia) presents at a later age. The symptoms of glucose 6-phosphatase deficiency would result from fasting, and would not be related to ingestion of fruit juice.

A 5-month-old boy is brought to his physician because of vomiting, night sweats, and tremors. History revealed that these symptoms began after fruit juices were introduced to his diet as he was being weaned off breast milk. The physical examination was remarkable for hepatomegaly. Tests on the baby's urine were positive for reducing sugar but negative for glucose. The infant most likely suffers from: A. aldolase B deficiency. B. fructokinase deficiency. C. galactokinase deficiency. D. β-galactosidase deficiency. E. glucose 6-phosphatase deficiency.

*The answer is A.* This patient most likely has glycogen storage disease type II (Pompe disease). This condition is caused by a deficiency of acid α-glucosidase (acid maltase), an enzyme responsible for breaking down glycogen within the acidic environment of lysosomes. Although most glycogen is degraded in the cytoplasm, a small amount is inadvertently engulfed by lysosomes, especially in cells containing high amounts of glycogen such as hepatocytes and myocytes. As such, deficiency of acid maltase results in pathologic accumulation of glycogen within liver and muscle lysosomes. Cardiac and skeletal muscle are particularly susceptible, as the ballooning lysosomes interfere with contractile function. The classic form of the disease presents in early infancy with marked cardiomegaly, severe generalized hypotonla. macroglossia. and hepatomegaly. Blood glucose levels are normal, unlike with glycogen storage diseases that primarily affect the liver (eg, von Gierke}. A key distinguishing feature is that muscle biopsy will show accumulation of glycogen In lysosomes. *Educational Objective:* Acid maltose (α-glucosidase) deficiency presents in early infancy with cardiomegaly, macroglossia, and profound muscular hypotonia. Abnormal glycogen accumulation within lysosomal vesicles is soon on muscle biopsy.

A 5-month-old boy is brought to the office due to poor feeding. His mother says that he has difficulty holding his head up while breastfeeding and his suckling seems weaker than it used to be. His current weight is between the 5th-10th percentile, and length and head circumference are tracking along the 25th percentile. Physical examination shows hepatomegaly and hypotonia in all 4 limbs. Cardiac auscultation reveals a gallop rhythm, and chest x-ray shows severe cerdiomegely. Muscle biopsy shows enlarged lysosomes containing periodic acid-Schiff (PAS)-positive material. Which of the following enzymes is most likely deficient in this patient? A. Acid α-glucosidase B. Debnancher enzyme C. Galactokinase D. Glucose-6-phosphatase E. Glycogen phosphorylase F. Pyruvate kinase

*The answer is A.* Pompe disease is a type II glycogen storage disease that primarily affects the heart ("Pompe trashes the pump"). Lysosomal α-1,4-glucosidase is absent, which is necessary for the hydrolysis of the outer branches of glycogen. As a result, glycogen is deposited in the myocardium, and by the sixth month of life children experience developmental delay, feeding problems, and eventual heart failure. Skeletal muscle and the liver are also affected. ECG shows short PR intervals with large QRS complexes evidencing biventricular hypertrophy; cardiomegaly is also evident on x- ray of the chest.

A 5-month-old child has cardiomyopathy, an enlarged tongue, severe generalized muscular hypotonia, and hepatomegaly. Short intervals with giant QRS complexes are seen on ECG, and blood work reveals elevated serum creatine kinase and decreased leukocyte acid maltase levels. Which enzyme is deficient in this infant? A. α-1,4-Glucosidase B. α-Galactosidase C. β-Glucocerebrosidase D. Giucose-6-phosphatase E. Glycogen phosphorylase

*The answer is A.* This child is presenting with the typical signs and symptoms of lead poisoning, which affects the activity of the enzyme aminolevulinate (ALA) dehydratase. Lead poisoning is a chronic disorder often seen in children who are exposed to environmental sources of lead, such as paint chips, putty, or plaster from old buildings which the child may ingest. Lead poisoning can lead to a variety of constitutional symptoms including lethargy, headache, memory loss. nausea, abdominal pain, and diarrhea or constipation. With high serum lead levels, lead lines may appear in the gums (usually seen in adults more than in children) and in bones on x-ray (lead lines that are opaque metaphysical bands in growing bone). Inhibition ofALA dehydratase and ferrochelatase can result in microcytic, hypochromic anemia due to impaired heme synthesis. Treatment with dimercaprol (which crosses the blood-brain barrier and so is the drug of choice in lead encephalopathy) or chelation therapy with ethylenediaminetetraacetic acid (EDTA) is indicated in cases with high blood lead levels. Chelating agents enhance lead excretion in urine. Lead also inhibits 5' nucleotidase activity, producing basophilic stippling of erythrocytes due to retained RNA.

A 5-year-old girl is brought to the emergency department because of increasing irritability, poor attention span. and lethargy. She also has anorexia, vague abdominal pain, and constipation. Physical examination shows a pale girl who is somewhat somnolent, with reduced reflexes in all extremities, Treatment is begun with intravenous ethylenediaminetetraacetic acid (EDTA). The toxicity responsible for this patient's condition most likely results from inhibition of which of the following enzymes? A. Aminolevulinate dehydratase B. Ceruloplasmic C. Glucose-6-phosphate dehydrogenase D. Pyruvate kinase E. Uroporphyrinogen I synthase

*The answer is C.* This patient 's presentation is suggestive of diabetic ketoacidosis, an emergency caused by insulin deficiency in patients with type 1 diabetes mellitus (OM). (Patients with type 2 DM are at risk for hyperosmolar nonketotic acidosis, as they still produce some insulin.) The diagnosis is based on the presence of he following: blood glucose >200 mg/dl, arterial pH < 7.3, bicarbonate< 15 mEq/L, and ketonemia or ketonuria. Hyperglycemia causes an osmotic diuresis, leading to dehydration and electrolyte disturbances. One major effect of insulin deficiency is increased lipolysis, which releases free fatty acids into the circulation; the liver metabolizes them into ketone bodies (primarily β-hydroxybutyrate and acetoacetate). Therefore, this patient will have ketone bod ies in her blood. The presence of these acids causes a potentially life-threatening metabolic acidosis. Acidosis leads to shifting of potassium into the extracellular fluid, leading to the false appearance of hyperkalemia. In fact, urinary losses of potassium lead to a total body potassium deficit that will manifest when acidosis is corrected. Treatment consists of aggressive fluid resuscitation, insulin administration, and potassium replacement. Histologic examination of the pancreas sometime prior to presentation may reveal a reduction in the number and size of islets, as well leukocytic infiltration of the islets, representing insulitis involving T lymphocytes

A 5-year-old girl presents to the emergency department with vomiting and diffuse abdominal pain. Upon further questioning, her mother states that she had recently been ill with a "cough and runny nose" and fever, which she believes she got at daycare. Her mother states that over the past few days she has gotten worse, and she has gone back to wetting her bed. Her vitals are heart rate 120 beats/min, blood pressure is 80/60 mm Hg, respirations are 26/min, and temperature is 102°F. Physical exam is notable for dry oral mucosa, tachycaridia, and diffuse abdominal tenderness. Laboratory tests show: Na+: 130 mEq/L K+ : 5.2 mEq/L Cl- : 94 mEq/L HCO3-: 9 mEq/L Blood urea nitrogen : 35 mg/dL Creatinine: 1.2 mg/dL Glucose: 368 mg/dL Arterial blood gas analysis reveals a pH of 7.20 Histologic examination one year prior to the presentation of the organ responsible for her symptoms will reveal which of the following? A. Amyloidosis B. Granulomas C. Lymphocytes D. Monocytes E. Neutrophils F. Viral inclusion bodies

*The answer is D.* The patient is experiencing ischemic reperfusion injury. When oxygen delivery to cardiac cells was compromised, the electron transfer chain in the mitochondria was fully reduced, as the terminal oxygen acceptor was missing. When oxygen is reintroduced to the cell, at a high concentration, the likelihood of electron transfer from reduced coenzyme Q to oxygen is increased, such that the possibility of superoxide generation is increased. The superoxide produced reacts with lipids and proteins and can lead to cell death above that originally occurring from the initial heart attack. Radicals do not form during the ischemic event since oxygen is missing from the tissues. There is no effect on glycogen stores or the HMP shunt pathway under these conditions. Intimal narrowing occurs over a long time period, not over the short time period covered in this case. Injured cells do leak enzymes into the bloodstream, but these enzymes do not cause the death of other, healthy cells.

A 52-year-old male complained of sudden onset of left-sided chest pain radiating down his left arm. Rapid breathing, sweating, and a feeling of doom accompanied this. He was rushed to the emergency department. An angiogram revealed 90% occlusion of the left anterior descending artery (LAD) and no occlusions in any other artery. The LAD was opened by angioplasty. However, shortly after this procedure, with normal blood flow through the LAD, the patient's condition worsened. This was most likely due to which of the following? (A) Disruption of the HMP shunt in cardiac cells (B) Damage to healthy cells by loss of essential enzymes from cells due to membrane damage (C) Development of intimal narrowing in another artery (D) Radical-induced damage once blood flow was reinitiated (E) Lack of glycogen for ATP synthesis in the heart

*The answer is C.* Aldose reductase catalyzes the breakdown of glucose into sorbitol. Sorbitol is then metabolized to fructose, a process that is relatively slow. In patients with hyperglycemia, as would be present in this patient with poorly controlled diabetes, sorbitol accumulation with the cells of the lens leads to a rise in intracellular osmolality, causing water movement into the cells. This results in cellular swelling and osmotic damage. It also leads to a decrease in intracellular myoinositol, interfering with cellular metabolism. Swelling of lens fiber cells can lead to rupture and cataract formation. Inhibition of aldose reductase could decrease sorbitol accumulation in the lens and thus prevent cataract formation. No drug is currently approved to inhibit aldose reductase, but aldose reductase inhibitors such as epalrestat and ranirestat are currently being tested.

A 52-year-old man with a 12-year history of poorly controlled diabetes mellitus presents to his physician complaining of changes in his vision. Physical examination reveals opacities on the lens of the eye similar to those seen in this image. Which enzyme most likely contributed to this complication? ' (A) 3-Hydroxy-3-methylglutaryl coenzyme A reductase (B) Adenosine deaminase (C) Aldose reductase (D) Galactose-1-phosphate uridyltransferase (E) Hexokinase (F) Insulin-like growth factor

*The answer is D.* HbA1c is glycosylated HbA and is produced slowly whenever the glucose in blood is elevated. It persists until the RBC is destroyed and the Hb degraded and so is useful as a long-term indicator of glucose level.

A 54-year-old man with type 1 (IDDM) diabetes is referred to an ophthalmologist for evaluation of developing cataracts. Pre-appointment blood work was requested and the results are shown below: Which of the following best indicates that the blood glucose in this patient has been elevated over a period of weeks? A. Presence of ketone bodies B. Hyperglycemia C. Lipemia D. Elevated HbA1c E. Lipoprotein lipase

*The answer is B.* Aldose reductase is rich in lens and nerve tissue (among others) and converts glucose to sorbitol, which causes the osmotic damage. In galactosemia, this same enzyme converts galactose to galactitol, also creating cataracts.

A 54-year-old man with type 1 (IDDM) diabetes is referred to an ophthalmologist for evaluation of developing cataracts. Pre-appointment blood work was requested and the results are shown below: Which of the following enzymes is most strongly associated with cataract formation in this patient? A. Galactokinase B. Aldose reductase C. Glucokinase D. Galactose 1-P uridyl transferase E. Aldolase B

*The answer is A.* Because the diabetes is not being well controlled, assume the response to insulin is low and the man would have overstimulated glucagon pathways.

A 54-year-old man with type 1 (IDDM) diabetes is referred to an ophthalmologist for evaluation of developing cataracts. Pre-appointment blood work was requested and the results are shown below: Which of the following enzymes would be more active in this patient than in a normal control subject? A. Hormone-sensitive lipase B. Glucokinase C. Fatty acid synthase D. Glycogen synthase E. Lipoprotein lipase

*The answer is B.* After an overnight fast (plasma glucose 73 mg/dL), the liver is producing glucose and glucokinase activity would be insignificant (high Km, low insulin). The other proteins would be needed for aerobic glucose oxidation in the brain or for hepatic gluconeogenesis.

A 55-year-old alcoholic was brought to the emergency department by his friends. During their usual nightly gathering at the local bar, he had passed out and they had been unable to revive him. The physician ordered an injection of thiamine followed by overnight parenteral glucose. The next morning the patient was alert and coherent, serum thiamine was normal, and blood glucose was 73 mg/dL (4 mM). The IV line was removed and he was taken home. At the time of discharge from the hospital, which of the following proteins would have no significant physiologic activity in this patient? A. Malate dehydrogenase B. Glucokinase C. α-Ketoglutarate dehydrogenase D. GLUT 1 transporter E. Phosphofructokinase-1

*The answer is E.* Most important TPP-dependent enzymes include pyruvate dehydrogenase, α-ketoglutarate dehydrogenase, and transketolase. Transketolase is in the HMP shunt and is not strictly essential for glucose oxidation.

A 55-year-old alcoholic was brought to the emergency department by his friends. During their usual nightly gathering at the local bar, he had passed out and they had been unable to revive him. The physician ordered an injection of thiamine followed by overnight parenteral glucose. The next morning the patient was alert and coherent, serum thiamine was normal, and blood glucose was 73 mg/dL (4 mM). The IV line was removed and he was taken home. Which of the following enzymes is thiamine-dependent and essential for glucose oxidation in the brain? A. Transketolase B. Transaldolase C. Succinyl-CoA thiokinase D. Acetyl-CoA carboxylase E. Pyruvate dehydrogenase

*The answer is D.* This patient presents with altered mental status, hyperventilation, and lactic acidosis with hypoglycemia. This presentation is consistent with ethanol-induced hypoglycemia. Oxidation of ethanol produces the reduced form of nicotinamide adenine dinucleotide (NADH) in the liver via two key enzymes: alcohol dehydrogenase and acetaldehyde dehydrogenase. A high ratio of NADH to oxidized nicotinamide adenine dinucleotide (NAD) induces pyruvate metabolism to lactic acid, which leads to lactic acidosis. Note the anion gap and the high lactate level. This elevated NADH / NAD ratio limits the supply of pyruvate needed for gluconeogenesis, hence hypoglycemia ensues.

A 55-year-old man is found unresponsive and breathing rapidly in his apartment. His daughter found him while stopping by to visit. She stated that she was concerned after he told her on the phone that he was "drowning his sorrows," having been fired from his job earlier that day. He has no significant medical history other than moderate hypertension, for which he takes a β-blocker. Relevant laboratory findings are: Na+: 135 mEq/L K+: 4.5 mEq/L Cl-: 95 mEq/L HCO3-: 9 mEq/L Glucose: 40 mEq/L Serum pH: 6.8 Lactate: 9.5 mmoL/L What metabolic process induced this patient's current condition? (A) Decreased levels of glycerol 3-phosphate (B) Elevated pyruvate levels (C) Inappropriate induction of gluconeogenesis (D) Overproduction of reduced nicotinamide adenine dinucleotide (E) Thiamine deficiency

*The answer is B.* Dietary fructose is obtained mainly from fruits, vegetables, honey, table sugar (sucrose), and processed foods. Fructose is rapidly absorbed in the proximal small bowel by the hexose transporter GLUT 5. initial metabolism of fructose involves three enzymes fructokinase, aldolase B, and triokinase. Fructose is phosphorylated on the first carbon by hepatic fructokinase, yielding fructose-1-phosphate. Metabolism of fructose-1-phosphate by aldolase B generates dihydroxyacetone phosphate (DHAP) and glyceraldehyde. Glyceraldehyde is then phosphorylated to glyceraldehyde-3-phosphate (GSP), an intermediate of glycolysis, by triose kinase. DHAP can also be converted to G3P by triosephosphate isomerase Aldolase B deficiency causes the potentially life-threatening disorder known as hereditary fructose intolerance. Patients typically present when fructose-containing foods are introduced into the diet. The primary manifestations are vomiting and hypoglycemia about 20-30 minutes after fructose ingestion. Hypoglycemia results from intracellular accumulation of fructose-1-phosphate and depletion of inorganic phosphate, which inhibit glycogenolysis and gluconeogenesis. Failure to thrive, hepatomegaly, and jaundice can also occur. Undiagnosed individuals may eventually develop liver and renal failure. Elimination of dietary fructose is the mainstay of treatment and results in symptom improvement with a good long~term prognosis.

A 6-month-old boy is brought to the emergency department by his mother because of recent onset of vomiting, irritability, and jaundice. The infant was born at term and had been healthy until the onset of these symptoms. All of his vaccinations are up-to-date. He had been breast-fed exclusively until 1 week ago, when cereals and fruit juices were introduced into his diet. Further evaluation reveals hepatomegaly and abnormal liver function tests. Which of the following enzymes is most likely to be deficient in this patient? A. Galactose-1-phosphate uridyl transferee B. Aldolase B C. Fructokinase D. Galactokinasa E. Acid α-glucosidase

*The answer is E.* The patient described above most likely has hereditary fructose intolerance. Hereditary fructose intolerance is an autosomal recessive disorder that results from aldolase B deficiency, and typically presents around the age of breast milk weaning and introduction of foods containing fructose, with symptoms of hypoglycemia, jaundice, and vomiting. The patients symptoms, along with the presence of reducing substance in the urine (i.e„ fructose), makes hereditary fructose intolerance the likely diagnosis. Aldolase B normally catalyzes the conversion of fructose-1 phosphate to dihydroxyacetone-phosphate and glyceraldehyde-3-phosphate. Fructokinese, the enzyme upstream of aldolase B, uses 1 adenosine triphosphate (ATP) to catalyze the irreversible reaction of fructose to fructose-I phosphate. However, the fructose-1-phosphate in the liver is unable to escape because aldolase B is the only enzyme that uses this carbohydrate as a substrate. Fructose-1-phosphate accumulation leads to liver cirrhosis end jaundice. The buildoup of fructose-1-phosphate results in phosphate trapping thet depletes both phosphate and ATP stores, which in turn inhibits gluconeogenesis and glycogenolysis, and the regeneretion of ATP. These metabolic changes result in hypoglycemia, defined clinicelly as blood glucose < 70 mg/dL, which manifests clinically as tremors, palpitations, anxiety, diaphoresis, altered mental status, and lethargy. Depleting phosphate end ATP leads to increased AMP, which stimulates the activity of AMP deaminase. AMP deaminase catalyzes the conversion of AMP to IMP, which is ultimately converted to uric acid. Therefore, patients with aldolase B deficiency may present with hyperuricemia.

A 6-month-old boy is brought to the emergency room by his mother because of a 3-day history of vomiting and lethargy. The patient has no significant previous medical history and is meeting all of his developmental milestones. Three days ago his mother began to supplement breastfeeding with solid food, including apple puree. On physical exam, the patient appears diaphoretic. He also exhibits scleral icterus and a palpable liver border. The patient's blood work reveals a glucose level of 50 mg/dL, and urinalysis is positive for reducing substance The defective enzyme in this patient normally catalyzes the formation of which product? A. Galactose-1-phosphate B. Glucose-1-phosphate C. Fructose-1,6-phosphate D. Galactitol E. Glyceraldehyde-3-phosphate

*The answer is G.* Carbohydrates are classified as monosaccharide, disaccharides, and polysaccharides. Disaccharides and polysaccharides must be broken down to their monosaccharide components for energy production and utilization. Aldolase B metabolizes fructose-1-phosphate, a product of fructose metabolism, to dihydroxyacetone phosphate (DHAP) and glyceraldehyde, which can then enter the glycolytic pathway. Aldolase B deficiency can result in fructose-1-phosphate accumulation, this toxic metabolite depletes intracellular phosphate and inhibits the activation of hepatic phosphorylase and gluconeogenesis. The resulting condition, hereditary fructose intolerance (eg, fructosemia), is an autosomal recessive disorder. Because gluconeogenesis is impaired, fructosemia typical presents with life-threatening hypoglycemia. Consequences of hypoglycemia include lethargy, sweating, vomiting, and dehydration. These symptoms manifest after intake of fructose or sucrose, such as from formula or fruit. Strict abstinence from dietary fructose and sucrose can result in dramatic recovery. *Educational Objective:* Aldolase B deficiency, or hereditary fructose intolerance, leads to accumulation of the toxic metabolite fructose-1-phosphate. Patients have hypoglycemia and vomiting when fructose or sucrose is consumed. Treatment involves strict removal of both carbohydrates from the diet.

A 6-month-old fall-term boy is brought to the emergency department with lethargy and vomiting. He was born by uncomplicated spontaneous vaginal delivery and has been growing and developing normally. The patient was breastfed exclusively until 2 days ago when homemade pureed feed was added to his diet He has had no fever or diarrhea. His parents are healthy and he has had no sick contacts. Examination shows a pale, diaphoretic, and ill-appearing infant Serum glucose is 30 mg/dL. Diagnostic testing confirms aldolase B deficiency. Which of the following should be removed from this patient's diet? A. Amylose B. Cellulose C. Galactose D. Glucose E. Lactose F. Maltose G. Sucrose

*The answer is A.* Pompe disease (type 2 glycogen storage disease) is caused by a deficiency in the lysosomal enzyme a-1,4-glucosidase. The condition is characterized by the accumulation of glycogen in the liver, heart, and skeletal muscle, causing cardiomegaly and other systemic findings, such as weakness and hypotonia. Early death ensues due to cardiorespiratory failure

A 6-month-old girl is brought to her pediatrician because of respiratory and feeding difficulties. On examination the infant is weak and hypotonic; cardiac evaluation reveals left ventricular enlargement with outflow tract obstruction and early cardiac failure. What is the most likely etiology of the patient 's condition? A. α- 1,4 -Glucosidase deficiency B. Amylo-1,6-glucosidase deficiency C. Galactose-1-phosphate uridyltransferase deficiency D. Glucose-6-phosphatase deficiency E. Glycogen phosphorylase def iciency

*The answer is C.* Fructose-1-phosphate aldolase deficiency is an autosomal recessive disorder of fructose metabolism caused by deficient fructose-1-phosphate aldolase activity, resulting in accumulation of fructose-1-phosphate, The accumulated fructose-1-phosphate inhibits glycogenolysis and gluconeogenesis, causing severe hypoglycemia after ingestion of fructose. Prolonged fructose ingestion in infants leads ultimately to hepatic failure and death. This condition is known as fructose intolerance. Essential fructosuria involves a defect in fructokinase (choice B) and is a benign asymptomatic condition.

A 6-month-old girl is brought to the emergency department because of increased crying, sweating, and sharing chills, The mother states that until recently the child had been exclusively breast-fed but she was now trying to see whether the child might enjoy other feeds. The symptoms started soon after feeding the child applesauce and pureed pears. which at first she ate eagerly. Onset of symptoms began approximately 1 hour later, Based on the history and laboratory results, a tentative diagnosis is made. Which of the following enzymes is most likely deficient in this patient? A. Aldose reductase B. Fructokinase C. Fructose 1-phosphate aldolase D. Hexokinase E. Lactase

*The answer is D.* This patient has Tay-Sachs disease, an autosomal recessive disorder caused by the deficiency of hexosaminidase A, It leads to the accumulation of ganglioside GM2 in neurons, thus producing a degenerative neurologic disease. Children appear normal at birth, Out then begin to suffer from diminished responsiveness. blindness loss of neurologic function, and seizures. Ophthalmoscopic examination may reveal a cherry-red spot on the macula (actually a normal appearing macula with surrounding retinal pallor). Death usually occurs by 4 to 5 years of age, There is no therapy. The incidence is higher among Jews of Eastern European descent. Since the parents must be heterozygotes for the mutant hexosaminidase A allele. they would be expected to have diminished levels of the enzyme. No hepatomegaly or splenomegaly is observed, differentiating this disease from infantile Niemann-pick disease

A 6-month-old girl is brought to the physician by her parents because of seizures. The girl has also developed diminished responsiveness and progressive blindness over the past 4 months. Physical examination of the abdomen shows no abnormalities. Fundoscopic examination shows retinal pallor except in the macular area. Activity of which of the following enzymes is most likely decreased in both of the parents? A. Arylsulfatase A B. α-Galactosidase A C. Glucocerebrosidase D. Hexosaminidase A E. Spingomyelinase

*The answer is B.* Debranching enzyme deficiency (Cori disease) usually presents in infancy or childhood with both liver and muscle involvement. Symptoms include hypoglycemia, hepatomegely, and ketoacidosis. Muscle weakness and hypotonla help to extinguish the condition from other glycogen storage diseases with hepatic involvement (eg, von Gierke). Hepatic fibrosis is common, but fatty infiltration is not usually seen. A key distinguishing feature is cytosolic accumulation of glycogen with abnormally short outer chains (limit dextrins). During glycogenolysis, glycogen phosphorylase shortens glycogen chains by cleaving α-1,4-glycosidic linkages between glucose residues, liberating glucose-1-phosphate in the process. This occurs until 4 residues remain before a branch point (the limit dextrin). At this point, debranchlng enzyme performs 2 enzymatic functions: 1) glucosyltransferase cleaves the outer 3 residues of the 4 glucose residues left by glycogen phosphorylase and transfers them to a nearby branch, and 2) α-1 ,6-glucosidase removes the single remaining branch residue, producing free glucose and a linear glycogen chin that can be further shortened by glycogen phosphorylase. *Educational Objective:* Debranching enzyme deficiency (Con disease) leads to accumulation of glycogen with abnormally short outer chains (limit dextrins) due to the inability to degrade α-1,6-glycosidic branch points. Patients present with hypoglycemia, ketoacidosis, hepatomegaly, and muscle weakness and hypotonia.

A 6-month-old girt is brought to the office by her mother for a checkup appointment. The mother states, "My baby doesn't seem to be growing much despite feeding as often as my previous children. I am worried that something is wrong with her." Physical examination shows hepatomegaly and height and weight below the 10th percentile. Laboratory studies show hypoglycemia and ketoacidosis. A liver biopsy shows hepatic fibrosis without fat accumulation. Further analysis reveals abundant quantities of a multi-branched polysaccharide with abnormally short outer chains within the cytosol of the hepatocytes. Which of the following enzymes is most likely deficient in this patient? A Acid maltase B Debranching enzyme C. Glucose-6-phosphatase D. Glycogen phosphorylase E Pyruvate kinase

*The answer is D.* Gluconeogenesis is the process by which glucose is produced in the liver and kidney from small noncarbohydrate precursors such as alanine and lactate. Glucose-6-phosphatase is a key enzyme involved in the final step of this pathway, catalyzing the conversion of glucose-6-phosphate to glucose. A deficiency of this enzyme causes von Gierke disease (type 1 glycogen storage disease) which is characterized by an inability to produce glucose from glycogen. Affected patients, like the one described in the vignette, usually present in infancy with hypoglycemia, seizures, hepatomegaly, and lactic acidosis. Von Gierke disease typically manifests at a mean age of 6 months, when the infant's feeding schedule begins to be more spaced out. Symptoms are more likely to arise during the fasting state- as the enzyme deficiency does not allow for hydrolysis of glucose-6-phosphate, which prevents glucose from leaving hepatocytes and entering the bloodstream.

A 6-month-old infant is brought into the emergency department with seizures. She is the product of a normal, full-term pregnancy and delivery and has had no health problems in the past. Physical examination reveals hepatomegaly. Her heart rate is 180/min and respiratory rate is 75/min (normal respiratory rate for infants: 30- 60/min). A heel stick shows that the infant is hypoglycemic. What is the most likely cause of the patient's presentation? A. Fructokinase deficiency B. Galactokinase deficiency C. Galactose-1-phosphate uridyltransferase deficiency D. Glucose-6-phosphatase deficiency E. Lactase deficiency

*The answer is D.* The child has Von Gierke disease, glycogen storage disease type I, a lack of glucose-6-phosphatase. In such a disorder, glucose-6-phosphate, whether produced from glycogen degradation or gluconeogenesis, cannot be dephosphorylated for glucose export, and the liver cannot maintain blood glucose levels. The small amount of glucose which is exported (10% of the expected) is derived from the activity of debranching enzyme, which hydrolyzes an α-1,6-glucose linkage, which produces free glucose. The hepatomegaly arises due to excess glycogen in the liver (glucose-6-phosphate will activate glycogen synthase D), as does the increase in kidney size. A picture of a 25-month-old untreated child with this disorder is shown below. A lack of glycogen synthase would not lead to hepatomegaly, while a lack of branching enzyme leads to a different glycogen storage disease, with very different symptoms. A lack of debranching activity would not lead to hepatomegaly and would allow more glucose release than is observed through the normal action of glycogen phosphorylase. A defect in fructose-1,6-bisphosphatase would impair gluconeogenesis, but should not affect the ability of glycogen to be degraded to raise blood glucose levels.

A 6-month-old infant was brought to the pediatrician due to fussiness and a tender abdomen. The child seemed to do well until the time between feeding was increased to more than 3 h. The baby always seemed hungry and irritable if not fed frequently. Upon examination, hepatomegaly and enlarged kidneys were noted, and blood work showed fasting hypoglycemia. Subsequent laboratory analysis demonstrated that in response to a glucagon challenge, only about 10% of the normal amount of glucose was released into circulation, which significantly contributed to the fasting hypoglycemia. Which enzyme defect in the patient is the most likely? (A) Glycogen synthase (B) Branching enzyme (C) Debranching enzyme (D) Glucose-6-phosphatase (E) Fructose-1,6-bisphosphatase

*The answer is C.* You should associate Ashkenazic (Eastern European) Jews with two diseases: Tay-Sachs disease and Type I Gaucher disease. Both of these diseases are sphingolipidoses. Tay-Sachs disease is the more devastating of the two, and is characterized by progressive neurologic (including visual) deterioration beginning at about 6 months of age and leading to death by age 3 years. In contrast, Type I Gaucher disease is compatible with a normal life span and causes hepatosplenomegaly with CNS involvement. (The infantile Type II and the juvenile Type III forms cause more serious disease but are not seen with increased incidence in Ashkenazic Jews.) None of the other conditions listed occur with greater frequency in Ashkenazi Jews, with the exception of Niemann-pick disease type A.

A 7-month-old boy is brought to a physician by his parents because the boy has stopped smiling, and seems to have lost the ability to roll over and grasp at objects. A review of the family history shows that both of his parents are Ashkenazi Jews. The physician should explain to them that, because of their heritage, their son may have an increased risk of which of the following disorders? A. Albinism and galactosemia B. Cystic fibrosis and Lesch-Nyhan disease C. Gaucher disease and Tay-Sachs disease D. Krabbe disease and Niemann-pick disease E. Metachromatic Ieukodystrophy and phenylketonuria

*The answer is C.* The child has a form of glycogen storage disease known as type VII, Tarui disease, which is a lack of muscle phosphofructokinase 1 (PFK-1) activity. The lack of muscle PFK-1 means that glycolysis is impaired, so anaerobic activities are significantly curtailed in such individuals. Slow, aerobic activities, which can be powered by fatty acid oxidation, are normal in such children. Strenuous activity will lead to muscle damage and weakness due to this block in glycolysis. Glucose-6-phosphatase is only found in the liver (and to a small extent, the kidney), and a lack of such activity would lead to fasting hypoglycemia, but would not affect muscle glycolytic activity. A defect in liver PFK-1 activity would not affect muscle glycolysis. A defect in liver glycogen phosphorylase would also lead to fasting hypoglycemia, but would not alter the rate of muscle glycolysis, or lactate formation from that pathway.

A 7-year-old boy is brought to the pediatrician due to severe exercise intolerance. In gym class, the boy has trouble with anaerobic activities. Laboratory tests showed a lack of lactate production under such conditions. The boy was eventually found to have a mutation in which one of the following enzymes? (A) Liver glycogen phosphorylase (B) Liver PFK-1 (C) Muscle PFK-1 (D) Muscle glucose-6-phosphatase (E) Liver glucose-6-phosphatase

*The answer is B.* Raw eggs contain a potent binding partner to biotin, avidin, which, while bound to biotin, blocks biotin's participation in carboxylation reactions. This leads to reduced activity of pyruvate carboxylase, a necessary step in many gluconeogenic pathways, thereby leading to a reduced ability of the liver to properly maintain blood glucose levels. As oxaloacetate levels drop due to the need of oxaloacetate for gluconeogenesis, acetyl-CoA derived from fatty acid oxidation increases, leading to ketone body formation. Avidin does not affect NAD+ or FAD levels, nor does it interfere with coenzyme A or vitamin C.

A 7-year-old girl, who lives on a farm, started to have shaking and sweating episodes. Upon physical examination, she was found to be hypoglycemic under fasting conditions (fasting blood glucose was 50 mg/dL) and positive for ketones in her blood and urine. Her growth curve is normal. Further analyses showed no other metabolic abnormalities. Probing further into her history, in the absence of her parents, revealed that one of her chores was to collect eggs from the chicken coop every morning, and she had gotten into the habit of eating one or two raw eggs every morning. This had been going on for the past 6 weeks or so. A reasonable explanation for her laboratory results is which one of the following? (A) Reduced levels of electron acceptors in her system, leading to reduced glucose production (B) Reduced effectiveness of carboxylation reactions, leading to reduced glucose production (C) Reduced effectiveness of acyl activation, leading to reduced glucose production (D) Reduced effectiveness of protein hydroxylation, leading to reduced enzymatic activity and reduced glucose production (E) Reduced levels of electron donors in her system, leading to reduced glucose production

*The answer is B.* Marked riboflavin deficiency is rare in the United States, but can be seen in chronic alcoholics and the severely malnourished. Clinical manifestations of marked riboflavin deficiency include angular stomatitis, cheilitis, glossitis, seborrhea dermatitis, eye changes (kg, keratitis, corneal neovascularization), and anemia. The diagnosis is established with performance of the erythrocyte glutathione reductase assay or evaluation of the urinary riboflavin excretion. Metabolic modifications of riboflavin occur most frequently in the cells of the heart, liver, and kidney. Typically, riboflavin is first phosphorylated to become the coenzyme flavin mononucleotide (FMN). It can then either be integrated into a coenzyme-flavin complex or can be further phosphorylated into flavin adenine dinucleotide (FAD). FMN and FAD are required cofactors for flavoproteins, which are enzymes that participate in numerous reduction-oxidation reactions within the human body. In the course of these reactions, the FMN and FAD cofactors are transformed into their reduced, energy-carrying states (FMNH.. and FADH..) through the acceptance of electrons. The riboflavin-containing coenzymes are key constituents of the electron transport chain; FMN is a component of complex I, while FAD is a component of complex II. FAD is an electron carrier in the tricarboxylic acid cycle (TCA) and serves as a cofactor for succinate dehydrogenase, which is an enzyme that mediates the conversion of succinate into fumerate.

A 76-year-old Caucasian female is evaluated for painful lesions on her lips and at the corners of her mouth. She is mildly demented and lives alone. Her urinary riboflavin excretion is very low. Activity of which of the following enzymes is most likely decreased in this patient? (A) Isocitrate dehydrogenate (B) Succinate dehydroganase (C) Succinate thiokinase (D) Malate dehydrogenate (E) Fumarase (F) Glucose-B-phosphate dehydrogenase (G) HMG-CoA reductase

*The answer is B.* This patient has diabetes mellitus and presents with hyperosmolar hyperglycemia, a metabolic derangement often precipitated by infection (pneumonia in this case) and characterized by dehydration hyperglycemia. and hyperosmolarity without significant ketoacidosis. ` His cataracts likely formed from oversaturation of the polyol pathway secondary to long-term hyperglycemia. Aldose reductase converts glucose into sorbitol during the first step in the polyol pathway of glucose metabolism. Sorbitol cannot readily cross cell membranes and is therefore trapped inside the cells within which it is famed. If the enzyme sorbitol dehydrogenate (sometimes referred to as polyol dehydrogenate) is also present in the cell, it can convert sorbitol into fructose. This pathway, known as the poly pathway, is especially active in the seminal vesicles, as spent use fructose as their primary energy source. Other tissues, such as the retina, renal papilla, and Schwann eels, have much less sorbitol dehydrogenase activity. The human lens does contain significant levels of sorbitol dehydrogenase, (see references), which allows for the production of fructose. However. this enzyme has a significantly lower Vmax in the sorbitol-to-fructose direction than in the reverse direction. When glucose levels are low, the limited forward activity of this enzyme is sufficient to convert enough sorbitol into fructose to prevent sorbitol accumulation. In contrast. states of long-standing hyperglycemia lead to the production of an excessive amount of sorbitol that is trapped in the cells. This increases the osmotic pressure and facilitates the influx of water into the lens cells. leading to the development of hydropic lens fibers that degenerate. Eventually, this results in lens opacification and cataract formation. In addition to osmotic cell injury, oxidative stress resulting from the depletion of NADPH contributes to cataract formation and other diabetic complications such as neuropathy and retinopathy.

A 76-year-old man is brought to the emergency department due to excessive fatigue and altered mental status. Hs was recently discharged home from the emergency department with a diagnosis of pneumonia. The patient has a long history of diabetes mellitus. His blood pressure is 100/60 mm Hg and pulse is 100/min. Physical examination shows a dry mouth, cracked lips, and severe cataract formation. Laboratory tests show a serum glucose level of 750 mg/dL and a normal serum ketone level. The pathophysiology of this patient's cataract formation involves certain enzymes within the lens. An enzyme called aldose reductase produces sorbitol, a substance that cannot readily exit the lens cells. Which of the following is the most likely end product of sorbitol metabolism in the lens of healthy individuals? A. Glucose B. Fructose C. Galactase D. Galactitol E. Xylusoe

*The answer is B.* Biotin acts as a CO2 carrier on the surface of the carboxylase enzyme. which encompasses the enzymatic subtypes acetyl-CoA carboxylase (ACC), pyruvate carboxylase (PC), propionyl carboxylase (PCC), and beta-methylcrotonyl CoA carboxylase (MCC). All of these enzymes play roles in carbohydrate and lipid metabolism. In the tissues responsible for gluconeogenesis, for instance, pyruvate carboxylase (and therefore biotin) is necessary for the conversion of pyruvate to oxaloacetate. In biotin-deficient individuals, the level of pyruvate rises and the pyruvate is converted to lactic acid instead. Metabolic acidosis results. Another example of metabolic derangement secondary to biotin deficiency is the need for propionyl CoA carboxylase (and therefore biotin) to synthesize succinyl CoA from amino acids such as valine. In biotin-deficient individuals, the propionyl-CoA builds up and is instead metabolized into a surplus of odd-chain fatty acids. Deficiencies in this cofactor are rare, but can occur secondary to poor diet, excessive now egg white consumption (due to the high levels of biotin-binding avidin in egg whites), and congenital disorders of biotin metabolism.

A 78-year-old female suffers from nausea, anorexia, fatigue, and skin rash on her legs. She is mildly demented and is believed to have severe malnutrition. Laboratory findings include an organic aciduria. You suspect that biotin deficiency may contribute to this patient's symptoms, Which of the following conversions is impaired in those with biotin deficiency? A. Pyruvate to alanine B. Pyruvate to oxaloacetate C. Glucose to ribose-5-phosphate D. Pyruvate to acetyl-CoA E. Succinate to oxaloacetate

*The answer is A.* One of the world's most common enzyme deficiencies is glucose-6-phosphate-dehydrogenase deficiency. This deficiency in erythrocytes is particularly prevalent among African and Mediterranean males. A deficiency in glucose-6-phosphate dehydrogenase blocks the pentose phosphate pathway and NADPH production. Without NADPH to maintain glutathione in its reduced form, erythrocytes have no protection from oxidizing agents. This X-linked recessive deficiency is often diagnosed when patients develop hemolytic anemia after receiving oxidizing drugs such as pamaquine or after eating oxidizing substances such as fava beans.

A Nigerian medical student studying in the United States develops hemolytic anemia after taking the oxidizing antimalarial drug pamaquine. This severe reaction is most likely due to a. Glucose-6-phosphate dehydrogenase deficiency b. Concomitant scurvy c. Vitamin C deficiency d. Diabetes e. Glycogen phosphorylase deficiency

*The answer is F.* The enzyme glyceraldehyde 3-phosphate dehydrogenase (GAPDH) has a cysteine residue, which has a sulfhydryl group, in its active site. Thus, if this is inhibited, glyceraldehyde 3-phosphate will accumulate because it will not not be able to catalyze the reaction to form 1,3-bisphosphoglycerate.

A biochemist has placed in a test tube all of the enzymatic machinery and cofactors necessary to convert glucose to pyruvate. The scientist than places within the test tube a sulfhydryl reagent which forms covalent bonds with free sulfhydryl groups in proteins. After adding the inhibitor, glucose was added to the test tube. Which ONE metabolite would initially accumulate within the test tube under these conditions? (A) 1,3 bisphosphoglycerate (B) Citrate (C) Fructose 1,6 bisphosphate (D) Fructose-6-phosphate (E) Glucose-6-phosphate (F) Glyceraldehyde-3-phosphate (G) Glycogen (H) Pyruvate

*The answer is A.* Cataracts + liver disease in a milk-fed infant = classic galactosemia.

A breast-fed infant begins to vomit frequently and lose weight. Several days later she is jaundiced, her liver is enlarged, and cataracts are noticed in her lenses. These symptoms are most likely caused by a deficiency of A. galactose 1-P-uridyltransferase B. lactase C. glucose-6-phosphatase D. galactokinase E. aldolase B

*The answer is B.* In the presence of low blood glucose, epinephrine or norepinephrine interacts with specific receptors to stimulate adenylate cyclase production of cyclic AMP. Cyclic AMP activates protein kinase, which catalyzes phosphorylation and activation of phosphorylase kinase. Activated phosphorylase kinase activates glycogen phosphorylase, which catalyzes the breakdown of glycogen. Phosphorylase kinase can be activated in two ways. Phosphorylation leads to complete activation of phosphorylase kinase. Alternatively, in muscle, the transient increases in levels of Ca++ associated with contraction lead to a partial activation of phosphorylase kinase. Ca++ binds to calmodulin, which is a subunit of phosphorylase kinase. Calmodulin regulates many enzymes in mammalian cells through Ca++ binding.

A man goes on a hunger strike and confines himself to a liquid diet with minimal calories. Which of the following would occur after 4 to 5 h? a. Decreased cyclic AMP and increased liver glycogen synthesis b. Increased cyclic AMP and increased liver glycogenolysis c. Decreased epinephrine levels and increased liver glycogenolysis d. Increased Ca++ in muscle and decreased glycogenolysis e. Decreased Ca++ in muscle and decreased glycogenolysis

*The answer is A.* Glycosides are formed by condensation of the aldehyde or ketone group of a carbohydrate with a hydroxyl group of another compound. Other linked groups (aglycones) include steroids with hydroxyl groups (e.g., cardiac glycosides such as digitalis or ouabain) or other chemicals (e.g., antibiotics such as steptomycin). Sucrose (α-D-glucose-β-1 → 2-D-fructose), maltose (α-D-glucose-α-1 → 4-D-glucose), and lactose (α-D-galactose-β-1 → 4-D-glucose) are important disaccharides. Fructose is among several carbohydrate groups known as ketoses because it possesses a ketone group. The ketone group is at carbon 2 in fructose, and its alcohol group at carbon 1 (also at carbon 6) allows ketal formation to produce pyranose and furanose rings as with glucose. Most of the fructose found in the diet of North Americans is derived from the disaccharide sucrose (common table sugar). Sucrose is cleaved into equimolar amounts of glucose and fructose in the small intestine by the action of the pancreatic enzyme sucrase. Deficiency of sucrase can also cause chronic diarrhea. Hereditary fructose intolerance is caused by deficiency of the liver enzyme aldolase B, which hydrolyzes fructose-1-phosphate.

A child develops chronic diarrhea and liver inflammation in early infancy when the mother begins using formula that includes corn syrup. Evaluation of the child demonstrates sensitivity to fructose in the diet. Which of the following glycosides contains fructose and therefore should be avoided when feeding or treating this infant? a. Sucrose b. Oaubain c. Lactose d. Maltose e. Streptomycin

*The answer is A.* Cyanide blocks respiration by displacing oxygen from hemoglobin. Oxidative phosphorylation in the mitochondria cannot proceed because cyanide cannot oxidize (remove electrons) from reduced cofactors like NADH. The citric acid cycle is the major pathway for generating ATP and reducing equivalents (NADH, H+) from catabolism of carbohydrates, amino acids, and lipids. Inability to regenerate NAD+ from NADH through mitochondrial oxidative phosphorylation depletes the cell of NAD+ and inhibits the citric acid cycle. Failure to generate ATP by oxidative phosphorylation using NADH from the citric acid cycle depletes the cell of energy and leads to cell and tissue death (organ failure). Enzymes (citrate synthase, aconitase) and intermediates of the citric acid cycle (citrate, acetyl coenzyme A) need only be present in trace amounts because they are not consumed.

A child has ingested cyanide from her parents' garage and is rushed to the emergency room. Which of the following components of the citric acid cycle will be depleted first in this child? a. NAD+ cofactor b. Citrate synthase c. Aconitase d. Citrate production e. Acetyl coenzyme A (CoA) production

*The answer is C.* The patient is experiencing the symptoms of vitamin B1 (thiamine) deficiency. Ethanol blocks thiamine absorption from the gut, so in the United States, one will usually only see a B1 deficiency in chronic alcoholics. One assay for B1 deficiency is to measure transketolase activity (which requires B1 as an essential cofactor) in the presence and absence of added B1. If the activity level increases when B1is added, a vitamin deficiency is assumed. None of the other enzymes listed (transaldolase, aldolase, β-ketothiolase, and acetylcholine synthase) require B1 as a cofactor, and, thus, could not be used as a measure of B1 levels. A reaction catalyzed by transketolase is shown below (note the breakage of a carbon-carbon bond, and then the synthesis of a carbon-carbon bond to generate the product of the reaction).

A chronic alcoholic presents to the emergency department with nystagmus, peripheral edema, pulmonary edema, ataxia, and mental confusion. The physician orders a test to determine if there is a vitamin deficiency. An enzyme used for such a test can be which of the following? (A) Transaldolase (B) Aldolase (C) Transketolase (D) β-ketothiolase (E) Acetylcholine synthase

*The answer is C.* The alcoholic has become deficient in vitamin B1, thiamine, which is converted to thiamine pyrophosphate for use as a coenzyme. One of the symptoms of B1 deficiency is neurological, due to insufficient energy generation within the nervous system. B1 is required for a small number of enzymes, including transketolase, pyruvate dehydrogenase, and α-ketoglutarate dehydrogenase. By reducing the activity of the latter two enzymes, glucose oxidation to generate energy is impaired, and the nervous system suffers because of it.

A chronic alcoholic, while out on a binge, became very confused and forgetful. The police found the man and brought him to the emergency department. Upon examination, he displayed nystagmus and ataxia. Which enzyme is displaying reduced activity in his brain under these conditions? (A) Glyceraldehyde-3-phosphate dehydrogenase (B) Isocitrate dehydrogenase (C) α-ketoglutarate dehydrogenase (D) Succinate dehydrogenase (E) Malate dehydrogenase

*The answer is D.* The following steps are required for the complete oxidation of citrate to carbon dioxide and water. First, citrate goes to isocitrate, which goes to α-ketoglutarate (this last step generates carbon dioxide and NADH, which can give rise to 2.5 ATP). The α ketoglutarate is further oxidized to succinyl-CoA, plus carbon dioxide and NADH (this is the second carbon released as CO2, and another 2.5 ATP). Succinyl-CoA is converted to succinate, generating a GTP (at this point, five high-energy bonds have been created, plus two carbons lost as carbon dioxide). Succinate goes to fumarate, with the generation of FADH2 (another 1.5 ATP), fumarate is converted to malate, and malate leaves the mitochondria (via the malate/aspartate shuttle) for further reactions. Once in the cytoplasm, the malate is oxidized to oxaloacetate, generating NADH (another 2.5 ATP if the malate/ aspartate shuttle is used). At this point, citrate has been converted to cytoplasmic oxaloacetate, with the generation of ten high-energy bonds and the loss of two carbons as carbon dioxide. The oxaloacetate is then converted to phosphoenolpyruvate and carbon dioxide at the expense of a high-energy bond (GTP, the phosphoenolpyruvate carboxykinase reaction). The high-energy bond is recovered in the next step, however, as PEP is converted to pyruvate, generating an ATP. Thus, at this point in our conversion, citrate has gone to pyruvate, plus three CO2, with a net yield of ten ATP (or high-energy bonds). The pyruvate reenters the mitochondria and is oxidized to acetyl-CoA and carbon dioxide, also generating NADH (another 2.5 ATP). When this acetyl-CoA goes around the TCA cycle, two carbon dioxide molecules are produced, along with another ten high-energy bonds. The net total is therefore six carbon dioxide molecules and 22.5 high energy bonds for the complete oxidation of citrate.

A crazed friend of yours has gone on an orange juice, fish, and vitamin pill diet. He tells you that the citric acid, since it is a component of the TCA cycle, is always recycled and does not count toward his caloric total each day. You disagree, and inform him that citrate can, in addition to having its carbons stored as glycogen or fat for later use, produce energy for his daily metabolic needs. The energy yield for the complete oxidation of citrate to six carbon dioxides and water is which of the following? (A) 15.0 moles of ATP per mole of citrate (B) 17.5 moles of ATP per mole of citrate (C) 20.0 moles of ATP per mole of citrate (D) 22.5 moles of ATP per mole of citrate (E) 25.0 moles of ATP per mole of citrate

*The answer is E and F.* When blood glucose levels are low, glycolysis will decrease, which will directly or indirectly affect phosphofructokinase-II and pyruvate kinase. Phosphofructokinase-II catalyzes the reaction of fructose 6-phosphate to fructose 2,6-bisphosphate and an inhibition of this enzyme will lead to a decreased level of f-2,6-BP. This will stimulate fructose 1,6-bisphosphatase to catalyze the reaction from fructose 1,6-bisphosphate to fructose 6-phosphate, which will eventually decreases glycolysis. Pyruvate kinase will also be inhibited because it leads to the formation of pyruvate. If the levels of glucose are too low, then pyruvate kinase activity will decrease, therefore decreasing glycolysis because low levels of glucose indicate that glycolysis is not the main priority, but gluconeogenesis is.

A decrease in blood glucose levels will, in the liver, lead to the inhibition, either directly or indirectly, of which of the following enzymatic activities? This question may have more than one correct answer; be sure to indicate all correct answers on the answer sheet. (A) Aldolase (B) Cyclic-AMP dependent protein kinase (C) Enolase (D) Glyceraldehyde-3-phosphate dehydrogenase (E) Phosphofructokinase-II (F) Pyruvate kinase

*The answer is A.* Lactic acidosis can result from a defect in an enzyme that metabolizes pyruvate (primarily pyruvate dehydrogenase and pyruvate carboxylase). The pyruvate dehydrogenase complex consists of three major catalytic subunits, designated E1, E2, and E3. The E1 subunit is the one that binds thiamine pyrophosphate and catalyzes the decarboxylation of pyruvate. The gene for the E1 subunit is on the X chromosome, so defects in this subunit are inherited as X-linked diseases, which primarily affects males. Since this is the second male child to have these symptoms, it is likely that the mother is a carrier for this disease. The pattern of inheritance distinguishes this diagnosis from that of an E2 or E3 deficiency. In addition, an E3 deficiency would affect more than pyruvate metabolism, as this subunit is shared with other enzymes that catalyze oxidative decarboxylation reactions, and other metabolites would also be accumulating. Defects in citrate synthase and malate dehydrogenase would not lead to severe lactic acidosis and would not be male-specific disorders.

A family that had previously had a newborn boy die of a metabolic disease has just given birth to another boy, small for gestational age and with low Apgar scores. The child displayed spasms a few hours after birth. Blood analysis indicated extremely high levels of lactic acid. Analysis of cerebrospinal fl uid showed elevated lactate and pyruvate. Hyperalaninemia was also observed. The child died within 5 days of birth. The biochemical defect in this child is most likely which of the following? (A) The E1 subunit of pyruvate dehydrogenase (B) The E2 subunit of pyruvate dehydrogenase (C) The E3 subunit of pyruvate dehydrogenase (D) Citrate synthase (E) Malate dehydrogenase

*The answer B.*UDP-hexose 4-epimerase converts UDP-glucose to UDP-galactose, thus providing the appropriate form of galactose for lactose synthesis. UDP-galactose, not free galactose, is the source of the galactose portion of lactose. Galactose is not converted to galactose 1-phosphate by hexokinase. Galactosemia is the result of a deficiency in GALT. Isomerization of fructose to galactose does not occur in the human body.

A female with classic galactosemia due to GALT deficiency is able to produce lactose in breast milk because: A. free (nonphosphorylated) galactose is the acceptor of glucose transferred by lactose synthase in the synthesis of lactose. B. galactose can be produced from a glucose metabolite by epimerization. C. hexokinase can efficiently phosphorylate dietary galactose to galactose 1-phosphate. D. the enzyme deficient in galactosemia is activated by a hormone produced in the mammary gland. E. galactose can be produced from fructose by isomerization.

*The answer is A.* Fructokinase deficiency has no clinical manifestations. It generally presents as an accidental finding of a reducing substance in the urine that is not glucose. It is autosomal recessive and no treatment is required. Fructokinase catalyzes the first step of dietary fructose metabolism by converting fructose to fructose-1 -phosphate with a deficiency of the enzyme fructose levels increase in the blood, but almost all of it is excreted in the urine because there is no renal threshold for fructose. Fructose-1,6-bisphosphate aldolase deficiency is a severe condition of infants that occurs with the ingestion of fructose-containing foods, The enzyme deficiency affects the liver, kidney, and intestine, Fructose-1,6 biphosphate aldolase (choice B) catalyzes the hydrolysis of fructose-1,6-bisphosphate into dihydroxyacetone phosphate and glyceraldehyde phosphate. It also hydrolyzes fructose-1-phosphate. With this deficiency, there is a rapid accumulation of fructose-1 -phosphate and severe toxicity when exposed to fructose. Galactokinase (choice C) catalyzes the phosphorylation of galactose. with deficiency, there is an accumulation of galactose in peripheral blood. Several tissues. including the lens of the eye have an enzyme (aldose reductase) that catalyzes the conversion of galactose to galactitol. As galactitol accumulates in the lens, it causes osmotic damage. Cataracts are usually the sole manifestation Galactose-1-phosphate uridyltransferase [choice D) deficiency results in classic galactosemia; without prompt diagnosis and treatment, the infant will die. Without the enzyme, the patient is unable to metabolize galactose-1-phosphate and it accumulates in tissues, causing injury to the kidneys, liver, and brain. The process may begin prenatally due to transplacental transport of galactose, After birth, with the initiation of feeding milk these infants have jaundice, have failure to thrive, and also develop cataracts Glucose-6-phosphatase deficiency (choice E) manifests as a glycogen storage disease (Von Gierire disease). With the deficiency, there is inadequate hepatic conversion of glucose-6-phosphate to glucose and the patient is susceptible to fasting hypoglycemia Children typically present at 3-4 months of age with hypoglycemic seizures. In addition to severe hypoglycemia, there is growth retardation, increased serum lactate, increased cholesterol, increased triglycerides, and hepatosplenomegaly and kidney enlargement These last two findings result in a protuberant abdomen, Liver adenomas develop in many patients and may undergo malignant transformation.

A healthy 2-year-old girl is brought to the physician for a well-child examination. Physical examination shows no abnormalities Urinalysis is positive for a reducing substance that, upon further testing, is not glucose. She has reached all appropriate developmental milestones. She does not take any medications and has no history of any major medical illness. The urinalysis finding most likely represents a deficiency of which of the following enzymes? A. Fructokinase B. Fructose 1,6-bisphosphate aldolase C. Galactokinase D. Galactose 1-phosphate uridyltransferase E. Glucose-6-phosphatase

*The answer is E.* The 2 major processes that maintain plasma glucose between meals are glycogenolysis and gluconeogenesis. Glycogenolysis is the primary source of glucose for the first 12-18 hours of fasting. Once hepatic glycogen stores become depleted, gluconeogenesis becomes the major process used by the body to keep blood glucose levels within the normal range. During gluconeogenesis, glucose is famed from lactate, glycerol, and glycogenic amino acids. This process uses many of the enzymes involved in glycolysis. However, hexokinase, phosphofructokinase, and pyruvate kinase are unidirectional and must be bypassed by distinct gluconeogenic enzymes. The first committed step of gluconeogenesis is the biotin-dependent carboxyiation of pyruvate to oxaloacetate by mitochondrial pyruvate carboxylase. Oxaloacetate is subsequently converted to malate by malate dehydrogenase to facilitate exit from the mitochondria, and then is converted back to oxaloacetate by cytosolic malate dehydrogenase (malate shuttle). In the cytosol, phosphoenolpyruvate carboxykinase (PEPCK) converts oxeloacetate to phosphoenolpyruvate. Therefore. pyruvate carboxylase and PEPCK work together to bypass pyruvate kinase. The 2 other unique gluconeogenic enzymes are fructose 1,6-bisphosphatase (bypasses phosphofructokinase) and glucose-6-phosphatase (bypasses hexokinase). (Choice A) Conversion of acetoacetyl-CoA to 3-hydroxy-3 methylglutaryl-CoA occurs during the synthesis of cholesterol and ketone bodies. Ketone body synthesis is increased in starvation situations, however, ketone bodies cannot be used to synthesize glucose. (Choice B) Palmitic acid is the first fatty acid produced from acetyl CoA during lipogenesis in the fed state. However, during prolonged fasting, lipolysis predominates and leads to the generation of glycerol and fatty acids. (Choice C) Conversion of fructose 6-phosphate to fructose 1,6 -bisphosphate occurs during glycolysis and is catalyzed by phosphofructokinase. During starvation, glycolysis is minimized and gluconeogenesis predominates. (Choice D) The first step of glycogenolysis is breakage of 1-4 glycosidic linkage to for glucose-1-phosphate. After 24 hours of fasting, maintenance of blood glucose levels is achieved mostly through gluconeogenesis, not by glycogenolysis. *Educational Objective:* After 12-18 hours of fasting, gluconeogenesis becomes the principal source of blood glucose. Gluconeogenesis uses many glycolytic enzymes. but hexokinase, phosphofructokinase, and pyruvate kinase need to be bypassed as they are unidirectional. The initial steps of gluconeogenesis involve the conversion of pyruvate to oxaloacetate and oxaloacetate to phosphoenolpyruvate by pyrwate carboxylase and phosphoenolpyruvate carboxykinase. respectively.

A healthy 34-year-old coal mine worker is trapped underground following partial collapse of an access shaft. Rescue efforts are directed toward clearing the obstructed tunnel, but it takes 2 days to reach him. While being taken to the surface, the miner tells rescuers that he feels dizzy and weak. He had an emergency supply of water but has not eaten anything for over 30 hours. Fingerstick blood glucose concentration is 78 mgJdL Which of the following biochemical reactions is most likely responsible for maintaining this patient's current blood glucose levels? A. Acetoacetyl CoA → 3-hydroxy-3-methylglutaryl-CoA B. Acetyl CoA → palmitic acid C. Fructose 6-phosphate → fructose 1,6-bisphosphate D. Glycogen → glucose-1-phosphate E. Oxaloacetate → phosphoenolpyruvate

*The answer is D.* This activity of the debranching enzyme removes 1 ,6-linked glucose residues from the branch points during glycogenolysis.

A liver biopsy is done on a child with hepatomegaly and mild fasting hypoglycemia. Hepatocytes show accumulation of glycogen granules with single glucose residues remaining at the branch points near the periphery of the granule. The most likely genetic defect is in the gene encoding a(n): (A) α-1,4 phosphorylase (B) α-1,4: α-1,4 transferase (C) Phosphoglucomutase (D) α-1,6 glucosidase (E) lysosomal α-1,4 glucosidase

*The answer is B.* Lactose in breast milk and infant formula is converted by intestinal lactase to glucose and galactose that are efficiently absorbed. In galactosemia, deficiency of galactose-1-phosphate uridyl transferase prevents the conversion of galactose into glucose-6-phosphate by the liver or erythrocytes. Most other organs do not metabolize galactose. The severe symptoms of galactosemia are caused by the reduction of galactose to galactitol (dulcitol) in the presence of the enzyme aldose reductase. High levels of galactitol cause cataracts, the accumulation of galactose-1-phosphate contributes to liver disease, and the accumulation of galactose metabolites in urine can be measured as reducing substances by the Clinitest method. Any carbohydrate, including glucose, with a C1 aldehyde registers as a reducing substance by Clinitest, so a Dextrostix (glucose only) test is often performed as a control. In normal children, galactose is first phosphorylated by ATP to produce galactose-1-phosphate in the presence of galactokinase. Next, galactose-1-phosphate uridyl transferase transfers UDP from UDP-glucose to form UDP-galactose and glucose-1-phosphate. Under the action of UDP-galactose-4-epimerase, UDP-galactose is epimerized to UDP-glucose. Finally, glucose-1-phosphate is isomerized to glucose-6-phosphate by phosphoglucomutase. Infants with suspected galactosemia must be withdrawn from breastfeeding or lactose formulas and placed on non-lactose formulas such as Isomil.

A newborn begins vomiting after feeding, becomes severely jaundiced, and has liver disease. Treatment for possible sepsis is initiated, and the urine is found to have reducing substances. A blood screen for galactosemia is positive, and lactose-containing substances are removed from the diet. Lactose is toxic in this case because (A) Excess glucose accumulates in the blood (B) Galactose is converted to the toxic substance galactitol (dulcitol) (C) Galactose competes for glucose during hepatic glycogen synthesis (D) Galactose is itself toxic in even small amounts (E) Glucose metabolism is shut down by excess galactose

*The answer is C. The ratio of glucose-1-phosphate (GAP) to free glucose reflects the number of glucose residues (released as G1P} between branch points (free glucose released) in the glycogen. In this patient. the G1P-to-free glucose ratio is abnormally high indicating that the glycogen in this infant had fewer branches than normal, suggesting a problem with branching enzyme. (α-1,4 → α-1,6) glucan transferase.

A newborn delivered at 38 weeks gestation is brought to the physician because of jaundice and anorexia. A sample of the patient's glycogen is incubated with inorganic phosphate, normal glycogen phosphorylase. and debranching enzyme. The ratio of glucose-1-phosphate (G1P) to glucose forted in the reaction is abnormally high, Which of the following is most likely deficient in this patient? A. α-1,6-glucosidase B. (α-1,4 → α-1,4) glucan transferase C. (α-1,4 → α-1,6) glucan transferase D. Glycogen phosphorylase E. Glycogen phosphorylase kinase

*The answer is A.* A pyruvate carboxylase deficiency will impair gluconeogenesis from lactate and pyruvate, thereby leading to fasting hypoglycemia more easily than a pyruvate dehydrogenase deficiency (which will primarily affect the ability to generate energy from carbohydrates). Alanine amino transferase activity in the blood is a measure of liver damage, which would not distinguish between the two possibilities. Free fatty acid levels would be the same under both conditions, during fasting conditions, as would insulin and glucagon levels.

A newborn displays lethargy and crying episodes. Blood analysis indicates lactic acidosis and hyperalaninemia. In order to distinguish between a pyruvate dehydrogenase complex deficiency and a pyruvate carboxylase deficiency, one can measure which of the following in the blood? (A) Fasting blood glucose (B) Alanine aminotransferase activity (C) Free fatty acids levels when fasting (D) Insulin levels when fasting (E) Glucagon levels when fasting

*The answer is C.* This infant may have galactosemia, a deficiency of galactose-1-phosphate uridyl transferase (GALT). Galactose from lactose in breast milk or infant formula is phosphorylated by galactokinase, activated to uridine diphosphogalactose (UDP-galactose) by GALT, and converted to UDP-glucose by UDP-galactose epimerase. The elevation of galactose metabolites is thought to cause liver toxicity, and their urinary excretion produces reducing substances. Infants with the signs and symptoms listed are placed on lactose-free formulas until enzyme testing is complete. Deficiencies of epimerase or kinase can cause mild forms of galactosemia.

A newborn infant presents with poor feeding, vomiting, jaundice, and an enlarged liver. The urine tests positive for reducing substances, indicating the presence of sugars with aldehyde groups. Which of the following processes is most likely to be abnormal? (A) Conversion of glucose to galactose (B) Conversion of lactose to galactose (C) Conversion of activated galactose to activated glucose (D) Excretion of glucose by the kidney (E) Excretion of galactose by the kidney

*The answer is E.*For monosaccharides such as glucose, galactose, and fructose to be absorbed after ingestion, they need to first pass through the apical side of the gastrointestinal (GI) epithelium and then again through the basolateral side of the epithelium into the blood, as illustrated in the diagram. The apical side of the GI epithelium has numerous microvilli with numerous dedicated transporters. Both glucose and galactose traverse the apical membrane by the action of the transporter sodium-dependent glucose transporter 1 (SGLTl). SGLTl is a symporter, in that it simultaneously transports sodium and either monosaccharide into the cell. It is able to draw these monosaccharides into the epithelial cells along with sodium because of the sodium gradient created by the sodium-potassium adenosine triphosphatase (ATPase) at the basolateral membrane. Fructose, on the other hand, traverses the apical epithelial membrane via a sodium-independent monosaccharide transporter (GLUT- S). After glucose, galactose, and fructose enter the epithelial cells of the small intestine, they all traverse the basolateral membrane into the bloodstream via another transporter, GLUT- 2. Therefore, a medication that would inhibit the absorption of glucose and galactose, but not fructose, would have to work by inhibiting the absorption of glucose and galactose at the apical membrane. Inhibition of the SGLTl symporter could be achieved by impairing the sodium gradient, established by the basolateral Na+;K+ ATPase, which helps drive these monosaccharides into the cel l.

A novel drug is developed that specifically acts only on the intestinal epithelium. This new drug is found to prevent the absorption of glucose and galactose from the lumen, but does not affect the absorption of fructose. Based on the new drug's effects, which of the following is the most likely mechanism of action? A. It targets insulin regulation of glucose transporters B. It targets the GLUT-2 transporter C. It targets the GLUT-4 transporter D. It targets the GLUT-5 transporter E. It targets the sodium-potassium pump within intestinal cellls

*The answer is D.* The hyperglycemia in an untreated diabetic creates osmotic diuresis, which means that excessive water is lost through urination. This can lead to a contraction of blood volume, leading to low blood pressure and a rapid heart beat. It also leads to dehydration. The rapid respirations result from acidosis-induced stimulation of the respiratory center of the brain, in order to reduce the amount of acid in the blood. Ketone bodies have accumulated, leading to diabetic ketoacidosis (thus, B is incorrect). A patient in a hypoglycemic coma (which can be caused by excessive insulin administration) does not exhibit dehydration, low blood pressure, or rapid respirations; in fact, the patient will sweat profusely as a result of epinephrine release (thus, C and E are incorrect). Answer A is incorrect because lack of a pancreas would be fatal.

A patient arrives at the hospital in an ambulance. He or she is currently in a coma. Before lapsing into the coma, his or her symptoms included vomiting, dehydration, low blood pressure, and a rapid heartbeat. He or she also had relatively rapid respirations, resulting in more carbon dioxide being exhaled. These symptoms are consistent with which of the following conditions? (A) The patient lacks a pancreas. (B) Ketoalkalosis (C) Hypoglycemic coma (D) Diabetic ketoacidosis (E) Insulin shock in a patient with diabetes

*The answer is A.* In this patient, starch will be digested by salivary and pancreatic α-amylases to small oligosaccharides and maltose, but a lower than normal amount of glucose will be produced because of the deficiency of the brush border disaccharidases, which have maltase, isomaltase, sucrase, and lactase activity. Sucrose and lactose will not be cleaved. There will be more maltose, sucrose, and lactose in the stool and less monosaccharides in the blood and tissues. Insulin levels will be lower than normal, due to the reduced levels of glucose entering the blood. Muscle glycogen will not increase since there is less glucose in the circulation, and insulin, which is required for glucose entry into the muscle, may not be secreted under these conditions.

A patient has a genetic defect that causes intestinal epithelial cells to produce disaccharidases of much lower activity than normal. Compared with a normal person, after eating a bowl of milk and oatmeal sweetened with table sugar, this patient will have higher levels of which one of the following? Choose the one best answer. (A) Maltose, sucrose, and lactose in the stool (B) Starch in the stool (C) Galactose and fructose in the blood (D) Glycogen in the muscles (E) Insulin in the blood

*The answer is A.* The patient is experiencing the symptoms of familial ALS. A mutation in superoxide dismutase 1 (SOD1) in humans has been linked to the development of familial ALS through an unknown mechanism. Familial ALS only constitutes between 5% and 10% of all ALS cases diagnosed. The disease process, when SOD1 is mutated, is not linked to a loss of enzymatic activity, although the SOD1 may have been mutated such that it will produce other radical species and is no longer specific for superoxide. A second model proposes a misfolding problem similar to prion disease.

A patient has an insidious and steadily progressing neurologic disorder that, after several years, results in wasting and paralysis of the muscles of the limbs and trunk, loss of ability to speak, and swallowing difficulties. His paternal uncle had the same disease. A mutation in which enzyme may lead to these symptoms? (A) Superoxide dismutase (B) Catalase (C) Myeloperoxidase (D) NO synthase (E) Tyrosine hydroxylase

*The answer is D.* If, after fasting, the branches were shorter than normal, glycogen phosphorylase must be functional and capable of being activated by glucagon (thus, A and B are incorrect). The branching enzyme (amylo-4,6-transferase) is also normal because branch points are present within the glycogen (thus, E is incorrect). Because glycogen is also present, glycogenin is present in order to build the carbohydrate chains, indicating that C is incorrect. If the debranching activity is abnormal (the amylo-1,6-glucosidase), glycogen phosphorylase would break the glycogen down up to four residues from branch points and would then stop. With no debranching activity, the resultant glycogen would contain the normal number of branches, but the branched chains would be shorter than normal.

A patient has large deposits of liver glycogen, which after an overnight fast, had shorter-than-normal branches. This abnormality could be caused by a defective form of which one of the following proteins or activities? (A) Glycogen phosphorylase (B) Glucagon receptor (C) Glycogenin (D) Amylo-1,6-glucosidase (E) Amylo-4,6-transferase

*The answer is D.* If G6PDH is deficient, then the pathway stops there and no reduced glutathione can be produced and, therefore, no removal of hydroperoxides occurs.

A patient in your care has had a hemolytic crisis event and you have on your differential diagnosis notes that the event may indicate a dysfunction of G6PDH. You know that G6PDH in the red blood cell uses glucose-6-phosphate from the glycolytic pathway to form NADPH through a series of reactions. A person with deficient G6PDH would be expected to have all of the following characteristics except which of the following? (A) Increased proportion of glucose metabolism to pyruvate versus ribulose-5-phosphate than in normal (B) Decreased glutathione reductase activity (C) Decreased glutathione peroxidase activity (D) Increased reduction of organic hydroperoxides to hydroxyl products (E) Decreased production of gluconolactone-6-phosphate

*The answer is C.* Glutathione is produced via the γ-glutamyl cycle; the HMP shunt pathway provides the NADPH that allows oxidized glutathione to be converted to reduced glutathione. The other pathways listed (TCA cycle, glycolysis, HMP shunt, and the polyol pathway) do not provide for glutathione synthesis. The TCA cycle is designed to oxidize acetyl-CoA to carbon dioxide and water. Glycolysis is the entry point of sugars into metabolism. The HMP shunt pathway generates five-carbon sugars and NADPH, and the polyol pathway generates sugar alcohols.

A patient is recovering from acute respiratory distress syndrome (ARDS). A major antioxidant found in the fluid lining the bronchial epithelium needed in high concentration for recovery from ARDS. Which of the following biochemical pathways produces the antioxidant referred to? (A) TCA cycle (B) Glycolysis (C) γ-Glutamyl cycle (D) HMP shunt (E) Polyol pathway

*The answer is E.* Glutathione is the major antioxidant in the fluid lining the bronchial epithelium. It is essential for recovery of these tissues. Depletion of glutathione in the airway is thought to greatly increase a person's susceptibility to upper respiratory infections such as influenza. Glutathione is formed in the γ-glutamyl pathway, and oxidized glutathione is regenerated to reduced glutathione using NADPH produced by the HMP shunt pathway. None of the other answers (glycogen, sorbitol, pyruvate, and glucuronate) offer protection against oxidative damage. Glycogen is utilized for the storage of glucose. Pyruvate is the end product of glycolysis. Sorbitol is a product of the polyol pathway. Glucuronic acid is used for xenobiotic metabolism, in general, to increase the solubility of the xenobiotic and to prepare it for excretion.

A patient is recovering from acute respiratory distress syndrome (ARDS). Which of the following is a major antioxidant found in the fluid lining the bronchial epithelium needed in high concentration for recovery from ARDS? (A) Glucuronic acid (B) Pyruvate (C) Sorbitol (D) Glycogen (E) Glutathione

*The answer is C.* The scenario described in the question stem is characteristic of the interaction of a hormone with the G-protein-mediated adenylate cyclase second messenger system. A number of hormones such as glucagon, thyroid-stimulating hormone (TSH), end parathyroid hormone (PTH) exert their effect through transmembrane receptors in coordination with G proteins and cyclic AMP. G-protein is a heterotrimer consisting of alpha, beta, and gamma subunits associated with the intracellular domains of cell membrane-associated receptors. The alpha subunit of the inactivated G-protein is bound to GDP. Upon activation of the receptor, the alpha subunit undergoes a conformational change and GDP is released. Subsequent binding of GTP then allows for the dissociation of the alpha subunit from the remainder of the G-protein complex. There are multiple subtypes of alpha G-proteins, each with different secondary effects. A specific alpha subunit known as Gs (present in the glucagon, TSH, and PTH receptor complexes) activates adenylate cyclase when released from the G-protein complex. Once formed from ATP, cyclic AMP activates a family of enzymes known as the cAMP-dependent protein kinases or protein kinase A. Protein kinase A phosphorylates specific serine or threonine residues in some enzymes, thereby leading to their activation or deactivation. Protein kinase A also phosphorylates several proteins that bind to regulatory regions of genes on the DNA molecule itself.

A patient with newly diagnosed type 2 diabetes mellitus comes to your office asking for more information about her disease. You recall that the physiology of glucose homeostasis is complex, involving multiple intercellular signaling pathways mediated through transmembrane receptor proteins. Binding of intracellular GTP to a specific membrane-associated protein causes rapid metabolic changes in hepatocytes. These metabolic changes include a decrease in intracellular glycogen stores and the release of glucose into the blood. Which of the following is the most likely mediator responsible for these effects? (A) cGMP-dependent protein kinase (B) Tyrosine-specific protein kinase (C) Protein kinase A (D) Phosphodiesterase (E) Janus tyrosine kinase (JAK)

*The answer is A.* This patient has fructose intolerance caused by a deficiency in aldolase B. Aldolase B catalyzes the reaction in which fructose-1-phosphate is metabolized to dihydroxyacetone-phosphate and glyceraldehyde. If there is a deficiency in aldolase B, the reactant fructose-1-phosphate accumulates in the liver. This depletes the liver's stores of free phosphate, which inhibits the production of glucose through gluconeogenesis and causes a fall in cellular ATP levels. The common preceding event to the drop in glucose and ATP levels is a decrease in free phosphate levels.

A pediatrician examines a baby with a deficiency in fructose metabolism. Upon administration of a fructose bolus, the child becomes symptomatic and blood glucose levels begin to decrease. Which of the following will also occur after the administration of a fructose bolus in this patient? (A) A fall in serum phosphate levels (B) A rise in cellular ATP levels (C) A sustained rise in serum fructose levels (D) An increase in the serum pH (E) Large amounts of fructose in the urine

*The answer is D.* There are several variants of G6PD deficiency (X-linked recessive) in Africa. The most common clinically significant variant is the so-called A-type which produces an unstable G6PD enzyme. This variant is found in up to 11% of African-American males. In red cells, NADPH from the HMP shunt is essential for detoxifying reactive oxygen species that otherwise would damage proteins and lipids in the RBC Glutathione reductase and glutathione peroxidase are involved in the normal detoxification process. The A-variant renders the individual's red cells susceptible to oxidative stresses such as those that occur with administration of the antimalarial drug primaquineSulfonamides and dapsone are other agents that can trigger hemolytic episodes. Infections (bacterial or viral) are another important trigger for hemolysis in these individuals. Rarely, G6PD deficiency presents as neonatal jaundice or with chronic hemolysis. In Mediterranean populations, G6RPD deficiency-associated hemolysis may occur in individuals who eat a meal of fava Oeans (Vicia fava) and several hours later develop hemoglobinuria and peripheral vascular collapse secondary to intravascular hemolysis as a result of the oxidant injury initiated by the fava beans

A physician from the United States decides to take a sabbatical from his responsibilities at a teaching hospital to work in a clinic in a remote part of Africa. During his first week at the clinic, he is told that he will be seeing a patient with glucose-6-phosphate dehydrogenase (G6PD) deficiency. Which of the following will be the most likely clinical presentation of this patient? A. A child with short stature, radial abnormalities, absent thumbs, and café-au-lait spots develops severe anemia B. A child who develops myoglobinuria following vigorous exercise C. A neonate with an enlarged spleen and severe anemia D. An adult who develops anemia following use of antimalarial drugs E. An adult who develops severe shortness of breath during an airplane ride

*The answer is E.* The patient has demonstrated a deficiency in riboflavin (urinary excretion of less than 30 micrograms riboflavin/ mg creatinine is considered clinically deficient). Riboflavin is a component of the cofactor FAD, which is required for the conversion of succinate to fumarate by succinate dehydrogenase.

A postoperative patient on intravenous fluids develops lesions in the mouth (angular stomatitis). Urinalysis indicated an excretion of 15 micrograms riboflavin/ mg creatinine, which is abnormally low. Which of the following TCA cycle enzymes is most likely to be affected? (A) Citrate synthase (B) Isocitrate dehydrogenase (C) Fumarase (D) Malate dehydrogenase (E) Succinate dehydrogenase

*The answer is D.* A reduction in overall glycogen synthesis suggests that the biosynthetic pathway is defective in some step. All glycogen molecules have, at their core, a glycogenin protein molecule, which autocatalyzes the addition of six glucose residues, using UDP-glucose as the carbohydrate donor. This structure then provides the initial primer required by glycogen synthase. If the Km for UDP glucose is increased, the rate of formation of glycogen primers will be decreased, as the levels of UDP-glucose may not be sufficient to allow glycogenin to self-prime. This would result in an overall reduction of glycogen levels within the cell. If a glycogen synthase had a reduced Km for UDP-glucose, then the enzyme would be active at lower UDP-glucose levels, and one would expect greater than normal glycogen synthesis. Phosphorylase kinase has as its substrate phosphorylase, not glycogen, so answer B is not correct. If the uridyl transferase had a reduced Km for a substrate, it would proceed at low substrate levels and would not give the resultant phenotype. And, if phosphorylase kinase had an increased Km for glycogen synthase, then glycogen synthase would not be inactivated as rapidly, and glycogen synthesis would be expected to continue under conditions where it should not, leading to enhanced glycogen synthesis.

A researcher created a liver cell line that displayed very low levels of glycogen. The glycogen that was synthesized was of normal structure, but the overall levels of glycogen were about 5% of normal. Which of the following is a potential alteration in the cell line that would lead to these results? (A) An altered glycogen synthase with a reduced Km for UDPglucose (B) An altered phosphorylase kinase with an increased Km for glycogen (C) An altered UTPglucose-1-phosphate uridyl transferase with a decreased Km for glucose-1-phosphate (D) An altered glycogenin with an increased Km for UDPglucose (E) An altered phosphorylase kinase with an increased Km for glycogen synthase

*The answer is D.* In the absence of glutathione, the enzyme glutathione peroxidase will be less active due to the lowered concentration of glutathione. Glutathione peroxidase catalyzes the oxidation of two reduced glutathione molecules by hydrogen peroxide, generating oxidized glutathione and two molecules of water. As glutathione peroxidase is one mechanism whereby hydrogen peroxide levels are reduced, hydrogen peroxide would be expected to accumulate, and can then lead to radical damage of membrane proteins and lipids. Glutathione peroxidase does not require, or react with, superoxide, nitrogen dioxide, nitrous oxide, and peroxynitrate. It is possible that under these conditions, superoxide would also accumulate, due to the increase in concentration of one of the reaction products of superoxide dismutase, hydrogen peroxide. However, there is no evidence that hydrogen peroxide accumulation will inhibit the reaction catalyzed by superoxide dismutase.

A researcher has generated a cell line in which the γ-glutamyl cycle is defective, and glutathione cannot be synthesized. Which radical species might you initially expect to accumulate in this cell? (A) Superoxide (B) Nitrogen dioxide (C) Nitrous oxide (D) Hydrogen peroxide (E) Peroxynitrate

*The answer is C.* In the absence of NADP+, the oxidative steps of the HMP shunt pathway are nonfunctional, so only the nonoxidative steps will occur. In addition, PFK-1 has been made nonfunctional, such that glyceraldehyde-3-phosphate (G3P) cannot be produced from either fructose-6-phosphate (F6P) or glucose-6-phosphate (G6P). In order to generate ribose-5-phosphate (R5P) under these conditions, both F6P and G3P need to be provided. These two substrates will react, using transketolase as a substrate, to generate erythrose-4-phosphate (E4P) and xylulose-5-phosphate (X5P, step 1 in the figure below). The X5P will be epimerized to ribulose-5-phosphate (Ru5P, step 2 in the figure below), and then isomerized to R5P (step 3 in the figure below). Glucose-6-phosphate cannot be used as a substrate because it cannot be converted to G3P (due to the block in PFK-1). Pyruvate cannot be used as a substrate in extracts of red blood cells because such cells do not have pyruvate carboxylase, so the pyruvate cannot be converted to either F6P or G3P.

A researcher is studying the HMP shunt pathway in extracts of red blood cells, in the absence of NADP+, and in which PFK-1 has been chemically inactivated. Which carbon substrates are required to generate ribose-5- phosphate in this system? (A) Glucose-6-phosphate and sedoheptulose-7-phosphate (B) Glucose-6-phosphate and glyceraldehyde-3-phosphate (C) Fructose-6-phosphate and glyceraldehyde-3-phosphate (D) Fructose-6-phosphate and pyruvate (E) Glucose-6-phosphate and pyruvate

*The answer is C.* Biochemical processes for cellular metabolism occur in distinct locations within cells. For example, β-oxidation of fatty acids, ketogenesis, the citric acid cycle, parts of the urea cycle (carbamoyl phosphate synthetase 1 and ornithine transcarbamoylase) and pyruvate carboxylation all occur exclusively within the mitochondria. Transketolase is an enzyme of the pentose phosphate pathway that uses thiamine (Vitamin B1) as a cofactor. All of the reactions of the pentose phosphate pathway occur in the cytoplasm, so the enzyme transketolase would remain in the homogenate described in the question stem while the other enzymes mentioned in the other answer choices would be removed because they all reside within the mitochondria. *Educational Objective:* Biochemical processes for cellular metabolism occur in distinct locations within cells. For example, β-oxidation of fatty acids, ketogenesis, the citric acid cycle, parts of the urea cycle (carbamoyl phosphate synthetase 1 and ornithine transcarbamoylase) and pyruvate carboxylation all occur exclusively within the mitochondria. All of the reactions of the pentose phosphate pathway occur in the cytoplasm.

A sample of liver parenchyma is homogenized and centrifuged to remove membrane components and organelles leaving only the cytosol and the proteins it contains. Which of the following enzyme activities is most likely present in the homogenate? A. Pyruvate carboxylase B. Ornithine transcarbamoylase C. Transketolase D. Succinate dehydrogenase E. 3-Hydroxy-3-methy1glutary1-CoA Iyase

*The answer is F.* In contrast to the case with glycolysis, the only site of substrate-level phosphorylation in the tricarboxylic acid cycle is the step catalyzed by succinyl CoA synthetase. In this step, the cleavage of CoA from succinyl CoA to produce succinate is the utilization of a high-energy bond of CoA to phosphorylate GDP with organic phosphate to produce GTP. Since NADH generated by glycolysis in the cytoplasm cannot pass across the mitochondrial membrane, shuttles are used to bring the electrons into the mitochondria for oxidative phosphorylation. In the glycerol phosphate shuttle, NADH + H+ in the cytoplasm reduces dihydroxyacetone phosphate to glycerol phosphate, which is capable of entering the mitochondria. In the mitochondria, the glycerol phosphate is oxidized back to dihydroxyacetone phosphate, which can then diffuse back out into the cytoplasm. During this process, flavin (FADH2) is reduced and is capable of generating 2 ATP via oxidation by the respiratory chain. In contrast, the malate-aspartate shuttle allows the formation of 3 ATP equivalents for each mole of cytoplasmic NADH + H+ generated. The malate-aspartate shuttle is found mainly in the heart and liver. The process of oxidative phosphorylation that is coupled to electron transport occurs because of the proton gradient maintained across the mitochondrial membrane. ATP is formed by mitochondrial ATPase by the movements of protons across this gradient. In the presence of substances like 2,4-dinitrophenol (DNP), oxidation of oxidative phosphorylation is uncoupled. This occurs because DNP carries the protons across the mitochondrial membrane, shortcircuiting the phosphorylations that normally occur. While this reaction is not biologically useful, it does mimic the normal uncoupling of phosphorylation that can occur under certain biologic conditions and is used to generate heat to maintain body temperature. This occurs in certain mammals adapted to cold, newborn mammals, and hibernating animals. In these animals, this process of thermogenesis occurs in specialized brown adipose tissue. The uncoupling protein is called thermogenin. Since electron transport is tightly coupled to phosphorylation, under physiologic conditions electrons do not flow through the electron transport chain to O2 unless ADP is simultaneously phosphorylated to ATP. If the level of ADP is low, oxidative phosphorylation does not occur at as high a rate and the rate of oxygen consumption in tissue decreases. Respiratory control is the regulation of the rate of oxidative phosphorylation by ADP levels.

A step in the tricarboxylic acid cycle is described by which of the following? a. Produces 3 mol of ATP equivalents for each cytoplasmic NADH produced b. Carries protons across the inner mitochondrial membrane c. Determines the rate of oxidative phosphorylation d. Yields 2 mol of ATP for each cytoplasmic NADH e. Is utilized by hibernating animals to produce heat f. Requires inorganic phosphate g. Defines respiratory quotient (RQ)

*The answer is A.* In gluconeogenesis, pyruvate cannot be directly converted to phosphoenolpyruvate in, therefore, pyruvate carboxylase converts pyruvate to oxaloacetate, which can be converted to phosphoenolpyruvate by phosphoenolpyruvate carboxykinase using two ATP equivalents per molecule of phosphoenolpyruvate. Fructose-1.6-bisphosphatase is the enzyme that splits fructose-1,6-bisphosphate into fructose-6-phosphate and inorganic phosphate. lt is also required for gluconeogenesis.

A study is conducted to evaluate the effects of pollution on the water supply. One of the substances present in the city's drinking water is found to be capable of inhibiting gluconeogenesis, although the step in which gluconeogenesis is affected is unknown. Which of the following pairs of enzymes is most likely targeted by the pollutant? A. Fructose-1,6-bisphosphatase and pyruvate carboxylase B. Glucose-6-phosphatase and phosphofructokinase-1 C. Glucose-6-phosphatase and pyruvate dehydrogenase D. Phosphoenolpyruvate carboxykinase and glucokinase E. Pyruvate kinase and pyruvate carboxylase

*The answer is C.* The woman has a BMI in the unhealthy range (15.7), indicating inadequate nutrient uptake. Since her nutrient uptake is poor, most of the glucose present in her blood is derived from gluconeogenesis, as her glycogen stores are most likely depleted. The substrates for gluconeogenesis are lactate (derived from red blood cell metabolism), glycerol (from triglyceride degradation), and amino acids derived from protein muscle degradation. Glutamine is a glucogenic amino acid, and is also used to transport nitrogen groups from the muscle to the liver for safe excretion via the urea cycle. Leucine is a strictly ketogenic amino acid (giving rise only to acetyl-CoA), and leucine carbons cannot be used to make glucose via gluconeogenesis. Fatty acids are also strictly ketogenic, and cannot be used for glucose production (fatty acids give rise to acetyl-CoA, which cannot be used to produce net glucose). Lysine is also a strictly ketogenic amino acid, and cannot be used for glucose production. Heme degradation gives rise to bilirubin, which cannot be further degraded, and none of the carbons of heme can be used for glucose production.

A thin, anxious woman, who is 5′ 6″ tall, weighs 92 lb. Blood work indicates a glucose level of 70 mg/dL under fasting conditions. Her liver is using which of the following as precursors for glucose production under these conditions? (A) Glycerol, lactate, and leucine (B) Fatty acids, alanine, and glutamine (C) Glycerol, lactate, and glutamine (D) Glycerol, fatty acids, and glutamine (E) Lactate, heme, and lysine

*The answer is D.* This patient displays exercise intolerance suggestive of type V glycogen storage disease (McArdle disease). The absence of muscle phosphorylase limits ATP generation by glycogenolysis and results in glycogen accumulation. Approximately 50% of patients report burgundy-colored urine after exercise due to myoglobinuria secondary to rhabdomyolyis. Intense myoglobinuria after vigorous exercise may cause renal fa ilure. The table compares glycogen storage diseases.

A well-developed 23-year-old man notices t hat he fatigues easily and often gets muscle cramps. He also notes that when exercising, he often needs to take short breaks because "my legs just give out." He reports having burgundy-colored urine after exercise. After an ischemic exercise test, the doctor notes exaggerated blood ammonia levels without an increase in blood lactate due to defects in muscle glycogenolysis. What finding would confirm the diagnosis of this patient's glycogen storage disease? A. Liver biopsy showing deficiency of phosphorylase B. Muscle biopsy showing a deficiency of debranching enzyme C. Muscle biopsy showing a deficiency of lysosomal α-glucosidase D. Muscle biopsy showing a deficiency of myophosphorylase E. Muscle biopsy showing deficiency of phosphoglycerate kinase

*The answer is B.* For this woman to synthesize lactose, she needs to synthesize the precursors UDP-galactose and glucose, both of which are available from glucose. Glucose is converted to glucose-6-phosphate by hexokinase in the breast, and then phosphoglucomutase will convert this to glucose-1-phosphate (G1P). The G1P will react with UTP in the glucose-1-phosphate uridyl transferase reaction, producing UDP-glucose. The C4 epimerase will then produce UDP-galactose from UDP-glucose. The UDP-galactose then condenses with free glucose (using lactose synthase) to produce lactose and UDP. The other enzymes listed as answers are not required to produce lactose from the single precursor glucose. Fructokinase is unique for fructose metabolism. Aldolase is a glycolytic enzyme, which is deficient in hereditary fructose intolerance. Phosphohexose isomerase converts glucose-6-phosphate to fructose-6-phosphate, which is not required for lactose synthesis. Classical galactosemia (severe, type 1) is a deficit of galactose-1-phosphate For this woman to synthesize lactose, she needs to synthesize the precursors UDP-galactose and glucose, both of which are available from glucose. Glucose is converted to glucose-6 phosphate by hexokinase in the breast, and then phosphoglucomutase will convert this to glucose-1-phosphate (G1P). The G1P will react with UTP in the glucose-1-phosphate uridyl transferase reaction, producing UDP-glucose. The C4 epimerase will then produce UDP-galactose from UDP-glucose. The UDP-galactose then condenses with free glucose (using lactose synthase) to produce lactose and UDP. The other enzymes listed as answers are not required to produce lactose from the single precursor glucose. Fructokinase is unique for fructose metabolism. Aldolase is a glycolytic enzyme, which is deficient in hereditary fructose intolerance. Phosphohexose isomerase converts glucose- 6-phosphate to fructose-6-phosphate, which is not required for lactose synthesis. Classical galactosemia (severe, type 1) is a deficit of galactose-1-phosphate.

A woman with nonclassical galactosemia is considering becoming pregnant and is concerned that she will be unable to synthesize lactose in order to breast-feed her child. Her physician, who recalls her biochemistry, tells her this should not be a problem, and that she will be able to synthesize lactose at the appropriate time. This is true due to the presence of which of the following? (A) Galactose-1-phosphate uridyl transferase (B) Phosphoglucomutase (C) Fructokinase (D) Aldolase (E) Phosphohexose isomerase

*The answer is E.* The physical symptoms suggest a deficiency in an enzyme responsible for carbohydrate degradation. The symptoms observed following the ingestion of dairy products suggest that the patient is deficient in lactase.

A young black man entered his physician's office complaining of bloating and diarrhea. His eyes were sunken and the physician noted additional signs of dehydration. The patient's temperature was normal. He explained that the episode had occurred following a birthday party at which he had participated in an ice cream eating contest. The patient reported prior episodes of a similar nature following ingestion of a significant amount of dairy products. This clinical picture is most probably due to a deficiency in: A. salivary α-amylase. B. isomaltase. C.pancreatic α-amylase. D.sucrase. E. lactase.

*The answer is A.* The child described has type 1 diabetes mellitus due to a defect in insulin production. Proinsulin is synthesized in the endoplasmic reticulum of cells in the pancreas. It is packaged into vesicles and transported to the Golgi apparatus, where it is cleaved by specific proteases to form the active hormone insulin. Chymotrypsinogen is secreted as a zymogen as well and is proteolytically cleaved by trypsin to produce the active enzyme chymotrypsin. Chymotrypsin breaks down protein by cleaving peptide bonds in the small intestine.

A young boy has a metabolic disorder that sometimes causes him to go into a ketoacidotic state. After performing a few tests, the physician discovers that the child is constitutively deficient in insulin as well as its precursor proinsulin. Which of the following enzymes is activated via the same type of mechanism as insulin? A. Chymotrypsin B. Glucose-6-phosphate dehydrogenase C. Glycogen synthase D. Hexokinase E. Phosphofructokinase

*The answer is C.* Elevated NADPH levels would inhibit G6PDH activity because [NADP+], a required substrate for the reaction is decreased. Thus, choices B, D, and E can be eliminated. Decreased membrane fatty acyl hydroperoxide content would require less peroxidase activity and, therefore, less G6PDH activity, whereas elevated levels would tend to increase G6PDH activity if NADPH levels are reduced. Elevated vicine and convicine levels would increase membrane fatty acid hydroperoxide content, thus an increased need for peroxidase activity, and, therefore, increased G6PDH activity if NADPH levels are low. Choice A can be eliminated because there is a lack of a need for G6PDH activity due to low levels of acyl hydroperoxides and vicine/convicine. Condition C will tend to increase G6PDH activity.

A young male patient with an episodic hemolytic crisis returns to your clinic several months later with another episode. He did not follow your advice to avoid certain foodstuffs. You now have another opportunity to pinpoint the precise cause of the malady. You know that G6PDH, the rate-limiting step of the hexose monophosphate shunt is regulated by the ratio of NADPH/ NADP+ and that changes in binding efficiency could change the ability to protect the erythrocyte. Which condition would signal the enzyme to increase its activity?

*The answer is D.* Glutathione peroxidase is the enzyme active in removing hydroperoxide products, resulting from reactive oxygen species attacks on membrane fatty acid substituents. Glutathione reductase reduces glutathione disulfide to reduced glutathione. 6-phosphogluconate dehydrogenase produces a molecule of CO2, and NADPH and ribulose-5-phosphates. Glutathione reductase requires a selenium cofactor. Glutathione peroxidase can only use reduced glutathione as an electron donor not NADPH and not NADH.

A young otherwise healthy male patient complains of weakness and has a low erythrocyte count. You suspect a recent hemolytic crisis and check his erythrocytes and G6PDH levels. His erythrocyte count is down and his G6PDH is normal. You know that the pathway for protecting the erythrocyte membrane from damage is complex and that glutathione peroxidase is involved. Peroxidases use reducing equivalents produced by metabolism from glucose to reduce organic hydroperoxides to the hydroxyl level. Which of the following statements best describes the action of glutathione peroxidase? (A) Glutathione peroxidase reduces glutathione disulfide back to reduced glutathione (B) Glutathione peroxidase produces 1 molecule of CO2 in its reaction cycle (C) Glutathione peroxidase requires a selenium ion cofactor (D) Glutathione peroxidase reduces organic hydroperoxides to organic hydroxyl compounds (E) Glutathione peroxidase can use NADPH or NADH directly as a source of reducing equivalents

*The answer is E.* This patient most likely has hereditary fructose intolerance caused by a deficiency in the enzyme aldolase B. During the normal metabolism of fructose (a 6-carbon sugar similar to glucose), fructose is ingested, taken up into liver cells, and phosphorylated by fructokinase, trapping fructose intracellularly. For fructose-1-phosphate to be utilized for energy, it is then cleaved into glyceraldehyde and dihydroxyacetone phosphate by the enzyme aldolase B. These 3-carbon structures, as well as their downstream products, are then utilized by an array of metabolic pathways, including glycolysis, the Krebs cycle, gluconeogenesis, and glycogen synthesis. Hereditary fructose intolerance is an autosomal recessive disorder with an incidence of 1 in 20,000 bi1ths. It typically presents with vomiting and lethargy in infants after the introduction of fructose into the diet/ Note that breast milk contains the sugars glucose and galactose (in the form of the disaccharide lactose) and thus does not cause symptoms in infants. However, symptoms will appear once the child is switched to baby formula or a normal diet, which both contain fructose. The absence of aldolase B causes significant impairment in the metabolism of fructose. In these patients, after an ingestion of a fructose-containing meal, fructose-1-phosphate cannot be processed by aldolase B, and thus accumulates in hepatocytes. The build-up of fructose-1-phosphate depletes the cellular stores of free phosphate. Glycogen is unable to break down, which causes hypoglycemia . Of all the choices listed, hypoglycemia correctly identifies the metabolic derangement experienced by patients after ingestion of a fructose load. To prevent the life-threatening consequences of hereditary fructose intolerance, including cirrhosis and severe hypoglycemia, affected individuals must follow a strict diet devoid of both fructose and sucrose.

An infant is born at term without perinatal complications. She grows appropriately throughout the time she is breast-fed but on transition to baby formula, she begins appearing very ill. Soon after ingestion of packaged baby food, she begins vomiting and becomes lethargic. Although she recovers somewhat in between feedings, she has lost a substantial amount of weight and is failing to thrive. The baby's pediatrician suspects an inborn error of metabolism. If the baby were to ingest conventional baby formula at the pediatrician's office, what blood work abnormalities might be appreciated shortly thereafter? A. Alkalemia B. Hyperphosphatemia C. Hypomagnesemia D. Marked hyperglycemia E. Marked hypoglycemia

*The answer is C.* Blood glucose is rapidly converted to glucose-6-phosphate upon entering cells by hexokinase or, in the case of the liver, by glucokinase. Glucose-6-phosphate is in equilibrium with glucose-1-phosphate via the action of phosphoglucomutase. Glucose-1- phosphate is activated by UTP to form UDP-glucose, which is added to glycogen by an α-1,4 linkage in the presence of glycogen synthase. To increase the solubility of glycogen and to increase the number of terminal residues, glycogen-branching enzyme transfers a block of about 7 residues from a chain at least 11 residues long to a branch point at least 4 residues from the last branch point. The branch is attached by an α-1,6 linkage.

After a meal, blood glucose enters cells and is stored as glycogen, particularly in the liver. Which of the following is the donor of new glucose molecules in glycogen? (A) UDP-glucose-1-phosphate (B) UDP-glucose (C) UDP-glucose-6-phosphate (D) Glucose-6-phosphate (E) Glucose-1-phosphate

*The answer is C.* During an overnight fast, glycogenolysis and gluconeogenesis occur to some degree. Following a well-rounded breakfast, amino acids are available for protein synthesis, glycogen synthesis occurs from excess glucose, gluconeogenesis decreases, and fatty acid synthesis occurs from excess acetyl CoA produced from dietary sources. Thus, the activity of enzymes of gluconeogenesis (pyruvate carboxylase and phosphoenolpyruvate carboxykinase) decreases, while the activity of enzymes of fatty acid synthesis (acetyl CoA carboxylase) increases.

After a well-rounded breakfast, which of the following would be expected to occur? a. Increased activity of pyruvate carboxylase b. Decreased activity of acetyl CoA carboxylase c. Decreased rate of glycogenolysis d. Decreased rate of protein synthesis e. Increased activity of phosphoenolpyruvate carboxykinase

*The answer is B.* Glucokinase catalyzes the initial step of glycolysis, which is the phosphorylation of glucose to glucose-6-phosphate. In most other tissues, this process is catalyzed by hexokinase. Both enzymes are found in the liver. Glucokinase has a higher Michaelis-Menten immunoconstant and a higher Vmax than hexokinase; it thus has a low affinity for glucose but large capacity of activity. Importantly, it is not inhibited by glucose-6-phosphate, as is hexokinase.

After consumption of a carbohydrate-rich meal, the liver continues to convert glucose to glucose-6-phosphate. The liver's ability to continue this processing of high levels of glucose is important in minimizing increases in blood glucose after eating. What is the best explanation for the liver's ability to continue this conversion after eating a carbohydrate-rich meal? (A) The hepatocyte cell membrane's permeability for glucose-6-phosphate (B) The high maximum reaction rate of glucokinase (C) The high maximum reaction rate of hexokinase (D) The high Michaelis-Menten constant of hexokinase (E) The low Michaelis-Menten constant of glucokinase

*The answer is E.* Most monosaccharides are transported with sodium from the intestinal lumen into the enterocyte. The energy for active transport of the carbohydrate is derived from the sodium gradient that is established by the Na+, K+ ATPase, which pumps sodium out of the cell (three atoms of sodium) in exchange for potassium (two atoms of potassium). This creates both a sodium gradient (outside concentration higher) and a charge gradient (outside positive as compared to inside the cell) across the plasma membrane. Due to these gradients, the entry of sodium into the cell is energetically favorable, and the monosaccharide piggybacks with the sodium for transport into the cell. The Na /H+ exchanger is not operative in intestinal epithelial cells, and none of the other enzymes (glucose-6-phosphate dehydrogenase, hexokinase, and chloride transporter) will create the necessary sodium gradient for monosaccharide transport.

After eating a meal containing carbohydrates, the monosaccharides must be absorbed from the intestinal lumen. This transport is dependent on which of the following enzymes? (A) Na+/H+ antiporter (B) Glucose-6-phosphate dehydrogenase (C) Hexokinase (D) Chloride transporter (E) Na+, K+ ATPase

*The answer is B.* The excessive bleeding, increased PTT, and correction of the PTT with addition of normal serum point to hemophilia. The most common types of hemophilia are hemophilia A (deficiency in clotting factor VIII) and hemophilia B (Christmas disease; caused by a deficiency in clotting factor IX). The genes encoding both of these proteins are carried on the X-chromosome, making these X-linked recessive diseases. The only disease listed above which is inherited in an X-linked manner is Fabry disease, caused by a defect in α-galactosidase A involved with degradation of glycosphingolipids.

After suffering injuries in a motor vehicle accident, a 7-year-old boy undergoes open reduction surgery to repair a compound fractured femur. Post-surgically, the boy undergoes severe hemorrhage and re-quires transfusion of 8 units of blood. Coagulation studies demonstrate the PT time to be normal, but the PTT time is prolonged. Mixing the boy's plasma with normal plasma returns the PTT time to normal. The mode of inheritance of this boy's disease is most similar to which of the following inherited enzyme deficiencies? A. Adenosine deaminase deficiency B. α-Galactosidase A deficiency C. Glucocerebrosidase deficiency D. Hexosaminidase A deficiency E. Dystrophia myotonica protein kinase deficiency

*The answer is C.* Cyclic AMP is synthesized by adenylate cyclase in response to hormonal stimulation of specific receptors in cells. In all eukaryotic cells studied to date, increased cyclic AMP levels activate a cyclic AMP-dependent protein kinase. The protein kinase, in turn, phosphorylates other enzymes, activating or inactivating them. The kinase is made up of two regulatory and two catalytic subunits. Binding of cyclic AMP to the regulatory subunits allows dissociation of the catalytic subunits. The catalytic subunits are active in the dissociated state. Decrease in cellular cyclic AMP levels by phosphodiesterase breakdown of the nucleotide frees the regulatory subunits from cyclic AMP binding. This allows reassociation of regulatory and catalytic subunits and subsequent inactivation.

All known effects of cyclic AMP in eukaryotic cells result from a. Activation of the catalytic unit of adenylate cyclase b. Activation of synthetases c. Activation of protein kinase d. Phosphorylation of G protein e. Stimulation of Ca++ release from endoplasmic reticulum

*The answer is B.* The pentose phosphate pathway produces a carbohydrate containing 7 carbon atoms known as sedoheptulose 7-phosphate. NADPH is a major product of the pathway used to synthesize important biomolecules, such as fatty acids. This pathway is also important in red blood cells because a product of the pathway (NADPH) is required for maintaining the proper oxidation-reduction state. A key enzyme in the pathway is glucose 6-phosphate dehydrogenase; it is the rate-limiting step. Five carbon atoms are present in this pathway, such as ribulose 5-phosphate, ribose 5-phosphate, and xylulose 5-phosphate.

All of the following statements about the phosphogluconate (pentose phosphate) pathway are correct EXCEPT which one? (A) A carbohydrate containing 7 carbon atoms can be produced by this pathway. (B) containing 5 carbon atoms are absent from this pathway. (C) A major product of the pathway is NADPH. (D) The pathway is important in red blood cells because a product of the pathway is required for maintaining the proper oxidation-reduction state. (E) A key enzyme in the pathway is glucose 6-phosphate dehydrogenase

*The answer is B.* A molecule of guanosine triphosphate is synthesized from guanosine diphosphate and phosphate at the cost of hydrolyzing succinyl CoA to succinate and CoA. This constitutes substrate-level phosphorylation, and, in contrast to oxidative phosphorylation, this is the only reaction in the citric acid cycle that directly yields a high-energy phosphate bond. The sequence of reactions from α-ketoglutarate to succinate is catalyzed by the α-ketoglutarate dehydrogenase complex and succinyl-CoA synthetase, respectively. α-ketoglutarate + NAD+ + acetyl CoA → succinyl CoA + CO2 + NADH succinyl CoA + Pi + GDP → succinate + GTP + acetyl CoA

Among the many molecules of high-energy phosphate compounds formed as a result of the functioning of the citric acid cycle, one molecule is synthesized at the substrate level. In which of the following reactions does this occur? (A) Citrate →α-ketoglutarate (B) α-ketoglutarate → succinate (C) Succinate → fumarate (D) Fumarate → malate (E) Malate → oxaloacetate

*The answer is A.* The patient is lacking muscle glycogen phosphorylase and cannot utilize muscle glycogen for energy. This is another glycogen storage disease, type V, McArdle disease. The lack of muscle glycogen phosphorylase is why lactate production during exercise is very low. There are many glycogen particles present in the muscle cells just below the sarcolemma, as the glycogen is not able to be degraded. Muscle damage also results from vigorous exercise, releasing myoglobin into the circulation, which is what leads to the reddish-brown urine after exercise. Alterations in liver enzymes (phosphorylase or PFK-1) would not affect exercise tolerance in the muscle. Muscle does not contain glucose-6-phosphatase, and this problem is not due to a lack of muscle GLUT4 transporters, as the muscle cannot utilize stored, internal glucose supplies.

An 18-year-old man visits the doctor due to exercise intolerance. His muscles become stiff or weak during exercise, and he sometimes cramps up. At times, his urine appears reddish-brown after exercise. An ischemic forearm exercise test indicates very low lactate production. A potential enzyme defect in this man is which of the following? (A) Muscle glycogen phosphorylase (B) Liver glycogen phosphorylase (C) Liver PFK-1 (D) Muscle glucose-6-phosphatase (E) Muscle GLUT4 transporters

*The answer is B.* Glucose-6-phosphate dehydrogenase (G6PD) is the rate-limit ing enzyme in the hexose monophosphate pathway, which produces nicotinamide adenine dinucleotide phosphate (NADPH) and helps detoxify free radicals. Patients who are deficient in G6PD have an increased susceptibility to the effects of free radicals, especially in their RBCs. Sulfa drugs, fava beans, nitrofurantoin, aspirin, and primaquine are all oxidizing agents that commonly lead to hemolysis in patients with G6PD deficiency, who cannot neutralize free radicals in their RBCs. The patient in this question is treated with a sulfa drug and later presents with the hallmark signs of hemolysis- fatigue, peripheral blood smear results (Heinz cells in the RBCs, characteristic of G6PD deficiency; see image), and scleral icterus- indicating an underlying G6PD deficiency. It is important to remember that G6PD deficiency is an X-linked recessive disease.

An 18-year-old woman presents to her university's health center with complaints of dysuria and increased frequency of urination for the past 3 days. A urine specimen reveals abundant gram-negative rods and leukocytes. The patient is given sulfamethoxazole/trimethoprim for treatment of her urinary tract infection. Two days later she returns complaining of fatigue. On physical examination the patient has conjunctival icterus. Peripheral blood smear reveals spherocytes and precipitates inside the RBCs. Which of the following is most likely deficient in this patient? A. α-Ketoacid dehydrogenase B. Glucose-6-phosphate dehydrogenase C. Niacin D. Pyruvate dehydrogenase E. Pyruvate kinase

*The answer is C.* Thiamine is a water-soluble vitamin that is converted to the coenzyme thiamine pyrophosphate, This coenzyme is used by pyruvate dehydrogenase to convert pyruvate to acetyl coenzyme A (acetyl-coA). Other coenzymes and cofactors it uses include lipoic acid. CoA, FAD *, and NAD T . In the absence of thiamine, pyruvate accumulates and can be converted by lactate dehydrogenase to lactate, which is spilled in the blood, causing lactic acidosis.

An 8-month-old boy is admitted to the pediatric intensive care unit because of respiratory distress. Laboratory studies show an acidosis with a very high serum concentration of lactate. Treatment with intravenous hydration and thiamine is begun. The rationale in this case for thiamine administration is that thiamine is converted to a coenzyme used by which of the following enzymes? A. Lactate dehydrogenate B. Pyruvate carboxylase C. Pyruvate dehydrogenase D. Pyruvate Kinase E. Transketolase

*The answer is B.* This patient has Pompe disease, also called glycogen storage disease type II (shown in the diagram at square II). Pompe disease is an autosomal recessive disease that is characterized by a deficiency or defect in lysosomal α-1,4-glucosidase. This enzyme is necessary for the dissolution of the polymer linkages in glycogen. In its absence, glycogen accumulates to toxic levels in both the cytoplasm and lysosomes. This disease is commonly associated with myopathies, particularly heart muscle thickening and subsequent failure from glycogen build-up in the cardiac muscle.

An 8-month-old boy is brought to the pediatrician by his parents because he has recently lost the ability to crawl or hold his toys. On examination the patient is tachypneic and breathing with considerable effort; the liver is palpable five finger widths below the right costal margin. X-ray of the chest reveals cardiomegaly. He has a difficult time sitting upright and cannot squeeze the physician's fingers or the ring of his pacifier with any noticeable force. Despite a number of interventions, the child's symptoms continue to worsen until his death 2 weeks later. On autopsy, it is likely that this patient's cells will contain an accumulation of which of the following substances? A. Glucose B. Glycogen C. Oxaloacetate D. Pyruvate E. Urea

*The answer is B.* This patient has Pompe disease, a glycogen storage disorder. Pompe disease is an autosomal recessive disease that is characterized by a deficiency or defect in lysosomal α-1,4-glucosidase. This enzyme is necessary for the dissolution of the polymer linkages in glycogen. In its absence, glycogen accumulates to toxic levels in both the cytoplasm and lysosomes.

An 8-month-old boy is brought to the pediatrician by his parents because he has recently lost the ability to crawl or hold his toys. On examination the patient is tachypneic and breathing with considerable effort; the liver is palpable five finger widths below the right costal margin. X-ray of the chest reveals cardiomegaly. He has a difficult time sitting upright and cannot squeeze the physician's fingers or the ring of his pacifier with any noticeable force. Despite a number of interventions, the child's symptoms continue to worsen until his death two weeks later. On autopsy, it is likely that this patient's cells will contain an accumulation of which of the following substances? (A) Glucose (B) Glycogen (C) Oxaloacetate (D) Pyruvate (E) Urea

*The answer is E.* The child has biotinidase deficiency, which results in a functional biotin deficiency. Biotinidase is required to remove covalently linked biotin from proteins in our diet and from proteins that have turned over within the body. An inability to do this leads to a biotin deficiency (as most ingested biotin is linked to proteins). The hair and scalp problems have been attributed to an inability to synthesize fatty acids (as acetyl-CoA carboxylase is missing biotin). Since pyruvate carboxylase is also inoperative (due to the lack of biotin), gluconeogenesis is impaired, and ketone bodies will be synthesized by the liver to compensate for reduced glucose production. Priopionyl-CoA carboxylase is also impaired, leading to the elevated levels of propionic acid. Since gluconeogenesis is impaired, excess pyruvate will be converted to lactate since it cannot be converted to oxaloacetate. The optic atrophy may be due to an inability to synthesize fatty acids within the neurons or a lack of energy due to reduced gluconeogenesis.

An 8-month-old girl was taken to the emergency department due to the onset of sudden seizures. The child had brittle hair, with some bald spots, and skin rashes. An ophthalmologist noted optic atrophy. Urinalysis showed slightly elevated ketones and the presence of other organic acids (such as propionate and lactate). Treatment of this child with which of the following can successfully alleviate the problems? (A) Thiamine (B) Niacin (C) Ribofl avin (D) Carnitine (E) Biotin

*The answer is D.* !-cell disease is an autosomal recessive disorder that results from improper intracellular trafficking. This impaired trafficking results from the failure to add a mannose-6-phosphate residue to proteins that should be directed to lysosomes. On a cellular level, this results in the presence of numerous intracytoplasmic inclusions in cells of mesenchymal origin. These inclusions are membrane-bound vacuoles that are filled with fibrillogranular material, including a variety of lipids, mucopolysaccharides, and oligosaccharides. Clinically this deficiency results in a select group of identifying features. Be on the lookout for coarse facial features in a baby that is developmentally delayed and has restricted joint movement .

An 8-week-old boy is brought to his pediatrician because his mot her notes abnormal limb movements. Although both pregnancy and birth were not complicated, and there is an unremarkable family history and healthy siblings, this child has had developmental delay since birth. On examination, the child has normal vital signs, coarse facial features, diffuse joint stiffness, claw hand deformities, and kyphoscoliosis. Muscle biopsy reveals numerous intracytoplasmic inclusions in cells. What is the pathophysiology of this disease? A. Congenital herpes infection B. Congenital rubella infection C. Failure to cross-link collagen fibrils D. Improper protein trafficking E. Mutations in the structural protein fibrillin

*The answer is E.* The patient is exhibiting the classic signs of lactose intolerance, in which intestinal lactase levels are low, and the major dietary component of milk products (lactose) cannot be digested. Lactase will split the β-1,4 linkage between galactose and glucose in lactose. The lactose thus passes unmetabolized to the bacteria inhabiting the gut, and their metabolism of the disaccharide leads to the observed symptoms. Combining two sugars in a dehydration reaction creates a glycosidic bond. Adding a sugar to the nitrogen of a nitrogenous base also creates an N-glycosidic bond. A sugar bond is not an applicable term in biochemistry. Ester linkages contain an oxygen linked to a carbonyl group. A phosphodiester bond is a phosphate in two ester linkages with two different compounds (such as the 3′-5′ link in the sugar-phosphate backbone of DNA and RNA). An amide bond is the joining of an amino group with a carboxylic acid with the loss of water.

An African native who is going to college in the United States experiences digestive problems (bloating, diarrhea, and flatulence) whenever she eats foods containing milk products. She is most likely deficient in splitting which type of chemical bond? (A) A sugar bond (B) An ester linkage (C) A phosphodiester bond (D) An amide bond (E) A glycosidic bond

*The answer is C.* The patient has McArdle disease, a glycogen storage disease caused by a deficiency of muscle glycogen phosphorylase. Because he or she cannot degrade glycogen to produce energy for muscle contraction, he or she becomes fatigued more readily than a normal person (thus, A is incorrect), the glycogen levels in her muscle will be higher than normal as a result of the inability to degrade them (thus, D is incorrect), and his or her blood lactate levels will be lower because of the lack of glucose for entry into glycolysis. He or she will, however, draw on the glucose in his or her circulation for energy, so his or her forearm blood glucose levels will be decreased (thus, B is incorrect), and because the liver is not affected, blood glucose levels can be maintained by liver glycogenolysis (thus, E is incorrect).

An adolescent patient with a deficiency of muscle phosphorylase was examined while exercising his or her forearm by squeezing a rubber ball. Compared with a normal person performing the same exercise, this patient would exhibit which of the following? (A) Exercise for a longer time without fatigue. (B) Have increased glucose levels in blood drawn from his or her forearm. (C) Have decreased lactate levels in blood drawn from his or her forearm. (D) Have lower levels of glycogen in biopsy specimens from his or her forearm muscle. (E) Hyperglycemia

The answer is C. The patient has McArdle disease, a glycogen storage disease caused by a deficiency of muscle glycogen phosphorylase. Because he or she cannot degrade glycogen to produce energy for muscle contraction, he or she becomes fatigued more readily than a normal person (thus, A is incorrect), the glycogen levels in her muscle will be higher than normal as a result of the inability to degrade them (thus, D is incorrect), and his or her blood lactate levels will be lower because of the lack of glucose for entry into glycolysis. He or she will, however, draw on the glucose in his or her circulation for energy, so his or her forearm blood glucose levels will be decreased (thus, B is incorrect), and because the liver is not affected, blood glucose levels can be maintained by liver glycogenolysis (thus, E is incorrect).

An adolescent patient with a deficiency of muscle phosphorylase was examined while exercising his or her forearm by squeezing a rubber ball. Compared with a normal person performing the same exercise, this patient would exhibit which of the following? (A) Exercise for a longer time without fatigue. (B) Have increased glucose levels in blood drawn from his or her forearm. (C) Have decreased lactate levels in blood drawn from his or her forearm. (D) Have lower levels of glycogen in biopsy specimens from his or her forearm muscle. (E) Hyperglycemia

*The answer is A.* The key to this question is recognizing that, in the setting of ischemia, the patient's acidosis is likely due to lactic acid production under anaerobic conditions. Glycolysis is the biochemical pathway in which glucose is oxidized to pyruvate within the cytosol, as shown in the illustration. In anaerobic conditions, pyruvate is converted into lactic acid by lactate dehydrogenase, an enzyme located in the cytoplasm. In aerobic conditions, pyruvate is then transported into the mitochondria, where the citric acid cycle occurs as the final common pathway of oxidative metabolism.

An arterial blood gas sample is drawn in a critically ill patient suffering from bowel ischemia. The study reveals that he has developed an anion-gap metabolic acidosis. The acid contributing to his acidosis is formed through a metabolic reaction occurring in which region of the cell? A. Cytoplasm B. Golgi apparatus C. Mitochondrial intermembranous space D. Mitochondrial matrix 1E. Smooth endoplasmic reticulum

*The answer is D.* The body's major energy source for gluconeogenesis is fatty acids, which are oxidized to acetyl-CoA, at which point acetyl-CoA enters the TCA cycle to produce ATP. Acetyl-CoA activates pyruvate carboxylase (and inhibits pyruvate dehydrogenase), a key gluconeogenic enzyme. Acetyl-CoA does not regulate any of the other enzymes listed as potential answers (PEPCK is transcriptionally regulated by CREB; Fructose-1,6-bisphosphatase is inhibited by fructose-2,6-bisphosphate; glucose-6-phosphatase is regulated by a regulatory protein; and pyruvate kinase has both allosteric and covalent controls in the liver, but none involve acetyl-CoA).

An important product of the oxidation of the body's major energy source to provide energy for gluconeogenesis regulates which of the following key gluconeogenic enzymes? (A) PEPCK (B) Fructose-1,6-bisphosphatase (C) Glucose-6-phosphatase (D) Pyruvate carboxylase (E) Pyruvate kinase

*The answer is C.* The cytoplasmic malate dehydrogenase is required in liver as part of the malate/aspartate shuttle in transferring reducing equivalents across the inner mitochondrial membrane. In the absence of such an activity, NADH levels will build up in the cytoplasm (since the electrons cannot be transferred to the mitochondrial matrix) and will lead to the reduction of pyruvate to lactate to regenerate NAD+ for other cytoplasmic reactions. A defect in glucokinase will block glycolysis, with no pyruvate or lactate formation from glucose. The same is true for an inactivating mutation in PFK-1. If pyruvate kinase were defective, PEP would accumulate, which cannot be converted to lactate without forming pyruvate first. A defect in glycerol-3-phosphate dehydrogenase will prevent the glycerol-3-phosphate shuttle from transferring electrons to the mitochondrial matrix, but the liver uses primarily the malate/aspartate shuttle for this activity.

An inactivating mutation in which of the following enzymes would lead to lactic acid accumulation in the liver? (A) Glucokinase (B) Phosphofructokinase-1 (C) Cytoplasmic malate dehydrogenase (D) Pyruvate kinase (E) Glycerol-3-phosphate dehydrogenase

*The answer is C.* After an overnight fast, glycogenolysis and gluconeogenesis act to maintain blood glucose levels in a normal person. Both pathways produce glucose 6-phosphate and require glucose-6-phosphatase to produce free glucose. If the phosphatase is inhibited, blood glucose levels will be lower and liver glycogen stores higher than normal. (When glucose-6-phosphatase is genetically deficient, a similar set of circumstances occurs, and the individual has a glycogen storage disease - von Gierke's disease). Due to the decrease in blood glucose levels, the amount of fatty acids in the circulation will increase because of the activation of hormone-sensitive lipase by protein kinase A.

An individual accidentally ingests an overdose of supplements containing chlorogenic acid, a chemical that inhibits glucose-6-phosphatase. After an overnight fast, this individual, compared with a healthy person, would exhibit which one of the following? (A) An increased rate of gluconeogenesis (B) An increased rate of glycogenolysis (C) An increased level of liver glycogen (D) An increased level of blood glucose (E) A decreased level of serum fatty acids

*The answer is D.* The patient is hypoglycemic because of deficient release of gluconeogenic amino acid precursors from muscle (low urea and glutamine, alanine and glucagon challenge tests). These results plus normal lactate and hyperketonemia eliminate deficiencies in glycogenolysis, gluconeogenesis, and lipolysis as possibilities; defective muscle glycogenolysis would not produce hypoglycemia.

An underweight 4-year-old boy presents semicomatose in the emergency room at 10 A.M. Plasma glucose, urea, and glutamine are abnormally low; acetoacetate is elevated; and lactate is normal. He is admitted to the ICU, where an increase in blood glucose was achieved by controlled infusion of glucagon or alanine. Which metabolic pathway is most likely deficient in this child? A. Hepatic gluconeogenesis B. Skeletal muscle glycogenolysis C. Adipose tissue lipolysis D. Skeletal muscle proteolysis E. Hepatic glycogenolysis

*The answer is A.* In order for citrate to be converted to glycogen, the citrate must first be converted to oxaloacetate in the TCA cyclen(which requires the participation of α-ketoglutarate dehydrogenase). From oxaloacetate, PEP carboxykinase will convert this to PEP, which will go through the gluconeogenic pathway up to glucose-6-phosphate. From there, G1P is produced, then UDP-glucose, and finally incorporation of the glucose into glycogen. Pyruvate carboxylase, while being a gluconeogenic enzyme, converts pyruvate to OAA, which is not required in this series of reactions. PFK-1 and pyruvate kinase are irreversible enzymes of glycolysis and are not used in the gluconeogenic pathway. Glucose-6-phosphatase removes the phosphate from G6P, which is not required when glycogen is being synthesized.

An individual has been eating a large number of oranges during the winter months to protect against getting a cold. The excess carbons of citrate can be used to produce glycogen in the liver. Which one of the following liver enzymes is required for this conversion to occur? (A) α-ketoglutarate dehydrogenase (B) Pyruvate carboxylase (C) Pyruvate kinase (D) PFK-1 (E) Glucose-6-phosphatase

*The answer is E.* As red cells age, the activity of G6PD declines. Despite this loss of enzyme activity, normal old red blood cells contain sufficient G6PD activity to generate NADPH and thereby sustain GSH levels in the face of oxidant stress. In contrast, the G6PD variants with hemolysis have much shorter half-lives. The clinical correlate of this age-related enzyme instability is that hemolysis in patients with G6PD A- generally is mild and limited to the older deficient erythrocytes. The anemia is self-limited because the older, vulnerable population of erythrocytes is replaced by younger RBCs with sufficient G6PD activity to withstand an oxidative assault. Although red cell survival remains shortened as long as use of the drug continues, compensation by the erythroid marrow effectively abolishes the anemia in subjects with G6PD A-. The individual's continuing sensitivity to the effects of the drug is revealed by discontinuing the drug for several months to allow the rate of red cell production by the bone marrow to normalize; during this phase, the older red cells are able to survive, and the red cell population is rendered sensitive to drug-induced hemolysis.

An individual was treated with primaquine from day 0 to day 120. Hemolysis occurred immediately after initiation of drug therapy, as indicated by progressive anemia, hemoglobinuria, and reticulocytosis. However, despite continued administration of the drug, the hemolysis spontaneously decreased and red cell survival improved with time. Red cell G6PD activity measured 2 months after termination of therapy was 10% of normal. Which one of the following statements about this patient is correct? A. The patient will continue to be resistant to drug-induced hemolysis after 6 months or longer. B. Erythrocytes in this patient exhibit a longer lifetime than in normal individuals. C. During the period of peak hemolysis, erythrocytes from the patient will show no G6PD activity. D. The intracellular concentration of NADPH in the patient's erythrocytes is greater than normal. E. The patient shows increased bone marrow erythropoiesis.

*The answer is E.* Under the conditions of a 48-h fast, the liver is exporting glucose, and glycolysis will be inhibited. PFK-1 activity is reduced due to a reduction of fructose-2,6-bisphosphate levels, brought about by glucagon-induced phosphorylation of PFK-2, which activates PFK-2 phosphatase activity, which converts fructose-2,6-bisphosphate to fructose6-phosphate. Pyruvate kinase activity, in the liver, is also reduced by phosphorylation by protein kinase A (which is activated by glucagon). As blood glucose levels have dropped during the fast, and the liver is exporting glucose, the concentration of glucose in the hepatocyte is not sufficient for glucokinase (which has a high Km) to phosphorylate glucose. Glucokinase is not regulated by phosphorylation.

An individual with a BMI of 34 was advised by the physician to eat less and exercise more. The patient took this advice to an extreme, and has not eaten for 48 h. Which of the following best describes the patient's activity and phosphorylation state of the following key liver enzymes?

*The answer is C.* The patient described in the vignette has characteristic features of galactosemia. Classic galactosemia is caused by impaired galactose-1 phosphate metabolism Lactose degradation by the intestinal disaccharidase lactase leads to the formation of galactose and glucose; this enzyme is defective in lactose deficiency. Galactose is then phosphorylated to galactose-1 -phosphate by the enzyme galactokinase. Galactose-1-phosphate is then converted to glucose-1-phosphate by epimerization. This reaction requires the transfer of uridine diphosphate (UDP) from UDP-glucose catalyzed by galactose-1-phosphate uridyltransferase (GALT) generating UDP galactose and glucose-1-phosphate. UDP-galactose-4 epimerase. In breast tissue. α -1 ,4 glycosidic linkage between glucose and galactose is formed resulting in galactosyl-1,4-glucose (lactose or milk sugar). Galactosemia can result from defects in any of the three enzymes involved in galactose metabolism, however, the most common form of galactosemia, classic galactosemia, occurs from a deficiency of galactose-1-phosphate uridyltransferase. The clinical features of this illness include vomiting, lethargy and failure to thrive soon after breastfeeding is begun. Galactosemia can result in impaired liver function, hyperchioremic metabolic acidosis, and aminoaciduria. Dietary restriction of lactose results in improvement in symptoms. Excess galactose in patients with galactosamia is converted to galactitol by galactose reductase or to galactonic acid by galactose oxidase. While galactonic acid can be metabolized by the HMP shunt, galactitol accumulates within the cells. Untreated galactosemia typically culminates in irreversible eye and liver damage. Galactokinase deficiency typically causes less severe manifestations with cataract being the most common manifestation. *Educational Objective:* Classic galactosemia results from a deficiency of galactose-1-phosphate uridyltransferase, this defect is the most common cause of galactosemia. The clinical features of this illness include vomiting, lethargy and failure to thrive soon after breastfeeding is begun.

An infant born to 22-year-old female experiences lethargy, vomiting and jaundice soon after birth. The infant is placed on a galactose-free formula and shows gradual improvement. Which of the following steps in galactose metabolism is most likely impaired in this patient?

*The answer is B.* Carbohydrates are classified as monosaccharides, disaccharides and polysaccharides. Disaccharides are composed of to monosaccharide molecules and polysaccharides consist of a large number of monosaccharide molecules. Monosaccharides are sub classified according to number of carbons they contain. Monosaccharides with 5 (ribulose, ribose) and 6 (glucose, fructose galactose and mannose) carbons are of major importance in mammals. Aldolases (subcategorized as A, B and C) are responsible for metabolizing fructose 6-bisphosphate to glyceraldehyde-3-phosphate and dihydro acetone phosphate (DHAP). Aldolases B also metabolize fructose-1-phosphate, a product of fructose metabolism to glyceraldehyde and DHAP. Aldolase B deficiency is a life-threatening disorder that can be treated by laminating dietary fructose. Sucrose consists of a molecule of fructose linked to a molecule of glucose. *Educational Objective:* Aldolase B participates in fructose metabolism, and a deficiency of this enzyme results in fructose intolerance. This disease is characterized by hypoglycemia following fructose ingestion due to phosphate trapping in fructose-6- phosphate. Fructose and sucrose, a glucose-fructose disaccharide, should be removed from diet.

An infant who is apparently healthy at birth is diagnosed with aldolase B deficiency two weeks later. Which of the following should be removed from this patient's diet? A. Lactose B. Sucrose C. Maltose D. Amylose E. Cellulose

*The answer is C.* Glycogen phosphorylase shortens the glycogen chain by cleaving the alpha 1,4 glycosidic linkages between glucose residues by simple phosphorylation. This occurs until four residues are remaining on the end of a glycogen polymer before a 1 ,6 glycosidic branch point. These glucose residues remaining before a branch point are referred to "limit dextrins". The debrancher enzyme acts on the glycogen polymer at this point. Debrancher enzyme contains two enzymatic activities. The first enzymatic action is a transferase action that transfers the outer three residues of the four- glucose residues left by glycogen phosphorylase on the 1, 6 chains and transfers these three residues to the 1,4 chain. The second action is an amylo-1 6-glucosidase action that cleaves the 1 6 glucosidic bond at the branch point to liberate a free glucose molecule. Glycogen phosphorylase can then resume cleaving alpha 1, 4 glycosidic linkages leading to the formation of glucose-1-phosphate, which is then converted to glucose-6-phosphate by the enzyme phosphoglucomutase. In debranching enzyme deficiency (Con disease), the clinical presentation includes hypoglycemia, hypertriglyceridemia, lactic acidosis, and hepatomegaly. These manifestations are common with other glycogen storage diseases; however, debranching enzyme can be differentiated from other glycogen storage diseases by demonstrating accumulation of abnormally short outer dextrin-like structures in the cytosol of hepatocytes and muscle cells with an absence of fatty infiltration of liver on histopathology. Debranching enzyme deficiency can affect both liver and muscle cells.

An infant with growth retardation, hepatomegaly and hypoglycemia demonstrate hepatic fibrosis without fat accumulation on liver biopsy. There is also accumulation of small chain dextrin-like material within the cytosol of the hepatocytes. Which of the following enzymes is most likely deficient in this patient? A. Glucose-6-phophatase B. Glycogen phosphorylase C. Debranching enzyme D. Amylo-a-(1, 6)-glucosidase E. Pyruvate kinase

*The answer is C.* The disease represented is Von Gierke disease, which is a deficiency in glucose-6-phosphatase. This results in the accumulation of glucose 6-phosphate, which will activate glycogen synthase, resulting in an increase in hepatic glycogen.

An inherited disorder of carbohydrate metabolism is characterized by an abnormally increased concentration of hepatic glycogen with normal structure and no detectable increase in serum glucose concentration after oral administration of fructose. These two observations suggest that the disease is a result of the absence of which of the following enzymes? (A) Fructokinase (B) Glucokinase (C) Glucose-6-phosphatase (D) Phosphoglucomutase (E) UDP glucose

*The answer is C.* There is an increased concentration of hepatic glycolysis with normal structure, which indicates that this may be a glycogen storage disease that particularly affects the liver. If there is a deficiency in glucose 6-phosphatase, there will be an accumulation of glucose 6-phosphate will prevent glucose from entering the blood; this leads to low blood glucose levels.

An inherited disorder of carbohydrate metabolism is characterized by an abnormally increased concentration of hepatic glycogen with normal structure and no detectable increase in serum glucose concentration after oral administration of fructose. These two observations suggest that the disease is a result of the absence of which of the following enzymes? (A) Fructokinase (B) Glucokinase (C) Glucose-6-phosphatase (D) Phosphoglucomutase (E) UDP glucose

*The answer is C.* Citrate is produced by citrate synthase from acetyl CoA and oxaloacetate. This reaction takes place in the mitochondria, but citrate can move freely from the mitochondria into the cytosol using the citrate shuttle. When the citric acid cycle slows down, citrate accumulates. In the cytosol, it acts as a negative allosteric regulator of phosphofructokinase I, the enzyme that catalyzes the committed step of glycolysis.

An investigator is measuring the activity of various enzymes involved in reactions of intermediary metabolism. The activity of one of the enzymes is greatly decreased compared to reference values. The buffer used for the enzyme assay contains sodium citrate. The activity of which of the following enzymes will most likely be directly affected by the use of citrate? A. Fructose 2,6-bisphosphatase B. Isocitrate dehdyrogenase C. Phosphofructokinase I D. Pyruvate carboxylase E. 6-phosphogluconate dehydrogenase

*The answer is C.* This patient has glucose-6-phosphate dehdyrogenase deficiency, which is common in those of Mediterranean descent. This disease is often triggered by the use of certain drugs, such as sulfamethoxazole, or fava beans, which can cause oxidative stress through the production of reactive oxygen species. In an attempt to rid cells of these molecules, glutathione peroxidase will convert hydrogen peroxide into water. However, this is inhibited because of the lack of NADPH production, which helps reform reduced glutathione from oxidized glutathione. As a result, oxidized glutathione cannot assist in the detoxification of hydrogen peroxide.

An otherwise healthy 20-year-old woman of Mediterranean descent is given sulfamethoxazole to treat a bladder infection. Three days after beginning the antibiotic regimen, the patient has moderately severe jaundice and dark urine. Pain with urination and a low-grade fever have resolved. Her hematocrit is 20%. Substantial numbers of erythrocytes contain Heinz bodies. Her condition worsens until day 6 of antibiotic therapy, when it begins to resolve. Symptoms are completely gone by day 9 of continued antibiotic therapy. Which of the following conditions is the most likely explanation for these findings? (A) Aplastic anemia (B) Generalized cytochrome-b5 reductase deficiency (C) Glucose-6-phosphate dehydrogenase deficiency (D) Pyruvate kinase deficiency (E) Systemic infection cured by antibiotic therapy

*The answer is D.* Under anaerobic conditions, the NADH generated by the glyceraldehyde-3-phosphate dehydrogenase step accumulates. Normally, the NADH would transfer its electrons to mitochondrial NAD+, and the electrons would be donated to the electron transfer chain. However, in the absence of oxygen the electron transfer chain is not functioning. Thus, as NADH accumulates in the cytoplasm, the levels of NAD+ decrease to the point that there would be insufficient NAD+ available to allow the glyceraldehyde-3- phosphate dehydrogenase reaction to proceed, thereby inhibiting glycolysis. To prevent glycolytic inhibition, lactate dehydrogenase will convert pyruvate to lactate, regenerating NAD+ for use in glycolysis, specifically as a substrate for the glyceraldehyde-3-phosphate dehydrogenase reaction. While hexokinase is inhibited by its product glucose-6-phosphate, this allosteric effect does not explain lactate formation under anaerobic conditions. Similarly, while phosphoglyceromutase does require 2,3-bisphosphoglycerate, anaerobiosis does not increase 2,3-bisphosphoglycerate levels, nor does it alleviate the lack of NAD+ under these conditions. Pyruvate kinase is not inhibited by pyruvate (ATP and alanine are the allosteric inhibitors of this enzyme). AMP is an activator of phosphofructokinase-1; however, this activation does not relate to lactate formation under anaerobic conditions.

Anaerobiosis leads to lactate formation in muscle due to which one of the following? (A) Inhibiting hexokinase by glucose-6-phosphate (B) Providing 2,3-bisphosphoglycerate for the phosphoglyceromutase reaction (C) Inhibiting pyruvate kinase by pyruvate (D) Providing substrate for glyceraldehyde-3-phosphate dehydrogenase (E) Inhibiting phosphofructokinase-1 by AMP

*The answer is A.* Cortisol stimulates transcription of the PEP carboxykinase gene in the liver but not in adipose tissue.

As part of a study to quantify contributors of stress to hyperglycemia and ketosis in diabetes, normal hepatocytes and adipocytes in tissue culture were treated with cortisol and analyzed by Northern blotting using a gene-specific probe. The results of one experiment are shown below. The ³²P-probe used in this experiment most likely binds to a mRNA encoding A. phosphoenolpyruvate carboxykinase B. lipoprotein lipase C. glucokinase D. hormone-sensitive lipase E. acetyl-CoA carboxylase

*The answer is B.* This question relates to renal production of glucose. The fact that 14C-Iabeled glucose falls faster than total glucose in the rats with kidneys (non-nephrectomized) indicates that the kidneys must be producing exogenous glucose that is diluting the 14C-Iabeled glucose. The kidneys produce this glucose through gluconeogenesis and glycogenolysis, but gluconeogenesis makes up about 85% of renal glucose production and glycogenolysis about 15%. Fructose-1,6- bisphosphatase is one of the rate-determining enzymes in renal gluconeogenesis, and is thus the correct answer.

As part of an investigation into glucose metabolism, investigators inject a single dose of 14C-Labeled glucose into two groups of rats. All rats in group 1 have been recently hepatectomized, and all rats in group 2 have been recently hepatectomized and bilaterally nephrectomized. As a hypoglycemic state is induced, concentrations of 14C- Labeled glucose and total serum glucose are monitored. In both groups, 14CIabeled glucose and total serum glucose concentrations fall at measurable rates. In the nephrectomized group, 14C-Labeled glucose and total serum glucose fall at the same rate. However, in the non nephrectomized group, concentrations of 14C-Labeled glucose fall at a faster rate than total serum glucose concentrations. Which enzyme in the kidney is associated with the difference between groups? A. Citrate synthase B. Fructose-1,6-Bisphosphatase C. Glycogen phosphorylase D. Phosphofructokinase E. Pyruvate kinase

*The answer is B.* Cocaine blocks the reuptake of norepinephrine. resulting in tachycardia as well as excessive vasoconstriction of coronary vessels, leading to ischemia and infarction of heart tissue known as contraction hand necrosis. Under these pathologic conditions. myocardial cells switch to anaerobic metabolism and therefore glycolysis becomes the sole source of ATP via substrate-level phosphorylations by phosphoglycerate kinase and pyruvate kinase. Phosphofructokinase-1 (PFK-1) is the rate-limiting enzyme of glycolysis. and its activity would therefore be increased.

At autopsy of a 28-year-old man. the left ventricle of the heart shows classic contraction band necrosis of the myocardium. Cocaine and its metabolites are detected in body fluids. Activity of which of the following enzymes was most likely increased in the patient's myocardial cells prior to his death? A. Phosphoenolpyruvate carboxykinase B. Phosphofructokinase-1 C. Pyruvate dehydrogenase D. Succinate dehydrogenase E. Transketolase

*The answer is A.* Wernicke syndrome manifests with the triad of ophthalmoplegia, ataxia, and confusion. It is lethal in 10-20% of patients. Foci of hemorrhage and necrosis in the mamillary bodies and periaqueductal gray matter are found on autopsy. This condition occurs due to chronic thiamine deficiency, a condition common in patients with alcoholism. Thiamine (vitamin B1) participates in a number of reactions of glucose metabolism. It is a cofactor for the following enzymes: 1. Pyruvate dehydrogenase converts pyruvate (the end-product of glycolysis) into acetyl CoA (which enters the citric acid cycle). 2. α-ketoglutarate dehydrogenase is an enzyme of the citric acid cycle. 3. Transketolase is an enzyme of the hexose monophosphate pathway. It converts pentoses (derived from glucose) to glyceraldehyde 3 P (an intermediary of glycolysis). Thiamine deficiency, therefore, results in decreased glucose utilization, which is especially pronounced in the CNS. If a patient with chronic thiamine deficiency is given a glucose infusion without thiamine supplementation, acute cerebral damage occurs. An increase in erythrocyte transketolase levels after thiamine infusion is diagnostic for thiamine deficiency. (In actual practice, if a patient might be an alcoholic or appears to be very malnourished, presume that the patient is thiamine deficient and give thiamine supplementation with glucose infusion.) *(Choices B and D)* Neither erythrocyte glutathione reductase, nor NAD are used for the diagnosis of thiamine deficiency. *(Choice C)* Erythrocyte glucose-6-phosphate dehydrogenase (G6 PD) catalyzes a rate-limiting step in the pentose phosphate pathway. This pathway is necessary for NADPH production and the function of the erythrocyte antioxidant system. Decreased glucose-6-phosphate dehydrogenase levels cause hemolytic anemia. *(Choice E)* Methylmalonic acid is a product of fatty acid oxidation. It is converted to succinyl CoA by methylmalonyl CoA mutase. This enzyme uses B12 as a coenzyme. Methylmalonic acid levels are increased in vitamin B12 deficiency. *(Choice F)* Protoporphyrin is one of the precursors of heme. An increased erythrocyte protoporphyrin concentration is the hallmark of erythropoietic protoporphyria (EPP): however, this elevation is nonspecific and can be seen in other conditions such as iron- deficient anemia and lead poisoning. *Educational Objective:* Chronic thiamine (B1) deficiency leads to the diminished ability of cerebral cells to utilize glucose. The mechanism is decreased function of the enzymes that use vitamin B1 as a cofactor (pyruvate dehydrogenase, a-ketoglutarate dehydrogenase, and transketolase). Thiamine deficiency can be diagnosed by measuring erythrocyte transketolase activity.

Autopsy of a 61-year-old homeless male who died in the ER shows foci of hemorrhage and necrosis in the mamillary bodies and the gray matter surrounding the third and fourth ventricles. This patient's condition could have been diagnosed early by measuring which of the following? (A) Erythrocyte transketolase activity (B) Erythrocyte glutathione reductase activity (C) Erythrocyte glucose-6-phosphate dehydrogenase activity (D) Serum NAD level (E) Serum methylmalonic acid level (F) Blood protoporphyrin level

*The answer is B.* Hexokinase is inhibited by glucose-6-phosphate and is found in all tissues in the body. Although present to some degree in liver and β-pancreatic cells, its function is negligible in these two tissues.

Blood glucose is tightly regulated after a meal by an enzyme found in muscle tissues. This enzyme catalyzes one of the steps in the pathway to convert glucose to glycogen. In addition, this process is subject to feedback inhibition by glucose-6-phosphate. Which of the following enzymes is being described? A. Glucokinase B. Hexokinase C. Phosphofructokinase-1 D. Pyruvate dehydrogenase E. Pyruvate kinase

*The answer is D.* If the phosphodiesterase that degrades cAMP were inhibited (an effect of caffeine) in the presence of epinephrine, cAMP levels would be elevated. Protein kinase A would become more active in the liver and muscle; pyruvate kinase would become less active in the liver; and glycogen synthase activity would be decreased in both muscle and liver. Phosphorylase activity would be increased in both muscle and liver owing to constant phosphorylation by phosphorylase kinase, which is activated by protein kinase A.

Caffeine, a methyl xanthine, has been added to a variety of cell types. Which one of the following would be expected in various cell types treated with caffeine and epinephrine? (A) Decreased activity of liver protein kinase A (B) Decreased activity of muscle protein kinase A (C) Increased activity of liver pyruvate kinase (D) Decreased activity of liver glycogen synthase (E) Decreased activity of liver glycogen phosphorylase

*The answer is F.* In the absence of insulin, glucagon-stimulated activities predominate. This leads to the activation of protein kinase A, the phosphorylation and inactivation of glycogen synthase, the phosphorylation and activation of phosphorylase kinase, and the phosphorylation and activation of glycogen phosphorylase.

Consider a person with type 1 diabetes who has neglected to take insulin for the past 72 hours and has not eaten much as well. Which of the following best describes the activity level of hepatic enzymes involved in glycogen metabolism under these conditions?

*The answer is C.* Under conditions in which the entry charge of liver cells is high, intermediates of the citric acid cycle are abundant. Citrate, an early intermediate in the cycle, readily diffuses across the inner membrane of mitochondria and out into the cytosol. Citrate allosterically inhibits phosphofructokinase and, conversely, stimulates fructose-1,6-bisphosphatase. Thus, when the energy level of hepatocytes is low and biosynthetic precursors are not abundant, phosphofructokinase is stimulated and glycolysis is favored. When the energy level is high, citrate inhibits phosphofructokinase, stimulates the diphosphatase, and thereby promotes gluconeogenesis.

Citrate has a negative allosteric effect on which one of the following enzymes? (A) Pyruvate kinase (B) Acetyl CoA carboxylase (C) Phosphofructokinase (D) Fatty acid synthetase (E) Enolase

*The answer is A.* Vigorously contracting muscle shows an increased formation of lactate and an increased rate of pyruvate oxidation compared with resting skeletal muscle. The levels of AMP and NADH increase, whereas change in the concentration of fructose 2,6-bisphosphate is not a key regulatory factor in skeletal muscle.

Compared with the resting state, vigorously contracting skeletal muscle shows: A. an increased conversion of pyruvate to lactate. B. decreased oxidation of pyruvate to CO2 and water. C. a decreased NADH/NAD+ ratio. D. a decreased concentration of AMP. E. decreased levels of fructose 2,6-bisphosphate.

*The answer is E.* Under the conditions described, DNA synthesis is occurring without any requirement for NADPH (such as fatty acid synthesis). Under these conditions, NADPH levels are high and glucose-6-phosphate dehydrogenase is inactive. The cell requires ribose-5-phosphate, however, for nucleotide biosynthesis, and this is synthesized from fructose-6-phosphate and glyceraldehyde-3-phosphate using the nonoxidative reactions of the pathway. Thus, both transketolase and transaldolase will be active under these conditions. PFK-1 is active as well, as the only way to generate glyceraldehyde-3-phosphate from a sugar precursor is via enzymes of the glycolytic pathway.

Consider an intestinal epithelial cell in S phase, and for which the major, active biosynthetic pathway is nucleotide synthesis. Which one of the following best represents the activity state of a series of key enzymes under these conditions?

*The answer is C.* The glucose in the sports drink will bind to liver glycogen phosphorylase a and inhibit its activity allosterically. Once the insulin signal reaches the liver, phosphorylase a will be converted to the dephosphorylated phosphorylase b by activated phosphatases. There is no allosteric inhibitor for glycogen synthase I, or protein phosphatase 1 (which is regulated by protein inhibitor 1). Adenylate kinase is not regulated allosterically, and there is no allosteric inhibitor of phosphorylase kinase a (the non-phosphorylated form can be activated by calcium).

Consider the case of an athlete who has just completed a work out. At this point, the athlete consumes a sports drink, which contains a large amount of glucose, which enters the circulation. Glycogen degradation is inhibited in the liver under these conditions, prior to insulin release, due to allosteric inhibition of which of the following enzymes? (A) Glycogen synthase I (B) Phosphorylase kinase a (C) Phosphorylase a (D) Protein phosphatase 1 (E) Adenylate kinase

*The answer is B.* Both PFK-1 and LDH participate in extrahepatic anaerobic glycolysis, but only PFK-1 is regulated by allosteric effectors.

During a myocardial infarction, the oxygen supply to an area of the heart is dramatically reduced, forcing the cardiac myocytes to switch to anaerobic metabolism. Under these conditions, which of the following enzymes would be activated by increasing intracellular AMP? A. Succinate dehydrogenase B. Phosphofructokinase-1 C. Glucokinase D. Pyruvate dehydrogenase E. Lactate dehydrogenase

*The answer is C.* The activity of regulatory enzymes such as fructose-1,6 bisphosphatase, hexokinase, phosphofructokinase 1, and pyruvate kinase are frequently controlled by binding allosteric effectors. These allosteric enzymes usually exhibit sigmoidal kinetics. Lactate dehydrogenase is not controlled by allosteric effectors and therefore would be expected to exhibit Michaelis-Menten kinetics.

During an extended period of exercise, the enzymes involved in the glycolytic pathway in muscle tissue are actively breaking down glucose to provide the muscle energy. The liver, to maintain blood glucose levels, is synthesizing glucose via the gluconeogenic pathway. Which of the following enzymes involved in these pathways would be most likely to exhibit Michaelis- Menten kinetics (i.e. have a hyperbolic curve when plotting substrate concentration versus velocity of the reaction)? (A) Fructose-1,6-bisphosphatase (B) Hexokinase (C) Lactate dehydrogenase (D) Phosphofructokinase 1 (E) Pyruvate kinase

*The answer is A.* The data show that protein tyrosine phosphatase 1B (PTP 1B) decreases the tyrosine kinase activity of the insulin receptor. 1. Insulin binding to the insulin receptor activates the cytoplasmic tyrosine kinase domain causing autophosphorylation of the receptor. These phosphotyrosine residues initiate the signal transmission pathways associated with insulin 2. PTP 1B opposes insulin signaling by dephosphorylating the phosphotyrosine residues of the insulin receptor, blocking further signal transmission 3. A mouse lacking PTP 1B activity would show an increased hepatic response to insulin

During an investigational study, a genetically engineered mouse completely lacking protein tyrosine phosphatase 1B (PTP 1B) activity (a PTP 1B knockout) is produced, Liver cells are isolated from the animal, and insulin and glucagon are added to the liver cell preparation. The graph shows the results of PTP 16 activity and tyrosine Kinase activity of the insulin receptor in liver cells from a normal mouse. Compared with a normal mouse, the PTP to knockout mouse would most likely show an increase in activity of which of the following enzymes? A. Acetyl-CoA carboxylase B. Hexokinase C. Glycogen phosphorylase D. Glycogen phosphorylase kinase E. Protein kinase A

*The answer is E.* In most biochemical pathways. only a few enzymatic reactions are under regulatory control. These often occur either at the beginning of pathways or at pathway branch points. The pyruvate dehydrogenase (PDH) complex controls the link between glycolysis and the citric acid cycle, and decarboxylates pyruvate (the end product of glycolysis} with production of NADH and acetyl-coA (the substrate for the citric acid cycle). POH in mitochondria is huge, visible as dots on electron microscopy, because it is composed of many copies of at least five separate enzymes. Three of these (POH dihydrolipoyl transacetylase. and dihydrolipoate dehydrogenase) catalyze the actual biochemical reactions. The other two. POH kinase and phosphatase, add and remove phosphate groups from the POH complex, respectively. The less active form of PDH is phosphorylated; the more active font of POH is not. In exercise, high ADP and pyruvate concentrations inhibit POH kinase. while increased Ca2+ stimulates POH phosphatase. Both effects produce more of the dephosphorylated, more active font of PDH.

During an investigational study. two muscle needle biopsy specimens are obtained. The first is taken while a healthy volunteer is at rest; the second is taken after the volunteer performs 30 minutes of aerobic exercise on a stationary bicycle. The activity of the muscle pyruvate dehydrogenase complexes is found to be much higher in the second specimen. Which of the following biochemical changes is most likely to produce this effect? A. Decreased ADP B. Decreased intracellular Ca2+ C. Increased acetyl-CoA D. Increased NADH/NAD+ E. Increased pyruvate concentration

*The answer is E.* The answer is E. NADH decreases during exercise to generate energy for the exercise (if it were increased, it would inhibit the cycle and slow it down); thus the NADH/NAD+ ratio is decreased; and the lack of NADH activates flux through isocitrate dehydrogenase, α-ketoglutarate dehydrogenase, and malate dehydrogenase. Isocitrate dehydrogenase is inhibited by NADH; so, answer A is not correct. Fumarase is not regulated, thus answer B is incorrect. The four-carbon intermediates of the cycle are regenerated during each turn of the cycle, so their concentrations do not decrease (thus, C is incorrect). Product inhibition of citrate synthase would slow the cycle and not generate more energy; hence, D is incorrect.

During exercise, stimulation of the TCA cycle results principally from which of the following? (A) Allosteric activation of isocitrate dehydrogenase by increased NADH (B) Allosteric activation of fumarase by increased ADP (C) A rapid decrease in the concentration of four-carbon intermediates (D) Product inhibition of citrate synthase (E) Stimulation of the flux through a number of enzymes by a decreased NADH/NAD+ ratio

*The answer is D.* Erythrocytes with fatty acyl hydroperoxides have "stiff" membranes, which cannot squeeze through the smaller diameters of the capillaries. Membrane permeability is increased and the erythrocytes rupture rather than pass through the capillaries.

During your third year, while on your hematology rounds, you are assigned a patient who has had an episode of hemolysis. Based on his recent consumption of fava bean soup and his Middle Eastern heritage you suspect a fava bean induced episode of active oxygen radicals may have caused the hemolysis. You know that fava beans contain divicine, isouramil, and aglycones that are redox active, permitting the transfer of electrons from one compound to another. When oxygen radical species are formed in individuals with deficient G6PDH activity, membrane lipids can undergo hydroperoxide formation. Which of the following statements is least accurate about erythrocytes in your patient? (A) Red blood cell membrane elasticity is reduced (B) Red blood cells are more easily hemolyzed (C) Red blood cells are more permeable (D) Red blood cells are easily traverse capillary beds (E) Red blood cells are less able to reduce fatty acyl hydroperoxides to hydroxyl forms

1. B 2. C 3. E 4. A 5. D A defect in the synthesis or processing of collagen will lead to a variety of diseases, of which Ehlers-Danlos syndrome is one (osteogenesis imperfecta is another). Parkinson's disease is due to low levels of dopamine in the nervous system. In the initial stages of Parkinson's disease, giving dihydroxyphenylalanine (DOPA) can reduce the severity of the symptoms, as DOPA can be decarboxylated to form dopamine. DOPA can easily enter the brain, whereas dopamine cannot. Tay-Sachs disease results from an inability to degrade GM2, a ganglioside (it is also a sphingolipid). The enzyme missing is β-hexosaminidase. McArdle's disease is due to a defective muscle glycogen phosphorylase, such that the muscle cannot generate glucose from glycogen, leading to exercise intolerance. Maple syrup urine disease is due to the lack of branched-chain α-keto acid dehydrogenase activity, a necessary step in the metabolism of the branched-chain amino acids (leucine, isoleucine, and valine).

Each condition listed below (the numbered question) can be caused by a problem with the metabolism of a particular compound. Match the condition with the appropriate compound. An answer (lettered choices) may be used once, more than once, or not at all. 1. Ehlers-Danlos syndrome 2. Parkinson's disease 3. Tay-Sachs disease 4. McArdle's disease 5. Maple syrup urine disease (A) Glycogen (B) Collagen (C) Dopamine (D) Valine (E) A sphingolipid

*The answer is B.* In many populations, a majority of adults are deficient in lactase and hence intolerant to the lactose in milk. In all populations, at least some adults have lactase deficiency. Since virtually all children are able to digest lactose, this deficiency obviously develops in adulthood. In lactase-deficient adults, lactose accumulates in the small intestine because no transports exist for the disaccharide. An outflow of water into the gut owing to the osmotic effect of the milk sugar causes the clinical symptoms. Steatorrhea, or fatty stools, is caused by unabsorbed fat, which can occur following a fatty meal in persons with a deficiency of lipoprotein lipase. Sialidase deficiency causes accumulation of sialic acid-containing proteoglycans and neurodegeneration.

Following a fad diet meal of skim milk and yogurt, an adult female patient experiences abdominal distention, nausea, cramping, and pain followed by a watery diarrhea. This set of symptoms is observed each time the meal is consumed. A likely diagnosis is a. Steatorrhea b. Lactase deficiency c. Maltose deficiency d. Sialidase deficiency e. Lipoprotein lipase deficiency

*The answer is C.* Only PFK-2 will be insulin-activated in the postprandial period.

Following an early-morning run, a 29-year-old man consumes an all-American breakfast consisting of cereal, eggs, bacon, sausage, pancakes with maple syrup, doughnuts, and coffee with cream and sugar. Which of the following proteins will most likely be activated in his liver after breakfast? A. Cytoplasmic PEP carboxykinase B. Plasma membrane GLUT-4 transporter C. Cytoplasmic phosphofructokinase-2 D. Mitochondrial carnitine transporter E. Cytoplasmic glycogen phosphorylase

1. *The answer is A.* Needed for transport of fatty acids across the mitochondrial inner membrane. 2. *The answer is A.* Mitochondrial inner membrane. 3. *The answer is B.* Mitochondrial matrix (ketogenesis). 4. *The answer is C.* CAT-1 (CPT-1) and fatty acyl synthetase are among the few enzymes associated with the outer mitochondrial membrane.

For each item listed below, select the appropriate location from the drawing shown above. 1. Carnitine shuttle 2. F0F1 ATP synthase 3. HMG-CoA lyase 4. Carnitine palmitoyltransferase-1

*The answer is B.* Pyruvate carboxylase catalyzes the conversion of pyruvate to oxaloacetate in gluconeogenesis: pyruvate + HCO3− + ATP → oxaloacetate + ADP + Pi In order for pyruvate carboxylase to be ready to function, it requires biotin, Mg++, and Mn++. It is allosterically activated by acetyl CoA. The biotin is not carboxylated until acetyl CoA binds the enzyme. By this means, high levels of acetyl CoA signal the need for more oxaloacetate. When ATP levels are high, the oxaloacetate is consumed in gluconeogenesis. When ATP levels are low, the oxaloacetate enters the citric acid cycle. Gluconeogenesis only occurs in the liver and kidneys.

Fully activated pyruvate carboxylase depends upon the presence of a. Malate and niacin b. Acetyl CoA and biotin c. Acetyl CoA and thiamine pyrophosphate d. Oxaloacetate and biotin e. Oxaloacetate and niacin

*The answer is B.* This mouse model has a defect in the GLUT family of glucose transporters (GLUT 1-5). These transporters are responsible for cellular uptake in many cell types. However, enterocytes and nephrons utilize SGLT glucose transporters that acquire glucose through cotransport with sodium ions.

Glucose is transported into human cells by two different families of membrane-associated carrier proteins: the glucose transporter facilitators (GLUT) and the sodium-coupled glucose transporters (SGLT). A knockout mouse model is created with a defect in GLUT family of glucose transporters (GLUT1- 5), Which of the mouse's cells will still be able to take up glucose normally? A. Adipocytes B. Enterocytes C. Erythrocytes D. Hepatocytes E. Myocytes F. Pancreatic beta cells

*The answer is C.* Other tissues have alternate pathways to generate NADPH inside mitochondria that are not altered by the deficiency.

G6PD deficiency affects all cells in the body, but RBCs are particularly vulnerable. Which of the following is the most plausible explanation for this? (A) Hemoglobin carries oxygen and, thus, is less susceptible to reactive oxygen molecules. (B) Other tissues have alternate pathways to generate NADPH in the cytoplasm that can be utilized by the cell. (C) Other tissues have alternate pathways to generate NADPH inside mitochondria that are not altered by the deficiency. (D) Hemoglobin carries oxygen and, thus, is more susceptible to reactive oxygen molecules.

*The answer is D.* Glucose-6-phosphatase is not present in muscles and adipose tissues. This is because glucose 6-phosphate (1) cannot escape out of the cell and (2) can readily transform into glycogen. However, in the hepatocytes (liver cells) and kidney cells, glucose 6-phosphate is converted into glucose in the lumen of the endoplasmic reticulum. This is because these organs regulate the blood glucose levels so they have to be able to form glucose because only the glucose form can actually leave the cell. Glucose-6-phosphate is trapped inside the cell.

Glucose-6-phosphatase is not present in (A) Liver and kidneys (B) Kidneys and muscles (C) Kidneys and adipose tissue (D) Muscles and adipose tissue

*The answer is E.* Given the enzymes present, only the nonoxidative reactions of the HMP shunt would take place. In order for the nonoxidative reactions to occur, the glucose-6-phosphate (G6P, labeled in the 6th position with 14C) must pass through glycolysis t produce fructose-6-phosphate (F6P, labeled in the 6th position) and glyceraldehyde-3-phosphate (labeled in the 3rd position). Transketolase will allow these two compounds to exchange carbons, which would generate erythrose-4-phosphate (E4P, labeled in the 4th position) and xylulose-5-phosphate (X5P, labeled in the 5th position). The X5P can then go to ribulose-5-phosphate (Ru5P) and ribose-5-phosphate (R5P), labeled in the fifth positions. The E4P (labeled in the 4th position) can react with another molecule of F6P (labeled in the 6th position) using transaldolase to generate sedoheptulose 7-phosphate (Se7P, labeled in the 7th position) and glyceraldehyde-3-phosphate (G3P), labeled in the 3rd position. Transketolase will then convert the Se7P and G3P to R5P and X5P, both labeled in the 5th positions.

Glucose-6-phosphate labeled in carbon 6 with 14C was added to a test tube with the enzymes phosphohexose isomerase, PFK-1, aldolase, transketolase, and transaldolase. ATP was also added to the test tube. At equilibrium, in which position would the radioactive label be found in the newly produced ribose-5-phosphate? (A) 1 (B) 2 (C) 3 (D) 4 (E) 5

*The answer is C.* Glycogen synthetase is an enzyme that transfers glucosyl moieties from UDP-glucose to a glycogen polymer primer. In plants, ADP-glucose plays a part similar to that of UDP-glucose in animals. The enzyme exists in two forms: an active, dephosphorylated form and an inactive, phosphorylated form. It is inactivated by phosphorylation of a specific serine residue. Glycogen breakdown, not synthesis, is positively affected by increased calcium levels.

Glycogen synthase, the enzyme involved in the biosynthesis of glycogen, may (A) Be activated by the phosphorylation of a specific serine residue (B) Be activated by increased calcium levels (C) Be more specifically defined as UDP-glucose-glycogen glucosyl transferase (D) Synthesize glycogen without a polymer primer (E) Employ UDP-D-glucose as a glucosyl donor in both plants and animals

The answer is C. Glycogen synthetase is an enzyme that transfers glucosyl moieties from UDP-glucose to a glycogen polymer primer. In plants, ADP-glucose plays a part similar to that of UDP-glucose in animals. The enzyme exists in two forms: an active, dephosphorylated form and an inactive, phosphorylated form. It is inactivated by phosphorylation of a specific serine residue. Glycogen breakdown, not synthesis, is positively affected by increased calcium levels.

Glycogen synthetase, the enzyme involved in the biosynthesis of glycogen, may (A) Be activated by the phosphorylation of a specific serine residue (B) Be activated by increased calcium levels (C) Be more specifically defined as UDP-glucose-glycogen glucosyl transferase (D) Synthesize glycogen without a polymer primer (E) Employ UDP-D-glucose as a glucosyl donor in both plants and animals

*The answer is C.* Glycogenolysis requires a dual action enzyme of α-1,6-glucosidase and transferase. Transferase catalyzes the shift of three glucose residues from one branch to the other. This process leaves a glucose that is linked via a α-1,6-glycosidic bond α-1,6-glucosidase then cleaves the α-1,6-glycosidic bond to release a free glucose and forms the modified glycogen. The branched glycogen is then converted into a linear polymer. This process includes the inactivation of glycogen synthase and the activation of glycogen phosphorylase, in order to produce glucose-1-phosphate from glycogen. This process does not use ATP. The enzyme phosphorylase kinase is what activates glycogen phosphorylase by phosphorylation.

Glycogenolysis is best described by which of the following statements? (A) It involves enzymes cleaving β-1,4 glycosidic linkage (B) Requires activation of glycogen synthase (C) Requires a dual action enzyme α-1,6-glucosidase and transferase (D) Uses ATP to produce glucose-1-phosphate (E) Requires inaction of phosphorylase kinase

*The answer is D.* 4 molecules of NAD+ are reduced as one molecule of pyruvate is metabolized through one turn of the Krebs cycle. Pyruvate oxidation involves the reduction of NAD+ to NADH. This can also be seen in the reaction of isocitrate to α-ketoglutarate (catalyzed by isocitrate dehydrogenase), α-ketoglutarate to succinyl-CoA (catalyzed by α-ketoglutarate dehydrogenase), and malate to oxaloacetate (catalyzed by malate dehydrogenase).

How many molecules of NAD+ are reduced as one molecule of pyruvate is metabolized through one turn of the Krebs cycle? (A) 1 (B) 2 (C) 3 (D) 4 (E) 5

*The answer is B.* Aerobic glycolysis can be defined as the oxidative conversion of glucose to two molecules of pyruvate. In the process, two molecules of ATP and two molecules of NADH are produced. Since reducing equivalents from the two molecules of NADH produced in the cytoplasm must be transported into the mitochondrion for oxidation, it is not known how many ATP molecules are produced. On the assumption that two ATP molecules are formed per molecule of NADH oxidized via the glycerol phosphate shuttle, the ATP yield in aerobic glycolysis can be calculated as six ATP molecules per mole of glucose utilized.

If all potential sources of ATP production are taken into account, the net number of ATP molecules formed per molecule of glucose in aerobic glycolysis is a. 2 b. 6 c. 18 d. 36 e. 54

*The answer is E.* One active PKA can activate in 1 s 100 molecules of phosphorylase kinase. Each phosphorylase kinase can, in 1 s activate 100 molecules of glycogen phosphorylase (so at this point we have 100 times 100 active molecules of phosphorylase, or 10,000 active phosphorylase molecules). Each active phosphorylase molecule can release 100 glucose residues per second from glycogen, and since there are 10,000 active phosphorylase molecules, 1,000,000 molecules of glucose are released per second once a single molecule of PKA has been activated. This is an example of cascade amplification, in which an increase in activity of just one molecule at the top of the cascade can result in a large response further down the cascade.

If the turnover number of all enzymes involved in glycogen metabolic regulation and activity is 100 reactions per second, how many glucose molecules could be removed from glycogen in 1 s upon activation of one molecule of protein kinase A (PKA)? (A) 100 (B) 1,000 (C) 10,000 (D) 100,000 (E) 1,000,000

*The answer is E.* In glucose 6-phosphate dehydrogenase deficiency, the associated hemolytic anemia is ultimately due to the decreased concentration of NADPH. In the red blood cells, NADPH is usually utilized by the enzyme glutathione reductase, which produces reductive glutathione. This version of glutathione protects the red blood cells from oxidative damage. Without this, the RBCs lyse.

In glucose 6-phosphate dehydrogenase deficiency, the associated hemolytic anemia is ultimately due to what changes from normal in red cells? (A) decreased formation of ATP by mitochondria. (B) decreased ability to carry out glycolysis. (C) increased leakage of potassium into the cells. (D) an intrinsic defect in membrane structure. (E) decreased concentrations of NADPH.

*The answer is A.* After ingestion of glucose the insulin:glucagon ratio increases, the cAMP phosphodiesterase is activated, cAMP levels drop, and protein kinase A is inactivated. This leads to the activation of glycogen synthase by PP-1. The ratio of phosphorylase a to phosphorylase b is decreased by PP-1 as well, thus glycogen degradation decreases. Red blood cells continue to use glucose and form lactate at their normal rate as glucose is the sole energy source for such cells. CREB is also inactivated under these conditions, thereby reducing the levels of PEPCK (via transcriptional regulation) within the cell.

In a glucose tolerance test, an individual in the basal metabolic state ingests a large amount of glucose. If an individual displays a normal response, this ingestion results in which one of the following? (A) Enhanced glycogen synthase activity in liver (B) An increased ratio of phosphorylase a to phosphorylase b in the liver (C) An increased rate of lactate formation by erythrocytes (D) Inhibition of PP-1 activity in the liver (E) Increased activity of CREB

*The answer is D.* In a healthy individual, fasting eventually leads to gluconeogenesis. Synthesis of glucose from pyruvate or lactate requires seven reactions of glycolysis that are reversible. However, there are three glycolytic reactions that are irreversible and must be circumvented by four alternative reactions that favor the synthesis of glucose. The first step in gluconeogenesis is the irreversible conversion of pyruvate to phosphoenolpyruvate (PEP). To convert pyruvate to PEP, pyruvate is first carboxylated to oxaloacetate by pyruvate carboxylase, the first enzyme in gluconeogenesis and the only one of the choice produced in the mitochondria. Pyruvate carboxylase is activated by acetyi-CoA and requires biotin and adenosine t riphosphate as cofactors.

In a healthy individual, fasting leads to increased lipolysis, increased accumulation of fatty acids, and increased production of acetyl -coenzyme A (acetyl -CoA). Acetyl -CoA is an allosteric activator of an important mitochondrial enzyme that catalyzes an irreversible step in gluconeogenesis. What is the product of the reaction catalyzed by this enzyme? A. Fructose-1,6-bisphosphate B. Glucose C. Glucose-6-phosphate D. Oxaloacetate E. Phosphoenolpyruvate F. Pyruvate

*The answer is D.*Ca2+ released from the sarcoplasmic reticulum during exercise binds to the calmodulin subunit of phosphorylase kinase, thereby allosterically activating the "b" form of this enzyme. The other choices are not caused by an elevation of cytosolic calcium.

In contracting skeletal muscle, a sudden elevation of the cytosolic Ca2+ concentration will result in: A. activation of cAMP-dependent protein kinase A. B. dissociation of cAMP-dependent protein kinase A into catalytic and regulatory subunits. C. inactivation of phosphorylase kinase caused by the action of protein phosphatase-1. D. direct activation of phosphorylase kinase b. E. direct activation of glycogen phosphorylase b. F. conversion of cAMP to AMP by phosphodiesterase.

*The answer is E.* In glucose 6-phosphate dehydrogenase deficiency, the associated hemolytic anemia is ultimately due to decreased concentrations of NADPH. Glucose 6-phosphate dehydrogenase catalyzes the reaction that converts glucose 6-phosphate into 6-phosphogluconolactone and NADPH. NADPH is utilized by glutathione reductase to maintain levels of glutathione in cells, which helps protect red blood cells against oxidative damage. As a result, this leads to hemolysis - the spontaneous destruction of red blood cells.

In glucose 6-phosphate dehydrogenase deficiency, the associated hemolytic anemia is ultimately due to what changes from normal in red cells? (A) decreased formation of ATP by mitochondria. (B) decreased ability to carry out glycolysis. (C) increased leakage of potassium into the cells. (D) an intrinsic defect in membrane structure. (E) decreased concentrations of NADPH.

*The answer is D.* The exposure of tissues to chronic hypoxia makes them rely more on anaerobic metabolism for the generation of energy as ATP and other high-energy phosphates. Most tissues except for red blood cells can metabolize glucose under anaerobic or aerobic conditions (red blood cells do not have mitochondria for electron transport and must rely on other tissues to generate glucose back from lactate). In most tissues, a switch from aerobic to anaerobic metabolism greatly increases glucose utilization and decreases energy production. (A reduction of glucose utilization under anaerobic conditions in bacteria is known as the Pasteur effect after its discoverer). Under aerobic conditions, the cell can produce a net gain in moles of ATP formed per mole of glucose utilized that can be as high as 18 times that produced under anaerobic conditions. Thus the cell generates more energy and requires less glucose under aerobic conditions. Such increased ATP concentrations, together with the release of citrate from the citric acid cycle under aerobic conditions, allosterically inhibit the key regulatory enzyme of the glycolytic pathway, phosphofructokinase. Decreased phosphofructokinase activity decreases metabolism of glucose by glycolysis.

In lung diseases such as emphysema or chronic bronchitis, there is chronic hypoxia that is particularly obvious in vascular tissues such as the lips or nail beds (cyanosis). Poorly perfused areas exposed to chronic hypoxia have decreased metabolic energy for tissue maintenance and repair. An important reason for this is a. Increased hexokinase activity owing to increased oxidative phosphorylation b. Increased ethanol formation from pyruvate on changing from anaerobic to aerobic metabolism c. Increased glucose utilization via the pentose phosphate pathway on changing from anaerobic to aerobic metabolism d. Decreased ATP generation and increased glucose utilization on changing from aerobic to anaerobic metabolism e. Decreased respiratory quotient on changing from carbohydrate to fat as the major metabolic fuel

*The answer is B.* Cellular damage is directly related to decreased ability of the cell to regenerate reduced glutathione, for which large amounts of NADPH are needed, and RBC have no other means of generating NADPH. Catalytic properties of G6PD in liver and RBCs are very similar. The pentose phosphate pathway does not generate ATP. RBCs do not have glucose 6-phosphatase.

In male patients who are hemizygous for glucose 6-phosphate dehydrogenase (G6PD) deficiency, pathophysiologic consequences are more apparent in erythrocytes (RBC) than in other cells, such as in the liver. Which one of the following provides the most reasonable explanation for this different response by these individual tissue types? A. Excess glucose 6-phosphate in the liver, but not in RBCs, can be channeled to glycogen, thus averting cellular damage. B. Liver cells, in contrast to RBCs, have alternative mechanisms for supplying the NADPH required for keeping metabolic and cellular integrity. C. Glucose 6-phosphatase activity in RBCs decreases the level of glucose 6-phosphate, thus resulting in cell damage. This does not happen in the hepatocyte. D. Because RBCs do not have mitochondria, production of ATP required to keep cell integrity depends exclusively on the routing of glucose 6-phosphate to the pentose phosphate pathway. E. The catalytic properties of the liver enzyme are significantly different than those of the RBC enzyme.

*The answer is C.* When glycogen produces glucose via the action of glycogen phosphorylase, glucose-1-phosphate is produced. As this is converted to two molecules of pyruvate, four moles of ATP are generated and one is utilized at the PFK-1 step for the net production of three moles of ATP. Two moles of NADH are also produced, but those are utilized by lactate dehydrogenase to reduce pyruvate to lactate (anaerobic conditions) such that NAD+ can be regenerated for the glyceraldehyde-3-phosphate dehydrogenase step. A small amount of free glucose will be released from glycogen by the debranching enzyme (about 5% of the total); for that glucose, the net yield is two moles of ATP (since hexokinase has to phosphorylate the free glucose to glucose-6-phosphate), but since the majority of glucose released is in the form of glucose-1-phosphate, three moles of ATP is the better answer.

In muscle, under anaerobic conditions, the net synthesis of ATP starting from one mole of glucose derived from muscle glycogen is which one of the following? (A) 1 mole of ATP (B) 2 moles of ATP (C) 3 moles of ATP (D) 4 moles of ATP (E) 5 moles of ATP

*The answer is C.* Glutathione is essential for red cell integrity and is maintained in its functional (reduced) form by NADPH-dependent glutathione reductase. The NADPH is generated by the oxidative portion of pentose phosphate pathway. Individuals with a deficiency of the initiating and regulated enzyme (G6PD) of this pathway have a decreased ability to generate NADPH, and thus a decreased ability to keep glutathione functional. When treated with an oxidant drug such as primaquine, some patients with G6PD deficiency develop a hemolytic anemia. Primaquine does not affect glucose 6-phosphate levels. NAD+ is neither produced by the pentose phosphate pathway nor used as a coenzyme by glutathione reductase. Ribulose 5-phosphate, another product of the oxidative portion of the pentose phosphate pathway, can be isomerized to ribose 5-phosphate, but a deficiency in either does not cause hemolysis.

In preparation for a trip to an area of India where malaria is endemic, a young man is given primaquine prophylactically. Soon thereafter, he develops a hemolytic condition. The most likely cause of the hemolysis is a less-than-normal level of which of the following? A. Glucose 6-phosphate B. Oxidized form of NAD C. Reduced form of glutathione D. Ribose 5-phosphate E. Ribulose 5-phosphate

*The answer is A.* Glutathione reductase will utilize NADPH and reduce oxidized glutathione to reduced glutathione, generating NADP+. If Glutathione reductase is superactive, NADP+ levels accumulate, which activates glucose-6-phosphate dehydrogenase. This will lead to NADPH production via the oxidative reactions of the HMP shunt, along with ribulose-5-phosphate (Ru5P). The Ru5P will lead to increased ribose-5-phosphate production, increased 5′-phosphoribosyl 1′-pyrophosphate (PRPP) production, and increased 5′-phosphoribosyl 1′-amine levels. This eventually leads to increased purine production, in excess of what is required. The excess purines are converted to uric acid, and excess uric acid will lead to gout. A superactive glutathione reductase will not lead to an alteration in the activities of transketolase or transaldolase.

Individuals with a superactive glutathione reductase will develop gout. This occurs due to which of the following? (A) Activation of glucose-6-phosphate dehydrogenase (B) Inhibition of glucose-6-phosphate dehydrogenase (C) Activation of transketolase (D) Activation of transaldolase (E) Inhibition of transketolase

*The answer is C.* Infection increases the generation of oxidants. It turns out that immune cells like macrophages increase the generation of reactive oxygen species to fight infection, but this will consume more reduced glutathione according to the figure below and the limited amount of G6PD cannot renew the pool of reduced glutathione. Allosteric activators of G6PD and glutathione reductase would be expected to counter oxidative stress, not worsen it.

Infection most commonly causes an exacerbation of anemia in patients with incomplete G6PD deficiency (i.e. these patients have some normal G6PD). What is a plausible explanation for this? (A) Infection decreases the generation of oxidants (B) Infection is an allosteric activator of glutathione reductase (C) Infection increases the generation of oxidants (D) Infection is an allosteric activator of the limited G6PD present

*The answer is C.* In glycolysis. glucose is metabolized to pyruvate and lactate. Under aerobic conditions, the dominant product in most tissues is pyruvate, which is converted to acetyl CoA to enter the TCA cycle. When oxygen is depleted, such as in exercising muscle, the dominant product is lactate. This is referred to as anaerobic glycolysis. During glycolysis, glyceraldehyde-3-phosphate is converted to 1,3 bisphosphoglycerate by the enzyme glyceraldehyde-3-phosphate dehydrogenase. This enzyme uses NAD-dependent oxidation, and in this reaction NAD+ is converted into NADH. NAD+ is present in limited amounts in most cells, and it must be regenerated from NADH for glycolysis to continue. Under aerobic conditions, NAD+ is converted to NADH in the TCA cycle and NADH is then converted back to NAD+ in the electron transport chain as the energy in NADH is utilizes to synthesize ATP. In anaerobic glycolysis, NAD+ is regenerated from NADH when pyruvate is converted to lactate by lactate dehydrogenase. In strenuously exercising muscle, glycolysis can be inhibited by limited regeneration of NAD+ from NADH (choice C).

Inhibition of lactate dehydrogenase in strenuously exercising skeletal muscles would eventually lead to an inhibition of glycolysis due to intracellular depletion of which of the following substances? A. AMP B. FADH2 C. NAD+ D. Carnitine E. Pyruvate F. Citrate

*The answer is D.* The expression of α-lactalbumin (protein B) is increased by the hormone, prolactin. UDP galactose is the form used by the galactosyltransferase (protein A). Protein A is also involved in the synthesis of the amino sugar, N acetyllactosamine. Protein B increases the affinity of protein A for glucose, and so decreases the Km.

Lactose synthesis is essential in the production of milk by mammary glands. In lactose synthesis: A. galactose from galactose 1-P is transferred to glucose by galactosyltransferase (protein A), generating lactose. B. protein A is used exclusively in the synthesis of lactose. C. α-lactalbumin (protein B) regulates the sugar specificity of protein A by increasing its Km for glucose. D. protein B expression is stimulated by prolactin.

1. *The answer is B.* Acetyl-CoA activates pyruvate carboxylase and gluconeogenesis during fasting. 2. *The answer is F.* Fumarate 3. *The answer is E.* Succinyl-CoA. 4. *The answer is D.* Glutamate is produced by B6-dependent transamination of α-ketoglutarate.

Link the following to the letters in the cycle. 1. Obligate activator of hepatic pyruvate carboxylase in the postabsorptive state. 2. Product formed by argininosuccinate lyase during urea synthesis. 3. Substrate and energy source for synthesis of δ-aminolevulinate in the heme pathway. 4. Converted to glutamate in a reaction requiring the coenzyme form of pyridoxine (B6)

*The answer is C.* Muscle phosphorylase deficiency leads to a glycogen storage disease [McArdle's disease (232600)] and, in young adults, an inability to do strenuous physical work because of muscular cramps resulting from ischemia. The compromised phosphorylation of muscle glycogen characteristic of McArdle's disease compels the muscles to rely on auxiliary energy sources such as free fatty acids and ambient glucose.

McArdle's disease causes muscle cramps and muscle fatigue with increased muscle glycogen. Which of the following enzymes is deficient? (A) Hepatic hexokinase (B) Muscle glycogen synthase (C) Muscle phosphorylase (D) Muscle hexokinase (E) Muscle debranching enzyme

*The answer is A.* The sources of NADPH for synthesis of fatty acids are the pentose phosphate pathway and cytosolic malate formed during the transfer of acetyl groups to the cytosol as citrate. The enzyme citrate lyase splits citrate into acetyl CoA and oxaloacetate. The oxaloacetate is reduced to malate by NADH. NADP-linked malate enzyme catalyzes the oxidative decarboxylation of malate to pyruvate and carbon dioxide. Thus, the diffusion of excess citrate from the mitochondria to the cytoplasm of cells not only provides acetyl CoA for synthesis of fatty acids but NADPH as well. One NADPH is produced for each acetyl CoA produced. However, most of the NADPHs needed for synthesis of fatty acids are derived from the pentose phosphate pathway. For this reason, adipose tissue has an extremely active pentose phosphate pathway.

Most of the reducing equivalents utilized for synthesis of fatty acids can be generated from (A) The pentose phosphate pathway (B) Glycolysis (C) The citric acid cycle (D) Mitochondrial malate dehydrogenase (E) Citrate lyase

*The answer is E.* Mechanical disruption of the intestinal epithelial cells (as brought about by acute viral gastroenteritis) leads to a loss of cell surface enzymes, but lactase is the most severely affected, as it is present at the lowest levels on these cells. While glucoamylase is also lost, its activity is in vast excess of what is required and its partial loss does not affect its ability to hydrolyze glucose-glucose linkages (it does not hydrolyze lactose). A lack of lactase means that the lactose in the diet passes undigested through the small intestine to the large intestine where the bacterial flora metabolize the lactose, producing gases and acids that disrupt the osmotic balance between the lumen of the bowel and the cells lining it. This leads to water secretion by the cells into the lumen of the bowel, resulting in diarrhea. Lactose is not directly transported by intestinal epithelial cells (its components, glucose and galactose, are, after hydrolysis of β-1,4 linkage between the two sugars), and a mechanical disruption of intestinal cells does not alter transcription of galactokinase and fructokinase.

Mr Smith recently had a bout of five days of severe nausea, vomiting, low-grade fever, and diarrhea. This condition had afflicted a number of people in Mr Smith's office. After recovering from this disorder, Mr Smith found that he could no longer drink milk before going to bed as he became flatulent, his stomach hurt, and he would develop diarrhea. If he did not drink milk, these conditions did not occur. He had never experienced these problems before the affliction. A possible explanation for Mr Smith's problem is which one of the following? (A) Mechanical disruption of the intestinal epithelial cells, leading to reduced transcription of the galactokinase gene (B) Mechanical disruption of the intestinal epithelial cells, leading to reduced transcription of the fructokinase gene (C) Mechanical disruption of the intestinal epithelial cells, leading to loss of lactose transport into the cells (D) Mechanical disruption of the intestinal epithelial cells, leading to loss of the glucoamylase complex from their surface (E) Mechanical disruption of the intestinal epithelial cells, leading to loss of lactase from their surface

*The answer is C.* In mucopolysaccharidoses, synthesis of proteoglycans is unaffected, both in terms of the structure and the amount of material synthesized. The diseases are caused by a deficiency of one of the lysosomal, hydrolytic enzymes responsible for the degradation of glycosaminoglycans (not the core protein).

Mucopolysaccharidoses are inherited lysosomal storage diseases. They are caused by: A. an increased rate of synthesis of the carbohydrate component of proteoglycans. B. the synthesis of polysaccharides with an altered structure. C. defects in the degradation of the GAGs in proteoglycans. D. the synthesis of abnormally small amounts of protein cores. E. an insufficient amount of proteolytic enzymes.

*The answer is C.* Muscle contraction is caused by the release of calcium from the sarcoplasmic reticulum following nervous stimulation. In addition to stimulating contraction, the calcium released from the sarcoplasmic reticulum binds to a calmodulin subunit on phosphorylase kinase. This activates phosphorylase kinase, converting it from the D form to the A form. The activated phosphorylase then breaks down glycogen and provides glucose for energy metabolism during exercise. In this way, muscle contraction and glucose production from glycogen are coordinated by the transient increase of cytoplasmic calcium levels during muscle contraction.

Nervous stimulation of skeletal muscle causes the release of calcium from sarcoplasmic reticulum and leads to muscle contraction. Simultaneously, the increased calcium concentration causes (A) A dramatic rise in cyclic AMP levels (B) Inactivation of glycogen phosphorylase (C) Activation of phosphorylase kinase (D) Activation of cyclic AMP phosphodiesterase (E) Activation of protein phosphatase

*The answer is D.* Fructose Is obtained in the diet primarily from fruits and food sweetners such as table sugar (sucrose) and high-fructose com syrup. Fructose is absorbed in the proximal intestine through the GLUT5 fructose transporter. It is normally phosphorylated by fructokinase in the liver, yielding fructose-1-phosphate, which is converted by aldolase B to dihydroxyacetone phosphate (DHAP) and glyceraldehyde (Choice C). Glyceraldehyde and DHAP can be converted to glyceraldehyde-3 phosphate, which can than be metabolized in the glycolytic pathway. Fructokinase deficiency (essential fructosuria) is an asymptomatic, autosomal recessive disorder that causes dietary fructose to be excreted unchanged in the urine. In fructokinase deficiency, hexokinase takes over the role of fructose metabolism, converting dietary fructose into fructose-6-phosphate. Fructose-6-phosphate can be metabolized in the glycolytic pathway or converted to glucose-6-phosphate or glucose-1-phosphate, which can be used in the pentose phosphate pathway or for glycogen synthesis, respectively.

Nutrition researchers investigating the relationship between fructose consumption and cardiovascular disease conduct a prospective cohort study on a population of randomly selected young adults. Study participants undergo semiannual measurement of waist circumference, blood pressure, and serum cholesterol and triglyceride concentrations. Dietary fructose consumption is assessed through the use of questionnaires end by measuring urinary fructose excretion. A 23-year old man enrolled in the study is found to excrete large amounts of fructose in his urine compared to other study participants despite maintaining a moderate fructose intake. Further evaluation shows a hereditary defect in fructose metabolism, but he is asymptomatic and has no other medical problems. This patient most likely remains able to metabolize fructose due to the compensatory activity of which of the following enzymes? A. Aldolase B B. Aldose reductase C. Fructokinase D. Hexokinase E. UDP-galactose-4-epimerase

*The answer is A.* Fructokinase converts fructose to fructose-1-phosphate in the first step in fructose metabolism. Fructokinase is deficient in patients with essential fructosuria, also known as benign fructosuria. Patients with this genetic condition are asymptomatic with the incidental finding of a reducing substance in the urine. Unlike the fructose intolerance seen in the first child, in essential fructosuria, fructose is not metabolized in the hepatocytes by fructokinase and does not deplete the cell's supply of phosphate. The fructose does not become trapped within the cell and is then excreted in urine

On the same day, one pediatrician sees two children with a hereditary defect fructose metabolism. The physician reminds the first set of parents to maintain their child on a low-fructose, low-sucrose diet. However~ he tells the second set of parents that their child is unlikely to experience any symptoms from the disorder, despite the fact that fructose was detected in the child's blood and urine. What action describes the function of the enzyme that is deficient in the second ch ild? A. Converts fructose to fructose-1-phosphate B. Converts fructose- 1-phosphate to dihydroxyacetone-phosphate C. Converts fructose- 1-phosphate to glyceraldehyde D. Converts glyceraldehyde to glycera ldehyde-3-phosphate E. Converts glyceraldehyde-3-phosphate to 1,3-bisphosphoglycerate

*The answer is C.* There are 5 pairs of electrons that can move through the electron transport after the pyruvate is metabolized through one turn of the Krebs cycle. Pyruvate oxidation involves the reduction of NAD+ to NADH. This can also be seen in the reaction of isocitrate to α-ketoglutarate (catalyzed by isocitrate dehydrogenase), α-ketoglutarate to succinyl-CoA (catalyzed by α-ketoglutarate dehydrogenase), and malate to oxaloacetate (catalyzed by malate dehydrogenase). The reaction of succinate to fumarate (catalyzed by succinate dehydrogenase) involves the reduction of FAD to FADH2.

Oxidation of one molecule of pyruvate by the combined action of the pyruvate dehydrogenase complex and the enzymes of the citric acid cycle is thought to result in the generation of how many pairs of electrons which can move through the electron transport chain? (A) 3 (B) 4 (C) 5 (D) 6 (E) 7

*The answer is E.* The net result of the citric acid cycle oxidation of acetyl CoA is shown below: acetyl CoA + FAD + 3 NAD+ + GDP + 2 H2O + Pi → 2 CO2 + CoA + FADH2 + 3 NADH + GTP + 2 H+ The cycle produces reducing equivalents (NADH, FADH2) and carbon dioxide directly, but not ATP. The reducing equivalents are used to produce ATP by mitochondrial oxidative phosphorylation. Two carbon atoms enter the cycle as acetyl CoA, with an immediate loss of CoA as citrate is formed from oxaloacetate. Two carbon atoms leave the cycle at the level of isocitrate dehydrogenase and α-ketoglutarate dehydrogenase. The two carbons that leave as CO2 are actually not the original acetyl CoA carbons. Two NAD+ molecules are reduced by isocitrate dehydrogenase and then α- ketoglutarate dehydrogenase. One FAD is reduced during the oxidation of succinate, and one NAD+ is reduced when malate is oxidized. GTP is formed from GDP by utilization of the high-energy thioester linkage of succinyl CoA.

Oxidative degradation of acetyl coenzyme A (CoA) in the citric acid cycle gives a net yield of which of the following chemicals? (A) Flavin adenine dinucleotide (FAD+) (B) Nicotinamide adenine dinucleotide (NAD+) (C) Adenosine triphosphate (ATP) (D) Guanosine diphosphate (GDP) (E) Carbon dioxide (CO2)

*The answer is B.* Pantothenic acid, also called coacetylase, is a component of coenzyme A (CoA). Acetyl CoA is the activated form of acetate employed in acetylation reactions, including the citric acid cycle and lipid and cholesterol metabolism. A deficiency of pantothenic acid would limit CoA and have deadly consequences in mammals. However, since it is common in foodstuffs, there is little evidence of pantothenic acid deficiency in humans.

Pantothenic acid is a constituent of the coenzyme involved in a. Decarboxylation b. Acetylation c. Dehydrogenation d. Reduction e. Oxidation

*The answer is D.* Ethanol oxidation to acetic acid (via acetaldehyde) generates large amounts of NADH. As liver glycogen stores have been depleted within 36 h of the fast, gluconeogenesis is required to maintain blood glucose levels. The major precursors for gluconeogenesis are glycerol, lactate, and amino acids (which give rise to pyruvate or TCA cycle precursors, which generate oxaloacetate). Because of the high NADH/NAD+ ratio (due to the ethanol metabolism), pyruvate destined for gluconeogenesis is shunted to lactate in order to regenerate NAD+ to allow alcohol metabolism to continue. Similarly, oxaloacetate is shunted to malate, also to regenerate NAD+ for ethanol metabolism. Glycerol, which is converted to glycerol-3-phosphate, cannot go to dihydroxyacetone phosphate due to the high NADH levels in the liver. Thus, the high NADH/NAD+ ratiodiverts gluconeogenic precursors from entering gluconeogenesis, and the liver has trouble maintaining adequate blood glucose levels. Liver glycogen stores have been depleted within the first 36 h of the fast, but glycogen regulation is not affected by the NADH/NAD+ ratio. Under conditions in which the liver is exporting glucose (glucagon administration, for example), liver glycolysis is inhibited by covalent modification of key regulatory enzymes, not the NADH/NAD+ ratio.

Paramedics bring a patient to the emergency department because he was found unconscious in an alley by passers by. The man was unshaven and dishevelled, and appeared to be about 40 years old. Blood alcohol levels were found to be 0.25% and blood glucose levels 32 mg/dL. IV glucose was initiated, and this enabled the man to regain consciousness, although he was still inebriated. While conscious, a history revealed that the man was a chronic alcoholic, and as far as he could remember, he had been only drinking for the past 2 weeks, with nothing to eat. Analysis of liver enzyme levels in his blood revealed normal readings. Assuming that his liver is still functioning normally, why is this patient hypoglycemic? A) Liver glycogen stores were depleted by the high NAD+/NADH ratio (B) Liver glycogen stores were depleted by the high NADH/NAD+ ratio (C) The high NAD+/NADH ratio impaired gluconeogenesis (D) The high NADH/NAD+ ratio impaired gluconeogenesis (E) The high NAD+ /NADH ratio impaired glycolysis

*The answer is D.* Glycogen synthase D (the inactive, phosphorylated form) can be allosterically activated by glucose-6-phosphate binding to the enzyme. Glucose-6-phosphate will inhibit the AMP-stimulation of muscle phosphorylase b, but does not have any allosteric effect on the other enzymes listed (PFK-1, glucose-6-phosphatase, or GLUT4 transporters) as answer choices for this problem.

Patients with von Gierke disease display hepatomegaly. Glycogen content in the liver is increased, relative to normal, due to which of the following effects of glucose-6-phosphate in these patients? (A) Inhibition of phosphorylase a (B) Stimulation of phosphorylase b (C) Inhibition of glycogen synthase I (D) Stimulation of glycogen synthase D (E) Inhibition of glycogen phosphorylase kinase

*The answer A.* . Fluctuating levels of sugars and sugar alcohols in the lens can cause fluctuating visual acuity. With high blood glucose, there would be increased levels of sorbitol in the lens. The lens does not contain mitochondria and cannot use the TCA cycle/electron transport chain to generate energy. Galactitol causes the same problems as sorbitol, but galactitol is derived from galactose, whereas sorbitol is produced from glucose. The patient has high glucose levels, so galactitol would not be expected to accumulate in the lens. Macular degeneration affects the retina, but in this case, it is the lens that is the affected tissue. Reducing fructose levels in the lens would reduce sorbitol levels, which would ease the visual acuity problem, not make it occur.

Patients with diabetes frequently report changing visual acuities when their glucose levels are chronically high. Which of the following could explain the fluctuating acuity with high blood glucose levels? (A) Increased sorbitol in the lens (B) Decreased fructose in the lens (C) Increased oxidative phosphorylation in the lens (D) Macular degeneration (E) Increased galactitol in the lens

*The answer is C.* There are two ATP-binding sites on the enzyme; a catalytic site with a high affinity for ATP, and a regulatory site with low ATP affinity. The enzyme is shut down only at high ATP concentrations when the energy charge is high.

Phosphofructokinase catalyzes the phosphorylation of fructose 6-phosphate to form fructose 1,6-bisphosphate. ATP is the phosphate donor and also inhibits phosphofructokinase. How can ATP act as a substrate and as a feedback inhibitor? (A) The formation of fructose 1,6-bisphosphate is forced toward completion because standard free energy for the reaction is negative. (B) There are two ATP-binding sites on the enzyme; a catalytic site with low affinity and a regulatory binding site with high affinity. At high ATP levels, the enzyme is inhibited fully. (B) There are two ATP-binding sites on the enzyme; a catalytic site with a high affinity for ATP, and a regulatory site with low ATP affinity. The enzyme is shut down only at high ATP concentrations when the energy charge is high. (C) There is a single ATP-binding site; binding ATP lowers the probability of reaction for a short time, even after ATP is unbound. Thus a high ATP concentration inhibits the reaction. (D) None of the above

*The answer is E.* Of the 5 major facilitative glucose transporters (GLUTs), only GLUT-4 Is responsive to insulin. GLUT-4 is expressed predominantly in muscle cells and adipocytes. In the absence of insulin, GLUT-4 is sequestered in the cytoplasm. However, as insulin concentrations rise, the receptors translocate to the plasma membrane, facilitating glucose transport down the concentration gradient into the cell. In the absence of insulin, muscle cells and adipocytes are impermeable to glucose. In contrast to GLUT-4, GLUT-1, 2, 3, and 5 are always present on the plasma membrane and constitutively transport glucose (insulin independent) (Choice A) Adipocytes and skeletal muscle cells have insulin responsive GLUT-4 transporters. The corresponding graph would show increasing GLUT expression on both cells with rising insulin concentrations. (Choices B, C, and D) Hepatocytes, pyramidal neurons, renal tubular cells, pancreatic B-cells, and intestinal epithelial cells all express insulin-independent GLUTs. The corresponding graph would show unchanging GLUT expression on both cells with using insulin concentrations. *Educational Objective:* GLUT-4 is expressed primarily in muscle cells and adipocytes and is the major glucose transporter that is responsive to insulin.

Researchers are investigating the relationship between glucose transporters and insulin concentration in various cells and tissues. Data are collected and plotted on the graph below. The graph shows the number of glucose transporters found on the surface of 2 types of cells (circles versus triangles) compared to serum insulin concentration. Which of the following cell types are most likely represented by the circles and triangles, respectively? A. Adipocytes and ekeletd muscle cells B. Hepatocytes and cortical pyramidal cells C. Hepatocytes and renal tubular cells D. Pancreatic l3-cells and intestinal epithelial cells E. Skeletalmuscle cells and renal tubular cells

*The answer is C.* Clinitest is a nonspecific test that produces a change in color if urine is positive for reducing substances, including reducing sugars (glucose, fructose, galactose, xylulose, lactose), amino acids, ascorbic acid, and certain drugs and drug metabolites. Because sucrose is not a reducing sugar, it is not detected by Clinitest. Glucose oxidase method will not detect increased levels of galactose or other sugars in urine. It is therefore important that a copper reduction method be used as a screening test. In those instances when the copper method is positive and the glucose oxidase method is negative, glucosuria is ruled out.

Routine examination of the urine of an asymptomatic pediatric patient showed a positive reaction with Clinitest (a copper reduction method of detecting reducing sugars), but a negative reaction with the glucose oxidase test. Which one of the following sugars is least likely to be present (assuming a single elevated saccharide)? A. Lactose B. Fructose C. Sucrose D. Xylulose E. Galactose

*The answer is A.* When phosphorylated, heart PFK-2 is activated to produce more fructose-2,6-bisphosphate to stimulate heart PFK-1 and to increase the glycolytic rate of the heart. Phosphorylation of heart PFK-2 can be accomplished through the AMP-activated protein kinase (when the heart is having trouble generating energy) or in response to insulin (indicating that high levels of glucose are available for use). Phosphorylation of heart PFK-2 does not affect its transcription or turnover rate, and also does not affect the degradation of fructose-1,6-bisphosphate.

Skeletal muscle PFK-2 is not regulated by phosphorylation, but heart muscle PFK-2 is. In the heart, phosphorylation of PFK-2 leads to what effect? (A) Enhanced production of fructose-2,6-bisphosphate (B) Reduced production of fructose-2,6-bisphosphate (C) Degradation of fructose-1,6-bisphosphate (D) Increased turnover of PFK-2 (E) Increased transcription of PFK-2

*The answer is B.* Glycolysis can occur in aerobic or anaerobic conditions. In aerobic environments, the NADH created during the conversion of glyceraldehyde 3-phosphate to 1,3-bisphosphoglycerate is regenerated to NAD through oxidation within the mitochondrial electron transport chain. The amount of NAD in cells is limited, therefore, regeneration of NAD from NADH is essential. In anaerobic conditions (and in erythrocytes under aerobic conditions), pyruvate cannot be oxidatively decarboxylated to acetyl CoA. Instead, pyruvate is converted to lactate by the enzyme lactate dehydrogenase. The conversion of pyruvate to lactate also serves to re-oxides NADH to NAD in the absence of oxygen. Erythrocytes are unique cells because they do not have mitochondria and cannot generate energy from the citric acid cycle. Glycolysis is the major pathway used by RBCs to produce energy. 2,3 bisphosphoglycerate (BPG) is generated as a byproduct of glycolysis from 1,3-BPG by the enzyme bisphosphoglycerate mutase (producing no ATP). It is catabolized to 3-phosphoglycerate by bisphosphoglycerate phosphatase (also producing no ATP). During normal glycolysis, 1,3-BPG is converted to 3-phosphoglycerate by the enzyme phosphoglycerate kinase, which does produce ATP in the process. By generating 2,3-BPG rather than proceeding with regular glycolysls, RBCs sacrifice the net ATP gem achieved in normal glycolysis. The major function of RBCs is to carry hemoglobin-bound oxygen from the lungs to the peripheral tissues, and 2.3-BPG is a very important regulator of oxygen-binding to hemoglobin. increased 2,3-BPG concentrations within erythrocytes enable increased oxygen delivery in the peripheral tissues in the presence of lower blood oxygen concentrations because 2,3-BPG allosterically decreases the affinity of hemoglobin for oxygen. The conversion of 1,3-BPG to 2,3-BPG is increased in hypoxia and chronic anemia,

Some cells have an alternative glycolytic pathway that produces no net ATP. These cells sometimes divert glycolytic intermediates into a eaction catalyzed by bisphosphoglycerate mutase rather than using those intermediates to produce energy. Which of the following cells are most likely to utilize this alternative pathway? A. Hepatocytes B. Erythrocytes C. Skeletal muscle cells D. Adipocytes E. Neurons

*The answer is E.* NADPH, the reduced form of NADP, is produced mainly when glucose is metabolized via the hexose monophosphate shunt (HMP shunt, pentose phosphate pathway). NADPH, in contrast to NAD, is primarily used as reducing equivalent in the cytosol rather than as a source of energy for ATP synthesis in the electron transport chain. The HMP shunt is also responsible for the production of ribose 5-phosphate needed for the synthesis of nucleotides. From one molecule of glucose. the HMP shunt forms a five-carbon sugar, two molecules of NADPH and CO2. The HMP shunt consists of two different types of reactions: oxidative (irreversible) end nonoxidative (reversible). All reactions of the HMP shunt occur exclusively in the cytoplasm. The primary enzymes involved in the non-oxidative steps of the HMP shunt are transaldolase and transketotase. Transketolase transfers two carbon groups between substrates of the HMP shunt and requires thiamine pyrophosphate as a cofactor, and transaldolase transfers three-carbon groups between substrates of the HMP shunt All cells can synthesize ribose from the glycolysis intermediates fructose 6-phosphate and glycereldehyde 3-phosphate with the help of transketolase and transaldolase even if the oxidative reactions of the HMP pathway are not active in those cells.

Some human cells are unable to generate NADPH from glucose metabolism but are able to synthesize ribose from fructose-6 phosphate. Which of the following enzymes is essential for the latter finding? A. Glucose-6-phosphate dehydrogenase B. Glutathione reductase C. Enolase D. Aconitase E. Transketoiase

*The answer is D.* By starting with glyceraldehyde 3-phosphate, the energy-requiring steps of glycolysis are bypassed. Thus, as glyceraldehyde 3-phosphate is converted to pyruvate, two molecules of ATP will be produced (at the phosphoglycerate kinase and pyruvate kinase steps) and one molecule of NADH will be produced (at the glyceraldehyde-3-phosphate dehydrogenase step).

Starting with glyceraldehyde 3-phosphate and synthesizing one molecule of pyruvate, the net yield of ATP and NADH would be which of the following? (A) 1 ATP, 1 NADH (B) 1 ATP, 2 NADH (C) 1 ATP, 4 NADH (D) 2 ATP, 1 NADH (E) 2 ATP, 2 NADH (F) 2 ATP, 4 NADH (G) 3 ATP, 1 NADH (H) 3 ATP, 2 NADH (I) 3 ATP, 4 NADH

*The answer is D.* Fluoride inhibits the glycolytic enzyme enolase, which catalyzes the dehydration of 2-phosphoglycerate to phosphoenolpyruvate. Thus, 2-phosphoglycerate accumulates under these conditions.

Streptococcus mutans, found in dental plaque, produces acids from the metabolism of carbohydrates. Topical fluoride treatment in the dental office can slow the production of acids, resulting in the accumulation of which metabolite? (A) Glucose-6-phosphate (B) Fructose-1,6-bisphosphate (C) Glyceraldehyde-3-phosphate (D) 2-phosphoglycerate (E) Phosphoenolpyruvate

*The answer is A.* Succinate dehydrogenase is the only TCA cycle enzyme located in the inner mitochondrial membrane. The other enzymes are in the mitochondrial matrix. Answer B is incorrect because succinate dehydrogenase is not regulated by NADH. Answer C is incorrect because α-ketoglutarate dehydrogenase also contains a bound FAD (the difference is that the FADH2 in α-ketoglutarate dehydrogenase donates its electrons to NAD+, whereas the FADH2 in succinate dehydrogenase donates its electrons directly to the electron-transfer chain). Answer D is incorrect because both succinate dehydrogenase and aconitase have Fe-S centers. Answer E is incorrect because succinate dehydrogenase is not regulated by a kinase. Kinases regulate enzymes by phosphorylation (e.g., the regulation of pyruvate dehydrogenase occurs through reversible phosphorylation).

Succinate dehydrogenase differs from all other enzymes in the TCA cycle in that it is the only enzyme that displays which of the following characteristics? (A) It is embedded in the inner mitochondrial membrane. (B) It is inhibited by NADH. (C) It contains bound FAD. (D) It contains Fe-S centers. (E) It is regulated by a kinase.

*The answer is E.* Under fasting conditions, the liver is exporting glucose, so the pathways of glycogenolysis and gluconeogenesis will be active, while glycolysis will be inhibited (all due to the effects of glucagon and activation of PKA). In glycolysis, PFK-2 is phosphorylated, activating its phosphatase activity, which leads to a reduction in fructose-2,6-bisphosphate levels. This results in a reduction of PFK-1 activity (thus, PFK-1 is not active, but is not phosphorylated). Glycogen degradation has been activated, and synthesis inhibited, via the phosphorylation of glycogen synthase, inactivating the enzyme (thus, glycogen synthase is not active, but is phosphorylated). Phosphorylase kinase has been activated, and phosphorylated, by PKA (so phosphorylase kinase is active, and phosphorylated). Pyruvate dehydrogenase is inactive under these conditions (due to fatty acid oxidation in the mitochondria acetyl-CoA levels and NADH levels are high, which slows down the TCA cycle and inhibits pyruvate dehydrogenase), and it is also phosphorylated by the PDH-kinase, which is activated by NADH.

Ten hours into a fast, in a normal individual, which of the following best represents the activity and phosphorylation state of a number of key enzymes within the liver?

*The answer is A.* The ATP yield from the conversion of glucose to 2 pyruvates under anaerobic conditions can be seen in the following calculation - 1 ATP used in the glucokinase/hexokinase reaction - 1 ATP used in the PFK-1 reaction + 2 ATP produced by substrate level phosphorylation in the phosphoglycerate kinase reaction + 2 ATP produced by substrate level phosphorylation in the pyruvate Kinase reaction Net yield : 2 ATP/glucose Pentavalent arsenic competes with inorganic phosphate as a substrate for the enzyme glyceraldehyde-3-phosphate dehydrogenase, forming a complex that spontaneously hydrolyzes to form 3-phosphoglycerate, By bypassing the synthesis of 1,3-bisphosphoglycerate and its subsequent role in the substrate-level phosphorylation by phosphoglycerate kinase, no ATP or NADH is generated at that step. Thus, there will be a loss of 2 ATP per glucose, reducing the net yield to 0 ATP/glucose. Two NADH/glucose are also produced by anaerobic glycolysis, but without oxygen (as in an infected area of tissue) or without an electron transport chain (as in the erythrocyte); these NADH are used to convert pyruvate to lactate and are not sources of ATP production. In an aerobic tissue with an electron transport chain, the NADH can provide high-energy electrons to make 4 to 6 additional ATP. This ATP production depends on either the malate or the glycerol phosphate shuttle to deliver electrons to the ETC.

The diagram shows the effects of pentavalent arsenic on the metabolism of glyceraldehyde-3-phosphate. As a result of the mechanism shown, anaerobic conversion of 2 glucose molecules to 4 pyruvate molecules will yield a net production of how many molecules of ATP'? A. 0 B. 1 C. 2 D. 3 E. 4

*The answer is A.* For a molecule of glucose-6-phosphate (G6P) to be incorporated into glycogen, the following pathway must be utilized: G6P is converted to glucose-1-phosphate (G1P) via phosphoglucomutase, the G1P reacts with UTP to form UDP-glucose via glucose-1-phosphate uridyl transferase, releasing pyrophosphate. The resultant pyrophosphate is hydrolyzed to two inorganic phosphates, with the loss of one high-energy bond. The UDP-glucose then reacts with glycogen to produce a glycogen chain with one additional sugar, and UDP is released. The overall equation for these steps is: G6P + UTP + glycogen n yields UDP + 2Pi + (glycogen) n+1. These steps are outlined below: Gluose-6-phosphate → Glucose-1-phosphate Glucose-1-phosphate + UTP → UDPglucose + PPi PPi + H2O → 2 Pi UDP-glucose + glycogen n → Glycogen n+1 + UDP UDP + ATP → UTP + ADP Sum: Glucose-6-phosphate + ATP + glycogen n + H2O → glycogen n+1 + ADP + 2Pi

The energy required to store one molecule of glucose-6-phosphate as a portion of glycogen is which of the following? (A) One high-energy bond (B) Two high-energy bonds (C) Three high-energy bonds (D) Four high-energy bonds (E) No high-energy bonds

*The answer is C.* When acetyl-CoA enters the TCA cycle, and is converted to two molecules of carbon dioxide, and oxaloacetate is regenerated, three molecules of NADH are produced, along with one molecule of FADH2 and one substrate-level phosphorylation resulting in the generation of GTP. As each NADH can give rise to 2.5 ATP, and each FADH2 to 1.5 ATP via oxidative phosphorylation, the net yield of high-energy bonds from one acetyl-CoA being oxidized by the cycle is 10 (7.5 from NADH, 1.5 from FADH2, and 1 from GTP). This is shown in the fi gure below.

The energy yield from the complete oxidation of acetyl-CoA to carbon dioxide is which of the following in terms of high-energy bonds formed? (A) 6 (B) 8 (C) 10 (D) 12 (E) 14

*The answer is C.* When acetyl-CoA enters the TCA cycle, and is converted to two molecules of carbon dioxide, and oxaloacetate is regenerated, three molecules of NADH are produced, along with one molecule of FADH2 and one substrate-level phosphorylation resulting in the generation of GTP. As each NADH can give rise to 2.5 ATP, and each FADH2 to 1.5 ATP via oxidative phosphorylation, the net yield of high-energy bonds from one acetyl-CoA being oxidized by the cycle is 10 (7.5 from NADH, 1.5 from FADH2, and 1 from GTP). This is shown in the figure below.

The energy yield from the complete oxidation of acetyl-CoA to carbon dioxide is which of the following in terms of high-energy bonds formed? (A) 6 (B) 8 (C) 10 (D) 12 (E) 14

*The answer is B.* Succinate + NAD+ + FAD → oxaloacetate + NADH + FADH2

The following is the sum of three steps in the citric acid cycle. A + B + FAD + H2O → C + FADH2 + NADH Choose the lettered answer that corresponds to the missing "A", "B", and "C" in the equation.

*The answer is B.* Epinephrine and glucagon both cause increased glycogen degradation in the liver through covalent modification (phosphorylation) of key enzymes of glycogen metabolism. Glycogen phosphorylase is phosphorylated and active ("a" form), whereas glycogen synthase is phosphorylated and inactive ("b" form). cAMP-dependent protein kinase A is active and phosphorylates (and activates) its substrate, phosphorylase kinase. It is phosphorylase kinase a that directly phosphorylates and activates phosphorylase.

The hormones, epinephrine and glucagon have which one of the following effects on glycogen metabolism in the liver? A. The net synthesis of glycogen is increased. B. Glycogen phosphorylase is phosphorylated and active, whereas glycogen synthase is phosphorylated and inactive. C. Both glycogen phosphorylase and glycogen synthase are activated by phosphorylation but at significantly different rates. D. Glycogen phosphorylase is inactivated by a rise in Ca2+, whereas glycogen synthase is activated. E. cAMP-dependent protein kinase A is activated, whereas phosphorylase kinase is inactivated.

*The answer is B.* Patients with von Gierke disease display elevated levels of lactate, which interferes with the kidney's ability to remove uric acid from the blood and place it in the urine. This leads to hyperuricemia. The reason lactate levels are elevated is that the high glucose-6-phosphate in the cell (recall, the defect in this disorder is a lack of glucose-6-phosphatase activity) forces glycolysis forward, producing pyruvate, which is converted to lactate in order to regenerate NAD+ to allow glycolysis to continue. Glucose-6-phosphate does not inhibit glucose-6-phosphate dehydrogenase (that enzyme is regulated by the NADP+ levels), nor does it regulate a committed step of de novo purine synthesis, amidophosphoribosyl transferase (which is regulated by adenine and guanine nucleotides). Glucose-6-phosphate does stimulate glycogen synthase D, but that activation does not play a role in elevated urate levels. Glucose-6-phosphate does not affect urate absorption within the kidney.

The hyperuricemia observed in patients with von Gierke disease comes about due to which of the following? (A) Glucose-6-phosphate inhibition of kidney tubule absorption of urate (B) Lactate inhibition of kidney tubule absorption of urate (C) Glucose-6-phosphate inhibition of glucose-6-phosphate dehydrogenase activity (D) Glucose-6-phosphate stimulation of glycogen synthase D (E) Glucose-6-phosphate activation of amidophosphoribosyl transferase activity

*The answer is B.* Phosphorylase produces glucose-1-phosphate from glucose residues linked α-1,4. Free glucose is produced from α-1,6-linked residues at branch points by an α-1,6-glucosidase activity of the debranching enzyme. Degradation of glycogen produces glucose1 phosphate and glucose in about a 10:1 ratio, which is the ratio of the α-1,4 linkages to α-1,6 linkages.

The immediate degradation of glycogen under normal conditions gives rise to which one of the following? (A) More glucose than glucose-1-phosphate (B) More glucose-1-phosphate than glucose (C) Equal amounts of glucose and glucose-1-phosphate (D) Neither glucose nor glucose-1-phosphate (E) Only glucose-1-phosphate

*The answer is D.* The pentose phosphate pathway (hexose monophosphate shunt) functions to generate NADPH for reductive synthesis of compounds such as fatty acids or steroids, and to generate ribose for nucleotide and nucleic acid synthesis. As is the case with many pathways, the first step in the pentose phosphate pathway is regulated. This irreversible step is catalyzed by glucose-6-phosphate dehydrogenase, which is stimulated by increasing levels of NADP+ and competitively inhibited by NADP+. The dehydrogenation (oxidation) of glucose-6-phosphate produces 6-phosphoglucono-δ-lactone and reduced nicotinamide dinucleotide phosphate (NADPH). NAD+, NADP+, FAD+, and flavin adenine mononucleotide (FMN+) function as oxidizing agents for many biochemical reactions. The removal of positive hydrogen ions (H+) from a substrate AH2 effectively removes electrons (oxidizes it) to form A, simultaneously adding H+ ions and reducing (adding electrons) to form the reduced cofactors NADH/NADPH/FADH/FMNH. The reduced cofactors are oxidized back to NAD+/NADP+/FAD+/FMN+ through chemical reactions driven by high-energy phosphates (anaerobic conditions) or by molecular oxygen during electron transport (aerobic conditions).

The key regulatory enzyme of the pentose phosphate pathway is positively regulated by (A) Reduced nicotinamide dinucleotide (NADH) (B) Adenosine diphosphate (ADP) (C) Guanosine triphosphate (GTP) (D) Nicotinamide dinucleotide phosphate (NADP+) (E) Reduced flavine adenine dinucleotide (FADH)

*The answer is E.* Under circumstances of intense muscular contraction, the rate of formation of NADH by glycolysis exceeds the capacity of mitochondria to reoxidize it. Consequently, pyruvate produced by glycolysis is reduced to lactate, thereby regenerating NAD+. Since erythrocytes have no mitochondria, accumulation of lactate occurs normally. Lactate goes to the liver via the blood, is formed into glucose by gluconeogenesis, and then reenters the bloodstream to be reutilized by erythrocytes or muscle. This recycling of lactate to glucose is called the Cori cycle. A somewhat similar phenomenon using alanine generated by muscles during starvation is called the glucose-alanine cycle. All of the other substances listed—oxaloacetate, glycerol, and pyruvate—can be made into glucose by the liver.

The major metabolic product produced under normal circumstances by erythrocytes and by muscle cells during intense exercise is recycled through the liver in the Cori cycle. The metabolite is a. Oxaloacetate b. Glycerol c. Alanine d. Pyruvate e. Lactate

*The answer is C.* Phosphofructokinase-1 is the pace-setting enzyme of glycolysis. It is inhibited by ATP and citrate, uses fructose 6-phosphate as substrate, and catalyzes a reaction that is far from equilibrium. The reaction is activated by fructose 2,6-bisphosphate.

The reaction catalyzed by phosphofructokinase-1: A. is activated by high concentrations of ATP and citrate. B. uses fructose 1-phosphate as substrate. C. is the rate-limiting reaction of the glycolytic pathway. D. is near equilibrium in most tissues. E. is inhibited by fructose 2,6-bisphosphate.

*The answer is E.* Hunter syndrome is an X-linked disorder that is caused by a deficiency of iduronate sulfatase. Although Hunter syndrome and Hurler syndrome are similar~ Hunter syndrome is notable for the absence of corneal clouding, which is present in Hurler syndrome. Patients with Hunter syndrome may also exhibit coarsened facial features, mildly aggressive behavior~ and pearly skin lesions on their scapulae.

The mother of a 3-year-old boy is referred to genet ic counseling after her son is diagnosed with an enzyme deficiency. Recently, the mother noticed that her son has been developing an abnormal facial appearance and is hyperactive compared with other children of the same age. Examination reveals pearly papular skin lesions over the scapulae and on the lateral upper arms and thighs. Funduscopic examination findings are unremarkable, and his corneas are clear bilaterally. This patient is likely def icient in which enzyme? A. α-L-Iduronidase B. β~Galactosidase C. β-Glucuronid ase D. Galactosamine-6-sulfatase E. Iduronate sulfatase F. Sulfamidase

*The answer is A.* The nonoxidative branch of the pentose phosphate pathway predominates over the oxidative branch when more ribose 5-phosphate than NADPH is required. Cells in the S phase require more ribose 5-phosphate in order to synthesize DNA. Therefore, these cells will go through glycolysis to produce the intermediates fructose 6-phosphate and glyceraldehyde 3-phosphate. These intermediates will then help to produce ribose 5-phosphate and the nonoxidative phase takes place, whereas the oxidative phase does not take place.

The nonoxidative branch of the pentose phosphate pathway predominates over the oxidative branch when (A) more ribose 5-phosphate than NADPH is required. (B) the needs for NADPH and ribose 5-phosphate are balanced. (C) more NADPH than ribose 5-phosphate is required. (D) more NADPH than sedoheptulose 7-phosphate is required.

*The answer is C.* The oxidative branch of the pentose phosphate pathway predominates over the nonoxidative branch when more NADPH is required than ribose 5-phosphate. Fat cells usually need more NADPH, which is produced in the oxidative phase. These cells go through the oxidative and nonoxidative phase and the intermediates then go through gluconeogenesis to produce more glucose 6-phosphate to produce more NADPH.

The oxidative branch of the pentose phosphate pathway predominates over the nonoxidative branch when (A) more ribose 5-phosphate than NADPH is required. (B) the needs for NADPH and ribose 5-phosphate are balanced. (C) more NADPH than ribose 5-phosphate is required. (D) more sedoheptulose 7-phosphate than NADPH is required.

*The answer is B.* In the oxidative reactions of the pentose phosphate pathway, glucose is converted to ribulose-5-phosphate and CO2, with the production of NADPH. These reactions are not reversible. Ribose-5 phosphate and xylulose-5-phosphate are formed from ribulose5 phosphate by two of the nonoxidative reactions of the pathway. Ribose 5-phosphate is used for biosynthesis of nucleotides such as ATP. A series of reactions catalyzed by transketolase and transaldolase produce the glycolytic intermediates fructose-6-phosphate and glyceraldehyde3 phosphate. Glucose is produced by gluconeogenesis in humans, and not directly by the hexose monophosphate shunt pathway.

The pentose phosphate pathway generates which one of the following? (A) NADH, which may be used for fatty acid synthesis. (B) Ribose-5-phosphate, which may be used for the biosynthesis of ATP. (C) Pyruvate and fructose 1,6-bisphosphate by the direct action of transaldolase and transketolase. (D) Xylulose-5-phosphate by one of the oxidative reactions. (E) Glucose from ribose-5-phosphate and CO2.

*The answer is E.* Degradation of glycoproteins follows the rule "last on, first off." Because sulfation is the last step in the synthesis of this sequence, a sulfatase is required for the next step in the degradation of the compound shown.

The presence of the following compound in the urine of a patient suggests a deficiency in which one of the enzymes listed below? A. Galactosidase B. Glucosidase C. Glucuronidase D. Mannosidase E. Sulfatase

*The answer is C.* NADH generated from glycolysis must be relieved of an electron to form nicotinamide adenine dinucleotide (NAD+) so that glycolysis may continue. However, mitochondrial membranes are impermeable to both NADH and NAD+. The solution to this problem is the transfer of electrons from NADH to molecules that traverse the membrane. In the glycerophosphate shuttle, dihydroxyacetone phosphate (DHAP) is reduced to glycerol-3-phosphate and thereby regenerates NAD+. The glycerol-3-phosphate diffuses into mitochondria and is oxidized by flavin adenine dinucleotide (FAD) back to DHAP, which can diffuse back into the cytosol. The mitochondrial reduced form of flavin adenine dinucleotide (FADH2) produced yields 2 ATP in the electron transport chain. In the heart and liver, the more energy-efficient malate-aspartate shuttle moves electrons into mitochondria. Cytoplasmic oxaloacetate is reduced to malate, which diffuses into the mitochondria and is oxidized by NAD+ back to oxaloacetate. The mitochondrial NADH produced yields 3 ATP on electron transport. The mitochondrial oxaloacetate is converted to aspartate, which diffuses into the cytosol, where it is converted back into cytoplasmic oxaloacetate.

The problem of regenerating NAD+ from NADH for cytoplasmic processes by using mitochondria is solved in the most energy-efficient manner by which one of the following intercellular shuttle systems? a. Citrate → pyruvate shuttle b. Dihydroxyacetone phosphate →α-glycerophosphate shuttle c. Malate → aspartate shuttle d. Citrate → citrate shuttle e. Lactate → pyruvate shuttle

*The answer is C.* NADH generated from glycolysis must be relieved of an electron to form nicotinamide adenine dinucleotide (NAD+) so that glycolysis may continue. However, mitochondrial membranes are impermeable to both NADH and NAD+. The solution to this problem is the transfer of electrons from NADH to molecules that traverse the membrane. In the glycerophosphate shuttle, dihydroxyacetone phosphate (DHAP) is reduced to glycerol-3-phosphate and thereby regenerates NAD+. The glycerol-3-phosphate diffuses into mitochondria and is oxidized by flavin adenine dinucleotide (FAD) back to DHAP, which can diffuse back into the cytosol. The mitochondrial reduced form of flavin adenine dinucleotide (FADH2) produced yields 2 ATP in the electron transport chain. In the heart and liver, the more energy-efficient malate-aspartate shuttle moves electrons into mitochondria. Cytoplasmic oxaloacetate is reduced to malate, which diffuses into the mitochondria and is oxidized by NAD+ back to oxaloacetate. The mitochondrial NADH produced yields 3 ATP on electron transport. The mitochondrial oxaloacetate is converted to aspartate, which diffuses into the cytosol, where it is converted back into cytoplasmic oxaloacetate.

The problem of regenerating NAD+ from NADH for cytoplasmic processes by using mitochondria is solved in the most energy-efficient manner by which one of the following intercellular shuttle systems? (A) Citrate → pyruvate shuttle (B) Dihydroxyacetone phosphate → α-glycerophosphate shuttle (D) Malate → aspartate shuttle (D) Citrate → citrate shuttle (E) Lactate → pyruvate shuttle

*The answer is D.* Pyridoxal phosphate is required for the transamination of aspartate to oxaloacetate and glutamic acid to α-ketoglutarate. Both the α-keto acids are TCA cycle components, and when their levels decrease, they can be replenished through such a reaction. Niacin, riboflavin, and lipoate are required for oxidative decarboxylation reactions, but that reaction type does not lead to a refilling of TCA cycle intermediates. Carnitine is required to transport acyl groups into the mitochondria and is not used to transport TCA cycle intermediates from the cytoplasm to the mitochondria. Biotin would be a correct answer (for the pyruvate carboxylase reaction, to regenerate oxaloacetate from pyruvate), but it was not offered as a choice. A typical transamination reaction is shown below.

The refilling of TCA cycle intermediates is frequently dependent upon which of the following cofactors? (A) Niacin (B) Riboflavin (C) Carnitine (D) Pyridoxal phosphate (E) Lipoate

*The answer is C.* Biotin is the coenzymeprosthetic group of pyruvate carboxylase. The carboxylation of pyruvate occurs in the mitochondria. Glucagon stimulates gluconeogenesis. Lactate is not an intermediate in the conversion of pyruvate to glucose; however, pyruvate can be produced from lactate. FAD is not involved in gluconeogenesis.

The synthesis of glucose from pyruvate by gluconeogenesis: A. occurs exclusively in the cytosol. B. is inhibited by an elevated level of glucagon. C. requires the participation of biotin. D. involves lactate as an intermediate. E. requires the oxidation/reduction of FAD.

*The answer is E.* In gluconeogenesis, phosphoglycerate kinase catalyzes the phosphorylation of 3-phosphoglycerate to 1,3-bisphosphoglycerate, a step which requires ATP. The other two steps requiring a high-energy phosphate bond in the conversion of pyruvate to glucose are pyruvate carboxylase and phosphoenolpyruvate carboxykinase. Fructose-1,6-bisphosphatase and glucose-6-phosphatase are enzymes that remove phosphates from substrates, releasing the phosphates as inorganic phosphate. They do not require, nor generate, ATP. Pyruvate kinase is not utilized for gluconeogenesis, and triose phosphate isomerase catalyzes the conversion of dihydroxyacetone phosphate and glyceraldehyde-3-phosphate, without the involvement of a high-energy phosphate bond. These are shown in the pathway below.

The synthesis of one mole of glucose from two moles of lactate requires six moles of ATP. Which one of the following steps requires ATP in the gluconeogenic pathway? (A) Pyruvate kinase (B) Triosephosphate isomerase (C) Glucose-6-phosphatase (D) Fructose-1,6-bisphosphatase (E) Phosphoglycerate kinase

*The answer is C.* Tay-Sachs disease results from a deficiency of hexosaminidase A activity. This disorder is inherited as an autosomal recessive trait and has a carrier frequency of 1:25 in the Ashkenazi Jewish population. Findings include loss of motor skills, increased startle reaction, macular pallor, and a cherry-red spot on the macula (as shown in the image). Diagnosis is confirmed by the quantification of hexosaminidase level in isolated WBCs in the blood. Future at-risk pregnancies can be monitored via prenatal diagnosis by either amniocentesis or chorionic villus sampling.

The wife in an Ashkenazi Jewish family brings her 1-year-old daughter to the pediatrician. Her previous pregnancy was uneventful and resulted in a full-term healthy girl who is now 4 years old. Her younger daughter, however, has demonstrated a progressive series of behaviors over the first year of life. Her motor skills have diminished and she demonstrates an increased startle reaction. Physical examination is notable only for the ocular findings shown in the image. Deficiency of which enzyme is responsible for this disease? (A) a-Galactosidase A (B) Arylsulfatase A (C) Hexosaminidase A (D) Iduronate sulfatase (E) Lysyl hydrolase

*The answer is C.* In the citric acid cycle, the conversion of α-ketoglutarate to succinate results in decarboxylation, transfer of an H+/e− pair to NADH + H+, and the substrate-level phosphorylation of GDP to GTP. The series of reactions involved is quite complex. First, α-ketoglutarate reacts with NAD+ + CoA to yield succinyl CoA + CO2 + NADH + H+. These reactions occur by the catalysis of the α-ketoglutarate dehydrogenase complex, which contains lipoamide, FAD+, and thiamine pyrophosphate as prosthetic groups. Under the action of succinyl CoA synthetase, succinyl CoA catalyzes the phosphorylation of GDP with inorganic phosphate coupled to the cleavage of the thioester bond of succinyl CoA. Thus, the production of succinate from α-ketoglutarate yields one substrate-level phosphorylation and the production of three ATP equivalents from NADH via oxidative phosphorylation.

Transfer of H+/e− pairs to electron transport carriers, decarboxylation, and substrate-level phosphorylation occur at some of the steps shown in the following diagram of the citric acid cycle. All three of these events occur at which step? (A) Step A (B) Step B (C) Step C (D) Step D (E) Step E

*The answer is D and E.* The answer is D and E. NADH and Oxaloacetate cannot cross the inner mitochondrial membrane. As a result, shuttles, such as the malate-aspartate shuttle are used to move the electrons from the cytoplasm to the matrix.

Transport molecules exist for all of the following compounds to cross the inner mitochondrial membrane except for which TWO? (A) Citrate (B) α-ketoglutarate (C) Malate (D) NADH (E) Oxaloacetate (F) Pyruvate

*The answer is E.* In general, you should associate hemolytic anemia with defects in glycolysis or the hexose monophosphate shunt (pentose phosphate pathway). Only two enzymes of those listed in the answer choices specifically involve these pathways and cause hemolytic anemia: pyruvate kinase and glucose-6-phosphate dehydrogenase. Glucose-6-phosphate dehydrogenase (G6PD) deficiency is inherited as an X-linked recessive trait, so females would not be affected. Pyruvate kinase is a glycolytic enzyme; pyruvate kinase deficiency is an autosomal recessive disorder, affecting males and females approximately equally. If this enzyme is deficient, red cells have trouble producing enough ATP to maintain the Na+/K+ pump on the plasma membrane, secondarily causing swelling and lysis.

Two sisters are diagnosed with hemolytic anemia. Their older brother was previously diagnosed with the same disorder. Two other brothers are asymptomatic. The mother and father are second cousins. Deficiency of which of the following enzymes would be most likely to cause this disorder? (A) Debranching enzyme (B) Glucose-6-phosphatase (C) Glucose-6-phosphate dehydrogenase (D) Muscle phosphorylase (E) Pyruvate kinase

*The answer is B.* The liver expresses glucokinase, which has a high Km for glucose, particularly as compared to the Km for hexokinase. This means that glucokinase will only phosphorylate glucose when the intrahepatic glucose concentrations are high, and the intrahepatic levels of glucose only reach these levels after a meal. Under normal, fasting conditions, the concentration of blood glucose is lower than the Km for glucokinase, and very little phosphorylation of glucose will occur. PFK-1 is not a phosphorylated enzyme, and glucagon does not stimulate an increase in glucose transporters in the liver.

Under conditions of hypoglycemia, the liver is not utilizing glucose as an energy source due to which of the following? (A) A low Km for glucokinase (B) A high Km for glucokinase (C) An inhibited, phosphorylated PFK-1 (D) An activated, phosphorylated PFK-1 (E) A reduction of glucose transporters in the membrane

*The answer is A.* Upon insulin release, the cAMP phosphodiesterase is activated, reducing cAMP levels in the liver, thereby leading to inactivation of protein kinase A. In addition, protein phosphatase 1 has become active and dephosphorylates the enzymes that were phosphorylated by protein kinase A. Therefore, PFK-2 is not phosphorylated, which leads to an active kinase activity and an inactive phosphatase activity (choices A, D, or E). The active kinase of PFK-2 produces more fructose-2,6-bisphosphate, leading to the activation of PFK-1 (answers A through D; combined with PFK-2 activity, now only choice A or D can be correct). Insulin stimulates preformed GLUT4 transporters in the muscle to fuse with the plasma membrane, thereby enhancing glucose transport into the muscle (choices A through C; combined with the other two columns, only choice A can be correct).

Which of the following changes in enzyme activity will occur within 1 h of a type 1 diabetic taking an injection of insulin?

*The answer is D.* In addition to NO, metabolism of nitroprusside also releases small quantities of cyanide, a potent and potentially lethal inhibitor of cyt a/a3 (complex IV). Thiosulfate is a common antidote for CN poisoning.

When nitroprusside is given in higher than usual doses, it may be accompanied by the administration of thiosulfate to reduce potential toxic side effects. Which complex associated with electron transport or oxidative phosphorylation is most sensitive to the toxic byproduct that may accumulate with high doses of nitroprusside? A. NADH dehydrogenase B. Succinate dehydrogenase C. Cytochrome b/c1 D. Cytochrome a/a3 E. F0F1 ATP synthase

*The answer is A.* Pyruvate dehydrogenase complex requires the activity of lipoic acid (cofactor for dihydrolipoyl transacetylase), thiamine pyrophosphate (cofactor for Pyruvate dehydrogenase), coenzyme A (cofactor for dihydrolipoyl transacetylase), and flavin adenine dinucleotide (cofactor for dihydrolipoyl dehydrogenase.

Which coenzyme is NOT required for the activity of the pyruvate dehydrogenase complex? (A) biotin (B) lipoic acid (C) thiamine pyrophosphate (D) coenzyme A (E) flavin adenine dinucleotide

*The answer is C.* A variety of agonists activate the plasma membrane bound enzyme phospholipase C, which hydrolyzes the phosphodiester bond of phosphatidyl inositol 4,5-bisphosphate and consequently releases diacylglycerol (DAG) and inositol 1,4,5-triphosphate (IP3). Phospholipase C is also known as phosphoinositidase and as polyphosphoinositide phosphodiesterase. Both DAG and IP3 are second messengers. DAG activates protein kinase C, which is important in controlling cell division and cell proliferation. IP3 opens calcium channels and allows the rapid release of the calcium stores in endoplasmic reticulum (in smooth muscle, sarcoplasmic reticulum). The elevated levels of calcium ion stimulate smooth-muscle contraction, exocytosis, and glycogen breakdown.

Which of the descriptions below best fits inositol 1,4,5-triphosphate? a. A depositor of calcium in endoplasmic reticulum b. An inhibitor of protein kinase C c. A second messenger produced by the action of phospholipase C d. An ionophore e. A calcium detector

*The answer is C.* Cellulose, the most abundant compound known, is the structural fiber of plants and bacterial walls. It is a polysaccharide consisting of chains of glucose residues linked by β 1→ 4 bonds. Since humans do not have intestinal hydrolases that attack β 1→ 4 linkages, cellulose cannot be digested but forms an important source of "bulk" in the diet. Lactose is a disaccharide of glucose and galactose found in milk. Amylose is an unbranched polymer of glucose residues in α-1,4 linkages. Glycogen is a branched polymer of glucose with both α-1,4 and α-1,6 linkages. Maltose is a disaccharide of glucose, which is usually the breakdown product of amylose.

Which of the following carbohydrates would be most abundant in the diet of strict vegetarians? a. Amylose b. Lactose c. Cellulose d. Maltose e. Glycogen

*The answer is A.* Amylo-(1,4 →1,6)-transglycosylase (also known as the branching enzyme) functions in glycogen synthesis. The other enzymes listed are involved in glycogenolysis. Deficiencies in these enzymes lead to glycogen storage with enlarged liver and hypoglycemia. Mobilization of glycogen stores to produce glucose in the liver requires phosphorolysis of the glycogen chain by the enzyme phosphorylase, which is phosphorylated by phosphorylase kinase. Also needed are the hydrolysis of α-1,6-glycosidic bonds by amylo-1,6-glucosidase (also known as the debranching enzyme) and the hydrolysis of glucose-6 phosphate derived from glucose-1-phosphate (a product of phosphorylase) by glucose-6-phosphatase to produce glucose for export into the blood.

Which of the following enzymes is associated with glycogen synthesis? a. Amylo-(1,4 → 1,6)-transglycosylase b. Phosphorylase c. Phosphorylase kinase d. Amylo-1,6-glucosidase e. Glucose-6-phosphatase

*The answer is E.* This disease is more prevalent in Africans because an advantage of this disease is that it confers protection against malaria, particularly Plasmodium falciparum. It is thought that cells infected with the parasites are cleared more rapidly by the spleen giving individuals with the G6PD deficiency gene an evolutionary advantage in malarial endemic environments.

Which of the following ethnic groups is most frequently affected by G6PD deficiency? (A) Asian females (B) Arab American culture (C) Native American culture (D) Ashkenazi Jews (E) African Americans (F) Hispanic

*The answer is A.* In the formation of phosphoenolpyruvate during gluconeogenesis, oxaloacetate is an intermediate. In the first step, catalyzed by pyruvate carboxylase, pyruvate is carboxylated with the utilization of one high-energy ATP phosphate bond: pyruvate + ATP + CO2 → oxaloacetate + ADP + Pi In the second step, catalyzed by phosphoenolpyruvate carboxykinase, a high-energy phosphate bond of GTP drives the decarboxylation of oxaloacetate: oxaloacetate + GTP → phosphoenolpyruvate + GDP + CO2 In contrast to gluconeogenesis, the formation of pyruvate from phosphoenolpyruvate during glycolysis requires only pyruvate kinase, and ATP is made.

Which of the following events occurs during formation of phosphoenolpyruvate from pyruvate during gluconeogenesis? (A) CO2 is consumed (B) Inorganic phosphate is consumed (C) Acetyl CoA is utilized (D) ATP is generated (E) GTP is generated

*The answer is A.* Fructose is taken in by humans as sucrose, sucrose-containing syrups, and the free sugar. Fructose is mainly phosphorylated to fructose-1-phosphate by liver fructokinase. Aldol cleavage by fructose-1-phosphate-specific aldolase, not enolase, yields glyceraldehyde and dihydroxyacetone phosphate. The glyceraldehyde is phosphorylated to glyceraldehyde-3-phosphate by triose kinase, and both triose phosphates can enter glycolysis. Excess fructose from commercial foods can exercise adverse effects by raising blood lipids and uric acid. Fructose phosphorylation bypasses phosphofructokinase, a regulatory enzyme of glycolysis, and provides excess glycerol metabolites and excess triglyceride/lipid biosynthesis. Fructose phosphorylation can also deplete liver cell ATP, lessening its inhibition of adenine nucleotide degradation and increasing production of uric acid. In adipocytes, fructose can be alternatively phosphorylated by hexokinase to fructose-6-phosphate. However, this reaction is competitively inhibited by appreciable amounts of glucose, as it is in other tissues.

Which of the following explains why individuals with hyperlipidemia and/or gout should minimize their intake of sucrose and high fructose syrups? (A) Fructose is initially phosphorylated by liver fructokinase (B) After initial modification, fructose is cleaved by a specific enolase (C) Fructose is converted to UDP-fructose (D) Fructose is ultimately converted to galactose (E) Fructose can be phosphorylated by hexokinase in adipose cells

*The answer is B.* Fava beans contain alkaloids that can induce a state of oxidative stress, causing hemolytic anemia in patients with G6PD deficiency.

Which of the following foods or food groups is most likely to trigger an exacerbation in a patient with G6PD deficiency? (A) Smoked food (B) Fava beans (C) Cheese (D) Ingestion of raw eggs (E) High fiber diet

*The answer is B.* NADPH has three primary functions: involvement in biosynthesis of lipids and cholesterol (the precursor to steroid hormones), production of bactericidal bleach in the lysosomes of certain white blood cells, and maintenance of a supply of reduced glutathione for protection against free radical damage. Energy carriage is an important function of NADH, not NADPH.

Which of the following is/are function(s) of NADPH in the cell? I. Antimicrobial resistance via bacterial destruction by bleach within lysosomes. II. Involvement in the production of the precursors to steroid hormones. III. Functional carriage of energy across organelle membranes for use within the mitochondria (A) I only (B) I and II only (C) II and III only (D) I, II, and III

*The answer is B.* Glucose-6-phosphate is a pivotal compound in many pathways. Immediately upon entering cells, blood glucose is phosphorylated to glucose-6 -phosphate by hexokinase in most cells and by glucokinase in the liver. Glucose may only leave a cell in the dephosphorylated form produced by glucose-6-phosphatase, which is only found in liver. Glucose-6-phosphate may be the starting point of glycolysis, glycogen synthesis, and the pentose phosphate pathway. It can be considered the end point or switching point of glycogenolysis and gluconeogenesis. Uridine-diphosphoglucose (UDP-glucose) and UDP-galactose are high-energy forms of their respective sugars that are involved in converting galactose-I-phosphate to glucose-1-phosphate (the block in galactosemia) and in donating sugar groups to polysaccharides such as glycogen or glycosaminoglycans. Fructose-6-phosphate is involved in glycolysis and gluconeogenesis.

Which of the following metabolites is involved in glycogenolysis, glycolysis, and gluconeogenesis? a. Galactose-1-phosphate b. Glucose-6-phosphate c. Uridine diphosphoglucose d. Fructose-6-phosphate e. Uridine diphosphogalactose

*The answer is D.* ATP is synthesized by two reactions in glycolysis. The first molecule of ATP is generated by phosphoglycerate kinase, converting 1,3-diphosphoglycerate to 3-phosphoglycerate. The second molecule of ATP is generated by pyruvate kinase, converting phosphoenolpyruvate to pyruvate.

Which of the following reactions generates ATP? a. Glucose-6-phosphate to fructose-6-phosphate b. Glucose to glucose-6-phosphate c. Fructose-6-phosphate to fructose-1,6-diphosphate d. Phosphoenolpyruvate to pyruvate e. Pyruvate to lactate

*The answer is B.* In mammalian systems ATP may be formed in the presence or the absence of molecular oxygen. In glycolysis, ATP is formed by substrate-level phosphorylation. In mitochondria, ATP is formed by the oxidation of NADH. ATP hydrolysis is an exergonic reaction that releases energy. When this released energy is chemically linked to an energy-requiring or endergonic reaction, that reaction is driven. Heat does not drive biologic reactions.

Which of the following statements about mammalian energy metabolism is true? a. ATP is only formed in the absence of Q2 b. ATP hydrolysis is an exergonic reaction c. ATP is only formed in the presence of Q2 d. Heat produced by ATP hydrolysis specifically drives other reactions e. NADH cannot be utilized to form ATP

*The answer is A, B, and D.* The pentose phosphate pathway begins with glucose-6-phosphate which is produced by the first step in glycolysis. Glucose-6-phosphate dehydrogenase, the first irreversible reaction, is the rate limiting step of the pentose phosphate pathway which provides NADPH for reductive biosyntheses and ribose-5-phosphate for nucleotide synthesis according to the cell's needs. If a fat cell needs to produce excess NADPH for fatty acid synthesis, the excess ribose-5-phosphate will be converted to glycolytic intermediates according to the cell's needs by the enzymes transketolase and transaldolase. If a cell is dividing intermediates from the glycolytic pathway can be converted to ribose-5-phosphate for DNA synthesis by the reversal of the transketolase and transaldolase reactions. This illustrates the relationship between the two pathways.

Which of the following statements about the nonoxidative section of the pentose phosphate pathway are correct? Please select all that apply. (A) Glycolysis provides intermediates for the pentose phosphate pathway. (B) The pentose phosphate pathway provides intermediates for glycolysis. (C) The pentose phosphate pathway cannot provide pentoses in the quantities the cell requires. (D) There is a relationship between glycolysis and the pentose phosphate pathway in the liver. (E) There is no relationship between glycolysis and the pentose phosphate pathway in the liver.

*The answer is D.* The oxidative section of the pentose phosphate pathway supplies ribose-5-phosphate for nucleotide and nucleic acid synthesis, ribose for coenzymes such as NAD and FAD, and NADPH for reductive biosyntheses. The nonoxidative section of the pathway is a route for excess pentose sugars to be brought into the mainstream of glucose metabolism. It is a mechanism by which sugars can be interconverted according to the cell's needs. The key reactions in this section are catalyzed by the enzymes transketolase and transaldolase. Transketolase transfers two-carbon units and transaldolase transfers three-carbon units from pentose sugar phosphates to other aldose sugars.

Which of the following statements about the nonoxidative section of the pentose phosphate pathway is correct? (A) The nonoxidative reactions of the pentose phosphate pathway are not reversible. (B) Transketolase is an enzyme that transfers three-carbon units in the pentose phosphate pathway. (C) Transaldolase is an enzyme that transfers two-carbon units in the pentose phosphate pathway. (D) Pentoses undergo isomerizations in the pentose phosphate pathway.

*The answer is D.* The pentose phosphate pathway supplies ribose-5-phosphate for nucleotide and nucleic acid synthesis, and for synthesis of coenzymes such as NAD+, FAD and CoA. The pathway also supplies NADPH for reductive biosyntheses. It is a route for excess pentose sugars to be brought into the mainstream of glucose metabolism. The pathway has two main parts. One part is the irreversible oxidation of glucose-6-phosphate to ribose-5-phosphate while NADP+ is reduced to NADPH. The rate-limiting reaction here is catalyzed by glucose-6-phosphate dehydrogenase. This section of the pathway is controlled by the availability of NADP+. The second part of the pentose phosphate pathway is made up of nonoxidative reaction sequences that interconvert sugars according to the cell's needs. Excess ribose-5-phosphate is converted to glycolytic intermediates by a sequence of reversible reactions.

Which of the following statements about the oxidative section of the pentose phosphate pathway is correct? (A) The pentose phosphate pathway generates NADH. (B) The pentose phosphate pathway oxidizes NADPH to NADP+. (C) The rate-limiting reaction of the pentose phosphate pathway is catalyzed by glucose-6-phosphatase. (D) The pathway supplies ribose-5-phosphate and NADPH in the quantities the cells require.

*The answer is D.* The pentose phosphate pathway produces ribose-5-phosphate for nucleotide and nucleic acid synthesis, and for coenzymes such as NAD and FAD. The pathway also supplies NADPH for reductive biosyntheses and it is a route for excess pentose sugars to be brought into the mainstream of glucose metabolism. The pathway has two main parts. The first is the irreversible oxidation of glucose-6-phosphate to ribose-5-phosphate while NADP+ is reduced to NADPH and the rate-limiting reaction is catalyzed by glucose-6-phosphate dehydrogenase. Control of this part of the pathway is mainly by the availability of NADP+. The other part of the pentose phosphate pathway is made up of reversible nonoxidative reactions that interconvert sugars according to the cell's needs using the enzymes transketolase and transaldolase.

Which of the following statements about the pentose phosphate pathway is not true? (A) The pentose phosphate pathway relies on the availability of NADP+. (B) The pentose phosphate pathway has an irreversible oxidative section and a reversible non oxidative section. (C) The pentose phosphate pathway enables excess ribose to be metabolised. (D) The pentose phosphate pathway relies on the availability of NADPH.

*The answer is D.* Glycogen is a highly branched polymer of α-D-glucose residues joined by α-1,4-glycosidic linkage. Under the influence of glycogen synthase, the C4 alcohol of a new glucose is added to the C1 aldehyde group of the chain terminus. The branched chains occur about every 10 residues and are joined in α-1,6-glycosidic linkages. Large\ amounts of glycogen are stored as 100- to 400-Å granules in the cytoplasm of liver and muscle cells. The enzymes responsible for making or breaking the α-1,4-glycosidic bonds are contained within the granules. Thus glycogen is a readily mobilized form of glucose.

Which of the following statements about the structure of glycogen is true? a. Glycogen is a copolymer of glucose and galactose b. There are more branch residues than residues in straight chains c. Branch points contain α-1,4 glycosidic linkages d. New glucose molecules are added to the C1 aldehyde group of chain termini, forming a hemiacetal e. The monosaccharide residues alternate between D- and L-glucose

*The answer is D.* There are multiple uses for the NADPH produced from the pentose phosphate pathway. It is required for reductive biosyntheses such as in fatty acid synthesis and for the regeneration of oxidised glutathione to its reduced state. Glutathione is involved in the glutathione peroxidase system for protection against peroxides. NADPH is also used in the cytochrome P450 monooxygenase system for drug metabolism and in the hydroxylation of steroids for cholesterol and steroid production.

Which of the following statements about the use of the NADPH generated from the pentose phosphate pathway is not true? (A) NADPH generated from the pentose phosphate pathway is used for steroid synthesis. (B) NADPH generated from the pentose phosphate pathway is used for the regeneration of glutathione to its reduced state. (C) NADPH generated from the pentose phosphate pathway is used for the synthesis of fatty acids. (D) NADPH generated from the pentose phosphate pathway and cytoplasmic NADH is metabolically interchangeable.

*The answer is D.* While mature red blood cells do not divide, or synthesize fat, they do utilize the pentose phosphate pathway for the production of NADPH which maintains glutathione in its reduced state to protect the cell against reactive oxygen species. It is involved in the glutathione peroxidase system for protection against peroxides.

Which of the following statements about the use of the pentose phosphate pathway in red blood cells is correct? (A) Mature red blood cells do not need the pentose phosphate pathway since they do not divide. (B) Mature red blood cells do not need the pentose phosphate pathway since they do not synthesize fat. (C) Mature red blood cells need the pentose phosphate pathway to oxidize the glucose-6-phosphate. (D) Mature red blood cells need the pentose phosphate pathway for the production of NADPH

*The answer is C.* Glucagon counteracts insulin's action, in that it is a catabolic hormone (thus, A is incorrect), and its levels decrease when insulin levels increase (thus, E is incorrect). As insulin levels rise after a meal, glucagon levels will decrease, so D is incorrect. The muscle lacks glucagon receptors and cannot respond to glucagon, hence B is incorrect.

Which of the following statements best describes glucagon? (A) It acts as an anabolic hormone. (B) It acts on skeletal muscle, liver, and adipose tissue. (C) It acts primarily on the liver and adipose tissue. (D) Its concentration in the blood increases after a high carbohydrate meal. (E) Its concentration increases in the blood when insulin levels increase.

*The answer is D.* Ketone bodies include acetoacetic acid and β-hydroxybutyrate, which are formed in the liver, and acetone, which is spontaneously formed from excess acetoacetate in the blood. Starvation results in glycogen depletion and deficiency of carbohydrates, causing increased use of lipids as energy sources. Increased oxidation of fatty acids produces acetyl coenzyme A (CoA) and acetoacetyl CoA, a precursor of ketone bodies. Although the liver synthesizes ketone bodies from excess acetyl CoA produced by β-oxidation of fatty acids, it cannot use ketone bodies as fuel for energy. Insulin deficiency in diabetes mellitus, like starvation, depletes carbohydrate energy sources because glucose cannot enter cells. Increased oxidation of fatty acids again results, with increased production of ketone bodies by the liver. Children with inherited blocks in fatty acid oxidation such as medium chain CoA dehydrogenase deficiency cannot switch to fatty oxidation when illness or overnight fasting causes carbohydrate depletion. They often present with low blood glucose (hypoglycemia) and without the expected ketone bodies (nonketonic hypoglycemia). The unavailability of fatty acid oxidation as an alternative energy source may lead to energy depletion, organ failure, and unexpected "sudden death" in an otherwise healthy child.

Which of the following statements correctly describes ketone bodies? a. They accumulate in children with fatty acid oxidation disorders b. They accumulate in diabetes mellitus after insulin therapy c. They are produced by muscle but not by liver d. They include β-hydroxybutyrate and acetone e. They are found in blood but not in urine

*The answer is C.* Lipolysis in adipose tissue leads to increased blood levels of fatty acids and glycerol. Since most tissues have little glycerol kinase, the liver takes up most of the free glycerol, phosphorylates it using glycerol kinase, and oxidizes it to dihydroxyacetone phosphate (DHAP) using a dehydrogenase: glycerol + ATP → glycerol-3-P + ADP glycerol-3-P + NAD+ → DHAP + NADH + H+ Aldolase allows both of the triose phosphates DHAP and glyceraldehyde-3-phosphate to condense and form fructose-1,6-bisphosphate. In this manner, aldolase allows adipocyte glycerol to enter hepatic gluconeogenesis.

Which of the following steps is involved in the generation of glucose from lipolysis? a. Glycerol from lipolysis is converted to triglycerides b. Fatty acids from lipolysis are oxidized, producing NADH and stimulating gluconeogenesis c. Glycerol from lipolysis is phosphorylated, converted to fructose-1,6-bisphosphate, and eventually converted to glucose d. Fatty acids from lipolysis stimulate the citric acid cycle e. Glycerol from lipolysis is taken up by liver cells and dimerized to fructose

*The answer is E.* The pentose phosphate pathway generates reducing power in the form of NADPH in the oxidative branch of the pathway and synthesizes five-carbon sugars in the nonoxidative branch of the pathway. The pentose phosphate pathway also carries out the interconversion of three-, four-, five-, six-, and seven-carbon sugars in the nonoxidative reactions. The final sugar product of the oxidative branch of the pathway is ribulose-5-phosphate. The first step of the nonoxidative branch of the pathway is the conversion of ribulose-5- phosphate to ribose-5-phosphate or xylulose-5-phosphate in the presence of the enzymes phosphopentose isomerase and phosphopentose epimerase, respectively. Thus ribulose-5-phosphate is a key intermediate that is common to both the oxidative and nonoxidative branches of the pentose phosphate pathway.

Which one of the following compounds is common to both the oxidative branch and the nonoxidative branch of the pentose phosphate pathway? (A) Xylulose-5-phosphate (B) Glucose-6-phosphate (C) Glyceraldehyde-3-phosphate (D) Fructose-6-phosphate (E) Ribulose-5-phosphate

*The answer is B.* The HMP shunt can have increased activity under two conditions, one being an increase in the cofactor NADP+ levels and the other being an increase in the substrate levels (glucose-6-phosphate). The only enzyme listed, which when defective would lead to an increase in either glucose-6-phosphate or NADPH, is glucose-6-phosphatase. A deficiency in glucose 6-phosphatase would lead to an accumulation of glucose-6-phosphate, which is the first step of the oxidative step of the pentose phosphate pathway. A deficiency in glycogen phosphorylase would not produce glucose-1-phosphate; thus, there would not be an increase in the HMP shunt under these conditions. A deficiency in fructose-1,6-bisphosphatase deficiency would impair gluconeogenesis and would not lead to the synthesis of glucose-6-phosphate. Deficiencies in either pyruvate carboxylase or pyruvate dehydrogenase would lead to pyruvate accumulation and NAD+ accumulation, but not NADP+ or glucose-6-phosphate accumulation.

Which one of the following disorders would lead to increased activity of the HMP shunt pathway? (A) Glycogen phosphorylase deficiency (B) Glucose-6-phosphatase deficiency (C) Fructose-1,6-bisphosphatase deficiency (D) Pyruvate kinase deficiency (E) Pyruvate dehydrogenase deficiency

*The answer is D.* All the enzymes listed are specific to either glycolysis or gluconeogenesis, except for phosphoglycerate kinase. It is one of seven enzymes common to both glycolysis and gluconeogenesis. The enzymes hexokinase, phosphofructokinase, and pyruvate kinase catalyze irreversible reactions unique to glycolysis. In order for gluconeogenesis to occur, the three irreversible reactions must be replaced. Pyruvate is synthesized into phosphoenolpyruvate by a two-step reaction. First, oxaloacetate is formed by carboxylation in the presence of pyruvate carboxylase. Then, phosphoenolpyruvate carboxykinase decarboxylates and phosphorylates oxaloacetate in the presence of GTP. The next irreversible step to be bypassed in gluconeogenesis requires fructose-6-phosphate to be produced by the action of fructose-1,6-phosphatase on fructose-1,6-phosphate. When glucose-6 phosphate is finally produced during gluconeogenesis, it is converted to glucose by glucose-6-phosphatase, an enzyme unique to the endoplasmic reticulum. The free glucose may then diffuse from the liver into the bloodstream. Of the enzymes given as possible answers, only phosphoglycerate kinase catalyzes a reversible reaction common to both glycolysis and gluconeogenesis.

Which one of the following enzymes is common to both glycolysis and gluconeogenesis? a. Pyruvate kinase b. Pyruvate carboxylase c. Hexokinase d. Phosphoglycerate kinase e. Fructose-1,6-bisphosphatase

*The answer is D.* In order for ribose-5-phosphate to be converted to glucose-6-phosphate, the nonoxidative reactions of the HMP shunt pathway must be used (the oxidative steps are not reversible reactions). In order for this to occur, the ribose-5- phosphate is isomerized to ribulose-5-phosphate, which is then epimerized to xylulose-5-phosphate (steps 1 and 2 in the figure on page 123). R5P and X5P then initiate a series of reactions utilizing transketolase (step 3 in the figure on page 123) and transaldolase (step 4 in the figure on page 123) to generate fructose-6-phosphate, which can be isomerized to glucose-6-phosphate (step 5 in the figure on page 123). Glyceraldehyde-3-phosphate is also formed during this series of reactions, which then goes back to fructose-6-phosphate production. Pyruvate, oxaloacetate, 1,3-bisphosphoglycerate, and 6-phosphogluconate are not obligatory intermediates in this conversion.

Which one of the following is an obligatory intermediate in the conversion of ribose-5-phosphate to glucose-6-phosphate? (A) Pyruvate (B) 1,3-bisphosphoglycerate (C) Oxaloacetate (D) Xylulose-5-phosphate (E) 6-phosphogluconate

*The answer is C.* The other reactions are common to both gluconeogenesis and glycolysis.

Which one of the following reactions is unique to gluconeogenesis? A. Lactate → pyruvate B. Phosphoenolpyruvate → pyruvate C. Oxaloacetate → phosphoenolpyruvate D. Glucose 6-phosphate → fructose 6-phosphate E. 1,3-Bisphosphoglycerate → 3-phosphoglycerate

*The answer is D.* During the overnight fast, glycogen is partially depleted and gluconeogenesis provides blood glucose. Gluconeogenesis is inhibited by fructose 2,6- bisphosphate and stimulated by elevated levels of acetyl CoA. Degradation of fatty acids yields acetyl CoA, which cannot be converted to glucose. This is because there is no net gain of carbons from acetyl CoA in the TCA cycle, and the PDH reaction is physiologically irreversible. Carbon skeletons of most amino acids are, however, gluconeogenic.

Which one of the following statements concerning gluconeogenesis is correct? A. It occurs in muscle. B. It is stimulated by fructose 2,6-bisphosphate. C. It is inhibited by elevated levels of acetyl CoA. D. It is important in maintaining blood glucose during the normal overnight fast. E. It uses carbon skeletons provided by degradation of fatty acids

*The answer is D.* Hexokinase, phosphofructokinase, and pyruvate kinase are all irreversible and are the regulated steps in glycolysis. The conversion of glucose to lactate (anaerobic glycolysis) is a process that does not involve a net oxidation or reduction and, thus, oxygen is not required. Glucokinase (not hexokinase) is important in hepatic glucose metabolism only in the absorptive period following consumption of a carbohydrate-containing meal. Fructose 2,6-bisphosphate is a potent activator (not inhibitor) of phosphofructokinase. The conversion of glucose to lactate yields two ATP but no net production of NADH.

Which one of the following statements concerning glycolysis is correct? A. The conversion of glucose to lactate requires the presence of oxygen. B. Hexokinase is important in hepatic glucose metabolism only in the absorptive period following consumption of a carbohydrate-containing meal. C. Fructose 2,6-bisphosphate is a potent inhibitor of phosphofructokinase. D. The regulated reactions are also the irreversible reactions. E. The conversion of glucose to lactate yields two ATP and two NADH.

*The answer is B.* Hemolytic anemia would be expected in pyruvate kinase deficiency. A lack of pyruvate kinase leads to decreased ATP production. ATP is needed in red blood to supply energy to the Na+/K+ pump and without this, sodium will accumulate inside the cell and there would be a net water movement inside of the cell. This causes the red blood cells to swell up and undergo hemolysis.

Which one of the following would be expected in pyruvate kinase deficiency? (A) Increased levels of lactate in the R.B.C (B) Hemolytic anemia (C) Decreased ratio of ADP to ATP in R.B.C (D) Increased phosphorylation of Glucose to Glucose-6-phosphate

*The answer is B.* The conversion of glucose to glucose-6-phosphate is different in liver and muscle. In muscle and most other tissues, hexokinase regulates the conversion of glucose to glucose-6-phosphate. When the major regulatory enzyme of glycolysis, phosphofructose kinase, is turned off, the level of fructose-6-phosphate increases and in turn the level of glucose-6-phosphate rises because it is in equilibrium with fructose-6- phosphate. Hexokinase is inhibited by glucose-6-phosphate. However, in the liver, glucose is phosphorylated even when glucose-6-phosphate levels are high because the enzyme regulating transformation of glucose into glucose-6-phosphate is glucokinase. Glucokinase is not inhibited by glucose-6-phosphate in the liver. While hexokinase has a low Km for glucose and is capable of acting upon low levels of blood glucose, glucokinase has a high Km for glucose and is effective only when glucose is abundant. Therefore, when blood glucose levels are low, muscle, brain, and other tissues are capable of taking up and phosphorylating glucose, while the liver is not. When blood glucose is abundant, glucokinase in the liver phosphorylates glucose and provides glucose-6-phosphate for the synthesis and storage of glucose as glycogen.

Which reaction in the figure below occurs in both muscle and liver but has substantially different qualities in the two? a. Reaction A b. Reaction B c. Reaction C d. Reaction D e. Reaction E

1. *The answer is A.* Both fatty acids and cholesterol are synthesized from acetyl-CoA in the cytoplasm. Acetyl-CoA, which is produced in the mitochondria, is delivered to these pathways using the citrate shuttle. 2. *The answer is C.* Oxaloacetate, produced from pyruvate, exits the mitochondrion after conversion to malate.

Which shuttle is required for the following? 1. Required for cholesterol and fatty acid synthesis in hepatocytes 2. Required for the hepatic conversion of pyruvate to glucose A. Citrate shuttle B. Glycerolphosphate shuttle C. Malate-aspartate shuttle D. Carnitine shuttle E. Adenine nucleotide shuttle

*The answer is C.* The patient in this vignette has glycogen storage disease II (Pompe disease), which is caused by lysosomal acid maltase deficiency. This disease presents in early infancy with hypotonia, poor feeding, failure to thrive, and cardiac insufficiency that is caused by hypertrophic cardiomyopathy. Hepatomegaly may be present, but is due to cardiac insufficiency. Macroglossia is another clinical feature of this disorder. Deficiency of the enzyme leads to accumulation of glycogen in lysosomes and cytoplasm, resulting in tissue destruction. It is inherited in an autosomal recessive manner.

You are evaluating a 3-month old boy in the emergency department who presents with 2 days of tachypnea. He has no fevers or upper respiratory symptoms. His past medical history is notable for hypotonia, feeding difficulties, and failure to thrive. Both parents have distant family members with similar symptoms in infancy. Vital signs are temperature 98.6F, heart rate 150/minute, blood pressure 78/42 mmHg, respiratory rate is 68/minute, and oxygen saturation is 93% on room air. On physical exam, the infant is floppy. The tongue is slightly enlarged. Heart rate is tachycardic with no murmurs. There is occasional grunting and subcostal retractions. There are diminished breath sounds at the bases of both lungs. A liver edge is palpable 2 cm below the right costal margin. A complete blood count, complete metabolic panel, lactate, and urinalysis are all within normal limits. Serum creatine kinase is elevated. A chest x-ray is notable for an enlarged cardiac silhouette and pulmonary edema. This infant's symptoms are most likely due to a deficiency in which enzyme? A. Phosphofructokinase B. Muscle phosphorylase C. Acid Maltase D. Glucose-6-phosphatase E. Glycogen debrancher

*The answer is A.* The child in this vignette likely has galactosemia, which is caused by a deficiency in the enzyme galactose-1- phosphate uridyltransferase. Symptoms develop in days to weeks after the introduction of lactose-containing infant formula or breastmilk and include lethargy, poor feeding, jaundice, and cataracts. Laboratory findings include: direct hyperbilirubinemia, elevated transaminases, prolonged prothrombin and partial thromboplastic times, hypoglycemia, metabolic acidosis, a positive urine test for reducing substances, and a negative urine test specific for glucose.

You are evaluating a seven-day-old infant in your clinic for a weight check. The patient was born fullterm via spontaneous vaginal delivery following an uncomplicated pregnancy She is exclusively breastfed and initially was nursing very well, but over the past few days she has been refusing some of her feeds and has had frequent episodes of emesis. Her weight is down 14% from her birth weight. On physical exam, her temperature is 37 oC (98.60F), heart rate is 160 beats per minute, respiratory rate is 38 breaths per minute, and the oxygen saturation is 100% on room air. The baby appears lethargic. Her liver is 3 cm below the costal margin and she has mild jaundice. Her urine is negative for ketones and glucose and positive for reducing substances. Her symptoms improve and she starts to gain weight after mom eliminates all dairy products from her diet. Basic laboratory evaluation reveals: What enzyme deficiency is the cause of this child's illness? A. Galactose-1-phosphate uridyltransferase B. Ornithine transcarbamylase C. Alpha-iduronidase D. Phenylalanine hydroxylase E. Cystathionine synthase F. Beta-glucosidase

*The answer is B.* The child in this vignette likely has galactosemia, which is caused by a deficiency in the enzyme galactose-1- phosphate uridylyltransferase- Symptoms develop in days to weeks after the introduction of lactose-containing infant formula or breastmilk and include lethargy, poor feeding jaundice, and cataracts. Laboratory findings include: direct hyperbilirubinemia, elevated transaminases, prolonged prothrombin and partial thromboplastic times, hypoglycemia, metabolic acidosis, a positive urine test for reducing substances, and a negative urine test specific for glucose.

You are evaluating a seven-day-old infant in your clinic for a weight check. The patient was born fullterm via spontaneous vaginal delivery following an uncomplicated pregnancy. She is exclusively breastfed and initially was nursing very well, but over the past few days she has been refusing some of her feeds and has had frequent episodes of emesis. Her weight is down 14% from her birth weight. On physical exam. her temperature is 370C (98.60F), heart rate is 160 beats per minute, respiratory rate is 38 breaths per minute. and the oxygen saturation is 100% on room air. The baby appears lethargic. Her liver is 3 cm below the costal margin and she has mild jaundice. Her urine is negative for ketones and glucose and positive for reducing substances. Her symptoms improve and she starts to gain weight after mom eliminates all dairy products from her diet. Basic laboratory evaluation reveals: What enzyme deficiency is the cause of this child's illness? A. Ornithine transcarbamylase B. Galactose-1-phosphate uridyltransferase C. Beta-glucosidase D. Phenylalanine hydroxylase E. Alpha-iduronidase F. Cystathionine synthase

*The answer is D.* Given the demographics of the patient's ancestry (and the need for obtaining an accurate history), and the fact that the patient is a male, the patient may have glucose-6-phosphate dehydrogenase deficiency (an X-linked disorder). If a person with this enzyme deficiency is given primaquine, which is a strong oxidizing agent, hemolytic anemia is likely to develop. If a physician suspects that a patient may have such an enzymatic deficiency, it is imperative to check before prescribing strong oxidizing agents to the patient, or prescribe another antimalarial prophylaxis that is not a strong oxidizing agent (such as tetracycline). If individuals were deficient in transketolase, pyruvate dehydrogenase, α-ketoglutarate dehydrogenase, or glyceraldehyde-3-phosphate dehydrogenase, red cell lysis would not occur. One should also recall that the red cells lack mitochondria, so these cells do not contain pyruvate dehydrogenase or α-ketoglutarate dehydrogenase.

You are seeing a male patient of African American descent, whose grandparents live in a chloroquine resistant malaria belt in Africa. He wants to visit his grandparents, and you want to give him primaquine as a malaria prophylaxis, but before you do so, you should test the patient for which of the following nonsymptomatic enzymatic deficiencies? (A) Transketolase (B) Pyruvate dehydrogenase (C) α-Ketoglutarate dehydrogenase (D) Glucose-6-phosphate dehydrogenase (E) Glyceraldehyde-3-phosphate dehydrogenase

*The answer is D.* The patient has thiamine deficiency, and because of this, his heart is having trouble generating sufficient energy to effectively pump his blood (due to a reduction in the rate of both pyruvate oxidation and TCA oxidative steps). The resultant congestive heart failure leads to edema in the lower extremities, pulmonary edema, and inability to participate in even mild exercise. The thiamine deficiency has resulted from the patient's poor diet and the effect of ethanol blocking thiamine absorption from the diet. The nervous system also suffers from thiamine deficiency, in which case, neurological signs of the deficiency would be evident. These are not yet observed in this patient. The symptoms observed are not due to niacin deficiency (which are dementia, dermatitis, and diarrhea). The problem is also not due to insufficient energy for the kidney to appropriately fi lter the blood.

You have been following a patient for several years, who has recently become clinically depressed, and is eating very little and drinking alcohol very heavily. He presents to you one day with noticeable swelling of the lower legs, increased heart rate, lung congestion, and complaints of shortness of breath with virtually any activity. These symptoms have come about due to which of the following? (A) Lack of energy to the nervous system due to niacin deficiency (B) Heart has trouble generating energy due to niacin deficiency (C) Lack of energy to the nervous system due to B1 deficiency (D) Lack of energy to the heart due to B1 deficiency (E) Lack of TCA cycle activity in the kidneys, leading to excessive water retention

*The answer is D.* This patient probably has an insulinoma that releases insulin inappropriately at any blood glucose level, which would lead to hypoglycemia. The released insulin would stimulate glucose uptake into the peripheral tissues (muscle and fat), and if the patient had not eaten, blood glucose levels would rapidly fall. The insulin is also inhibiting the liver from producing glucose, either from glycogen degradation or gluconeogenesis, which only compounds the problem. The hypoglycemia and resultant epinephrine release account for all of his symptoms.

Your 20-year-old male patient received a medical discharge from the US Army. He has had multiple episodes of lightheadedness, sweating, fatigue, tremor, and intense hunger. He had one seizure. During two of these episodes, his blood glucose was 40 mg/dL. Which of the following tests could help you diagnose his problem? (A) Fasting blood glucose (B) HbA1c (C) Noncontrast CT scan of the abdomen (D) Blood glucose and insulin levels measured while he was symptomatic (E) Determining the presence of islet cell antibodies

*The answer is B.* Metformin leads to a reduction of hepatic gluconeogenesis. This is accomplished through the activation of the AMP-activated protein kinase, which phosphorylates and sequesters within the cytoplasm TORC2, which is a coactivator of CREB activity (a transcription factor needed for expression of two gluconeogenic enzymes, PEP carboxykinase and glucose-6-phosphatase). Thus, when TORC2 is absent from the nucleus, gluconeogenesis is impaired as the synthesis of two key enzymes is greatly reduced. One of the major gluconeogenic precursors is lactate, generated from the red blood cells and exercising muscle. In the Cori cycle, two lactates are converted to one glucose, which is then exported. If gluconeogenesis is blocked, lactate is not utilized and its levels can increase, and potentially lead to lactic acidosis. However, in the absence of congestive heart failure or renal insufficiency, this does not occur. The heart, with its massive amount of muscle and mitochondria, can utilize the lactate for energy unless the heart is dysfunctional or has lost muscle mass. Good, functional kidneys can also overcome the lactate imbalance caused by metformin treatment. Metformin does decrease the insulin resistance, but this does not increase lactate in the aerobic state. Metformin does not inhibit the TCA cycle, glycolysis, or dietary protein absorption.

Your obese patient has type 2 diabetes mellitus and you have started him on metformin. One of the possible complications of metformin therapy is lactic acidosis. Why is this a concern with metformin therapy? (A) Metformin reduces insulin resistance (B) Metformin blocks hepatic gluconeogenesis (C) Metformin blocks the TCA cycle (D) Metformin inhibits glycolysis (E) Metformin inhibits dietary protein absorption

*The answer is B.* Marathon runners deplete their stores of glycogen during a race and need to catabolize other sources for energy to continue running. In the vernacular of the sport, when all the glycogen stores are exhausted, the runner "hits the wall." This is usually somewhere around mile 20. Research has shown that proper "carb loading" prior to a race can increase body stores of glycogen and increase performance. Though it is a small increase (1% to 2%), it has been documented repeatedly in research studies even in highly trained athletes. Therefore, it is not a myth. To properly carbohydrate load, one must deplete glycogen stores with very vigorous exercise about 2 to 3 days prior to a race. This stimulates glycogen synthase which increases glycogen stores over the next 2 to 3 days before it returns to baseline levels. This is a critical step in the process of "overbuilding" glycogen stores. This is the step the patient is not doing properly. Vigorous exercise cannot then be continued during the 2 to 3 days of glycogen building or the glycogen stores will be utilized. Pancakes, potatoes, brown rice, and pasta are excellent sources of simple carbohydrates.

Your patient is a marathon runner and has visited your office to ask you about carbohydrate loading to increase his performance during a race. For a full week prior to a race, he eats three meals a day of pancakes, potatoes, brown rice, and pasta and does not exercise at all. He has not noticed any success with this regimen. Which of the following answers best explains why he is getting no benefit from his "carb loading"? (A) Carbohydrate loading is a myth (B) He is not depleting glycogen stores prior to loading (C) He is not on the carbohydrate loading diet long enough prior to the race (D) He is eating the incorrect foods for carbohydrate loading (E) He is too highly trained as an athlete for anything to increase his performance


Kaugnay na mga set ng pag-aaral

PSI Real Estate - Chapter 6: Financing

View Set

Taxes, Retirement, and other Insurance Concepts

View Set

Nursing Diagnosis Prioritization Worksheet

View Set